0% found this document useful (0 votes)
291 views186 pages

Breast Augmentation

ASPS

Uploaded by

omar waleed
Copyright
© © All Rights Reserved
We take content rights seriously. If you suspect this is your content, claim it here.
Available Formats
Download as PDF, TXT or read online on Scribd
0% found this document useful (0 votes)
291 views186 pages

Breast Augmentation

ASPS

Uploaded by

omar waleed
Copyright
© © All Rights Reserved
We take content rights seriously. If you suspect this is your content, claim it here.
Available Formats
Download as PDF, TXT or read online on Scribd
You are on page 1/ 186

Breast Augmentation

In implantation of saline breast prostheses, which of the following fill levels is


most likely to result in rupture due to fold flaw?

(A) Above the manufacturer’s recommended maximum


(B) Below the manufacturer’s recommended minimum
(C) Between the manufacturer’s recommended minimum and maximum
(D) Manufacturer’s recommended maximum
(E) Manufacturer’s recommended minimum

The correct response is Option B.

Based on engineering principles, studies have shown that implant longevity


requires an adequate fill level to decrease fold-flaw failures and premature failures
that result from underfilling (filling at or below the manufacturer’s recommended
minimum level). They have also shown that filling the implants to their least-
wrinkled fill level increases implant longevity and decreases premature failure.
This generally requires overfilling exceeding the manufacturer’s recommended
maximum level.

At lower fill levels, implants are softer and more sloping in contour but tend to
wrinkle more and have a demonstrably shorter life span because of stress caused
by wrinkling. As implant volume increases, palpable and visible wrinkling
decreases and longevity increases.

References:
1. Dowden RV. Saline breast implant fill issues. Clin Plast Surg. 2001;28:445-450.
2. Dowden RV, Reisman NR. Breast implant overfill, optimal fill, and the standard
of care. Plast Reconstr Surg. 1999;104:1185-1186.

A healthy 24-year-old woman undergoes bilateral cosmetic breast augmentation


with subglandular saline implants. Which of the following percentages best
represents this patient’s 10-year risk for reoperation because of an implant-related
indication?

(A) 5%
(B) 25%
(C) 50%
(D) 75%
(E) 95%
The correct response is Option B.

After breast augmentation with saline implants, the 10-year risk for reoperation for
any implant-related indication is about 25%. Implant-related indications include
deflation of the implant, capsular contracture, hematoma, wound infection, and
seroma.

In one multicenter retrospective study of 450 patients with a mean follow-up


period of 13 years, the reoperation rate for implant-related indications was 25.8%.
In another multicenter retrospective study of 504 patients with a mean follow-up
period of 6 years, the rate was 21%. In a third retrospective study of 749 women
with a mean follow-up period of 5 years, the rate was 12% for cosmetic breast
augmentations and 34% for breast reconstructions.

References:
1. Cunningham BL, Lokeh A, Gutowski KA. Saline-filled breast implant safety
and efficacy: a multicenter retrospective review. Plast Reconstr Surg.
2000;105:2143-2151.
2. Gutowski KA, Mesna GT, Cunningham BL. Saline-filled breast implants: a
Plastic Surgery Educational Foundation multicenter outcomes study. Plast
Reconstr Surg. 1997;100:1019-1027.
3. Gabriel SE, Woods JE, O’Fallon WM, et al. Complications leading to surgery
after breast implantation. N Engl J Med. 1997;336:677-682.

A woman comes to the office for consultation regarding explantation of breast


prostheses without replacement. In this patient, which of the following
quantifications best determines whether mastopexy will be needed in addition to
removal of the prostheses?

(A) Amount of breast tissue overlying the prostheses


(B) Degree of preoperative ptosis
(C) Position of the prostheses
(D) Size of the areolae
(E) Type of implants

The correct response is Option B.

The degree of preoperative breast ptosis is the most important factor in


determining whether a patient will need mastopexy after explantation of breast
protheses. Because breast ptosis remains relatively unchanged or worsens
postoperatively, patients with grade II or III ptosis are excellent candidates for
breast contouring procedures.

The amount of breast tissue overlying the prostheses determines the safety of a
breast contouring procedure done simultaneously with explantation. In general, at
least 4 cm of breast tissue should be present to allow for adequate vascularity of
the skin and separated glandular-nipple flap used for breast contouring, as assessed
by the superior and inferior “breast pinch” test.

Important factors in determining the type of mastopexy after explantation include


the position of the prostheses, the size of the areolae, and the type of implants. For
example, if the areolae exceed 50 mm, circumareolar mastopexy is an option.

References:
1. Rohrich RJ, Beran SJ, Restifo RJ. Aesthetic management of the breast
following explantation: evaluation and mastopexy options. Plast Reconstr Surg.
1998;101:827-837.
2. Spear SL, Giese SY, Ducic I. Concentric mastopexy revisited. Plast Reconstr
Surg. 2001;107:1294-1299.

A 32-year-old woman is undergoing breast augmentation. Which of the following


antibiotic solutions is most appropriate for irrigation of the breast pocket?

(A) Bacitracin, cefazolin, gentamicin


(B) Polymyxin B, gentamicin, cefazolin
(C) 10% Povidone-iodine, gentamicin, cefazolin
(D) 50% Povidone-iodine

The correct response is Option A.

Breast pocket irrigation has been advocated for many years to decrease the
incidence of capsular contracture and periprosthetic breast implant infection.
Multiple organisms have been cultured around breast implants, and in vitro studies
have demonstrated that a combination triple antibiotic (10% povidone-iodine,
gentamicin, cefazolin) combination provided improved broad-spectrum activity
against the bacteria commonly cultured around breast implants compared with
other antibiotic combinations, including polymyxin B, gentamicin, and cephazolin.

In 2000, the U.S. Food and Drug Administration approved the premarket
application for saline implants; however, contact of the implant with povidone-
iodine was stated as a contraindication. Subsequent in-vitro studies examined
alternative nonBpovidone-iodine-containing breast pocket irrigation solutions and
similar broad-spectrum antibiotic activity was found with the triple combination of
bacitracin, cefazolin, and gentamicin.
Povidone-iodine (50%) does not provide optimal broad-spectrum activity, and
contact of the implant with povidone-iodine is contraindicated. The triple
combination of 10% povidone-iodine, gentamicin, and cefazolin is a viable
alternative. However, if this combination is to be used, pockets would need to be
irrigated clear after its instillation; therefore, this is not the optimal choice.

The combination of polymyxin B, gentamicin, and cefazolin has been shown in in-
vitro studies to have inferior activity against the common bacteria cultured around
breast implants.

References:
1. Adams WP Jr, Conner WC, Barton FE Jr, et al. Optimizing breast pocket
irrigation: the post betadine era. Plast Reconstr Surg. 2001;107:1596.
2. Adams WP Jr, Conner WC, Barton FE Jr, et al. Optimizing breast pocket
irrigation: an in vitro study and clinical implications. Plast Reconstr Surg.
2000;105:334.

Breast Reconstruction

A 60-year-old woman with breast cancer is scheduled to undergo modified radical


mastectomy followed by postoperative radiation therapy. Which of the following
techniques will yield the most natural appearance of the breast with the fewest
complications?

(A) Delayed reconstruction with an autologous tissue flap


(B) Delayed reconstruction with a tissue expander followed by implantation of a
prosthesis
(C) Immediate reconstruction with an autologous tissue flap
(D) Immediate reconstruction with a tissue expander followed by implantation of a
prosthesis

The correct response is Option A.

Reconstruction of the breast with a delayed (nonradiated) autologous tissue flap


will yield the best breast appearance if postmastectomy chest radiation therapy is
needed.

Radiation therapy negatively affects reconstructive outcomes with implants,


causing a marked increase in capsular contracture and other complications. These
complications are unrelated to the type of implant.

Totally autologous reconstruction is the best option for reconstruction of a radiated


breast. In implant reconstruction of a radiated breast, the latissimus flap can be
used to salvage tissue with periprosthetic contractures, particularly in a patient
who is not a good candidate for a TRAM flap. The final outcome is not as good as
that obtained with autologous tissue alone but can be acceptable.

In one study, patients who had immediate reconstruction with a TRAM flap
followed by radiation therapy were compared with patients who had radiation
therapy followed by delayed reconstruction with a TRAM flap. The study found
that the incidence of late complications was significantly higher in the immediate
reconstruction group than in the delayed reconstruction group. In fact, 28% of
patients in the immediate reconstruction group required an additional flap to
correct a distorted contour caused by flap shrinkage or severe flap contraction.
These findings indicate that delayed reconstruction is preferred in patients who are
candidates for breast reconstruction with a free TRAM flap and need
postmastectomy radiation therapy.

References:
1. Spear SL, Onyewu C. Staged breast reconstruction with saline-filled implants in
the irradiated breast: recent trends and therapeutic implications. Plast Reconstr
Surg. 2000;105:930-942.
2. Tran N, Chang D, Gupta A, et al. Comparison of immediate and delayed free
TRAM flap breast reconstruction in patients receiving postmastectomy radiation
therapy. Plast Reconstr Surg. 2001;108(1):78-82.

A 56-year-old woman comes to the office for routine follow-up 10 days after
undergoing skin-sparing right modified radical mastectomy and immediate breast
reconstruction with a subpectoral tissue expander. Physical examination shows a 2
_ 3-cm area of frank necrosis of the lower lateral mastectomy skin flap. The tissue
expander in this area is not covered by muscle, but it is not exposed and there are
no signs of infection. Which of the following is the most appropriate next step in
management?

(A) Admission to the hospital for intravenous administration of antibiotics and


observation
(B) Debridement of necrotic tissue and primary closure
(C) Debridement of necrotic tissue and removal of the tissue expander
(D) Initiation of tissue expansion by instillation of 100 mL of saline into the tissue
expander
(E) Oral administration of an antibiotic and follow-up in one week

The correct response is Option B.

For this patient, the most appropriate next step is to debride the necrotic tissue of
the mastectomy skin flap using sterile technique and then perform primary closure
of the resulting wound. This management minimizes the risk of infection and of
exposure of the tissue expander.
Oral or intravenous administration of an antibiotic does not prevent eventual
exposure and infection of a breast tissue expander that is covered with nonviable
soft tissue. Initiation of tissue expansion in a mastectomy skin flap with frank
necrosis risks dehiscence of the mastectomy wound and exposure of the tissue
expander.

Removal of the tissue expander is not necessary because it is not infected at this
time. If the skin flap necrosis can be debrided while the sterility of the tissue
expander is maintained, the resulting wound can undergo primary closure and the
breast reconstruction can be preserved. If the pectoralis major or serratus anterior
muscle were covering the tissue expander in the necrotic area of the skin flap, this
necrotic tissue could be managed nonoperatively by allowing it to slough and by
performing regular dressing changes while secondary healing occurs.

References:
1. Grotting J. Reoperation following implant breast reconstruction. In: Reoperative
Aesthetic & Reconstructive Surgery. Vol. 2. St. Louis, MO: Quality Medical
Publishing; 1995:1032.
2. Spear S, Howard M, Boehmler J, et al. The infected or exposed breast implant:
management and treatment strategies. Plast Reconstr Surg. 2004;113:1634-1644.

A 14-year-old girl has absence of the nipple and lack of development of the right
breast. The left breast has normal shape and normal nipple-areola complex and fits
a B-cup brassiere. Family history includes normal breast development in the
parents and siblings. On physical examination, both pectoralis muscles are present
and fully developed. No abnormalities of the hands are noted. Which of the
following is the most likely diagnosis?

(A) Anterior thoracic hypoplasia


(B) Congenital absence of the breast
(C) Hypoplasia of the breast
(D) Poland syndrome
(E) Tubular breast deformity

The correct response is Option B.

Congenital absence of the breast is defined by the absence of the nipple and
mammary gland. This rare genetic condition is highly heterogeneous in
presentation and inheritance.

Anterior thoracic hypoplasia is defined by unilateral sinking of the anterior chest


wall, hypoplasia of the breast, superior location of the nipple-areola complex, and
normal pectoralis muscles. In hypoplasia of the breast, both nipples are present. In
Poland syndrome, the defining feature is partial or complete aplasia of the
sternocostal head of the pectoralis muscle and deformity of the breast and upper
extremity. Tubular breast deformity involves constriction at the base of the breast,
hypoplasia of the breast, and herniation of tissue into the nipple-areola complex.

References:
1. Spear SL, Pelletiere CV, Lee ES, et al. Anterior thoracic hypoplasia: a separate
entity from Poland syndrome. Plast Reconstr Surg. 2004;113(1):69-77.
2. Lin KY, Nguyen DB, Williams RM. Complete breast absence revisited. Plast
Reconstr Surg. 2000;106(1):98-101.

A 13-year-old girl is brought to the office by her parents because her left breast is
not developing. On examination, both nipples and areolae are present. The breasts
are asymmetric; the left breast is considerably smaller. The left anterior axillary
fold is absent. This patient is most likely to have which of the following additional
developmental differences?

(A) Ambiguous genitalia


(B) Craniosynostosis
(C) Microtia
(D) Pectus excavatum
(E) Syndactyly

The correct response is Option E.

This patient has Poland syndrome, which is characterized by unilateral breast or


nipple hypoplasia, unilateral absence of the sternal head of the pectoralis major
muscle, absence of the pectoralis minor muscle, and ipsilateral syndactyly or
hypoplasia of the ipsilateral extremity. In severe Poland syndrome, rib anomalies
also occur.

Ambiguous genitalia, craniosynostosis, microtia, and pectus excavatum are not


associated with Poland syndrome.

References:
1. Roth D. Thoracic and abdominal wall reconstruction. In: Aston SJ, Beasley
RW, Thorne CH, eds. Grabb & Smith’s Plastic Surgery. 5th ed. Philadelphia:
Lippincott Williams & Wilkins; 1997:1023-1030.
2. Bostwick J. Reconstructive problems. In: Bostwick J, ed. Plastic and
Reconstructive Breast Surgery. Vol 2. 2nd ed. St. Louis, MO: Quality Medical
Publishing; 1999:1530-1533.

A 56-year-old woman with recurrent cancer of the right breast and no evidence of
distant metastatic disease is scheduled to undergo completion mastectomy five
years after undergoing lumpectomy (segmental mastectomy) for stage II
carcinoma with radiation. In this patient, risk of complications is highest with
which of the following methods of immediate breast reconstruction?

(A) Free superior gluteal artery perforator (SGAP) flap


(B) Free TRAM flap
(C) Latissimus dorsi myocutaneous flap with implant
(D) Pedicled TRAM flap
(E) Tissue expansion

The correct response is Option E.


In a patient with a previously irradiated breast, reconstruction with a tissue
expander followed by a permanent implant (expander/implant) has a higher rate of
complications than does reconstruction with any autologous tissue.

The latissimus dorsi myocutaneous flap, pedicled TRAM flap, free TRAM flap,
and free SGAP flap are autologous methods of breast reconstruction that bring in
new unradiated tissue to reconstruct the breast.

One study showed that previous radiation exposure was a significant risk factor for
major complications after breast reconstruction with an expander/implant but not
after reconstruction with a TRAM flap. Another study showed that breast cancer
patients who underwent radiation and reconstruction with an expander/implant had
a significantly higher rate of complications than patients who had radiation and
breast reconstruction with a TRAM flap. These findings remained the same
whether radiation exposure occurred before or after breast reconstruction.

References:
1. Bostwick J. Tissue expansion reconstruction. In: Bostwick J, ed. Plastic and
Reconstructive Breast Surgery. 2nd ed. St. Louis, MO: Quality Medical
Publishing; 1999:818-1420.
2. Lin K, Johns F, Gibson J, et al. An outcome study of breast reconstruction:
presurgical identification of risk factors for complications. Ann Surg Oncol.
2001;8:586-591.
3. Chawla A, Kachnic L, Taghian A, et al. Radiotherapy and breast reconstruction:
complications and cosmesis with TRAM versus tissue expander/implant. Int J
Radiat Oncol Biol Phys. 2002;54:520-526.

Breast Reduction

On the basis of evaluation of cutaneous light-pressure thresholds of the breast with


Semmes-Weinstein monofilaments, which of the following areas of the breast is
most sensitive?

(A) Areola
(B) Inferior quadrants
(C) Nipple
(D) Superior quadrants

The correct response is Option D.

Based on the evaluation of cutaneous light-pressure thresholds with the Semmes-


Weinstein monofilaments, there are marked differences in sensory perception
between the skin of the breast, the areola, and the nipple. Irrespective of breast
size, the skin in the superior quadrant is the most sensitive part of the breast, the
areola is less sensitive, and the nipple is the least sensitive part to light pressure.
The fact that the nipple is the least sensitive area in the female breast is thought to
allow the mother to nurse without discomfort. Vibration is most sensitive in the
areola.
Larger breasts are significantly less sensitive than smaller breasts in all anatomic
areas, and there is a significant decrease of sensibility with increasing breast
ptosis. Sensitivity tends to decrease with age.

References:
1. Tairych GV, Kuzbari R, Rigel S, et al. Normal cutaneous sensibility of the
breast. Plast Reconstr Surg. 1998;102(3):701-704.
2. Courtiss EH, Goldwyn RM. Breast sensation before and after plastic surgery.
Plast Reconstr Surg. 1976;58:1-13.
3. Slezak S, Dellon AL. Quantitation of sensibility in gigantomastia and alteration
following reduction mammaplasty. Plast Reconstr Surg. 1993;91:1265-1269.

A 15-year-old girl has a one-year history of asymmetric enlargement of the left


breast. Physical examination shows a 10-cm mass as well as ptosis, marked
stretching of the nipple-areola complex, skin ulceration superolateral to the nipple,
and presence of a prominent, dilated vein. Mammography and ultrasonography
show a dense, circumscribed, homogeneous mass occupying the entire breast.
Which of the following is the most appropriate management?
(A) Enucleation of the mass
(B) Hormone therapy
(C) Lumpectomy and postoperative radiation therapy
(D) Reduction mammaplasty
(E) Total mastectomy and reconstruction

The correct response is Option A.

This patient has giant fibroadenoma, which is best managed by enucleation of the
mass. A large breast lesion in a female adolescent may result from giant
fibroadenoma, phyllodes tumor, and juvenile breast hypertrophy. Fibroadenoma is
the most common breast neoplasm in adolescents; giant fibroadenoma is
characterized by a lesion larger than 5 cm in diameter, presentation at or soon after
puberty, and a short doubling time. Usually, the lesion is solitary, firm, and
nontender and causes rapid, asymmetric enlargement of the breast, prominent
overlying veins, and occasionally skin ulceration caused by pressure. Giant
fibroadenomas are benign lesions that can be excised by enucleation with minimal
risk of recurrence. They do not require mammaplasty, lumpectomy, mastectomy,
or hormone or radiation therapy.

Phyllodes tumors are large, benign tumors that occur primarily in perimenopausal
patients. They are histologically distinct from giant fibroadenomas and rarely
affect adolescents. They are treated with local excision or mastectomy.

Juvenile breast hypertrophy is a rare but well-described disorder in young girls


early in puberty. It presents as diffuse enlargement of the breast without nodules or
discrete masses. It is treated by reduction mammaplasty.

References:
1. McGrath MH. Benign tumors of the teenage breast. Plast Reconstr Surg.
2000;105:218-222.
2. Souba WW. Evaluation and treatment of benign breast disorders. In: Bland KI,
Copeland EM, eds. The Breast: Comprehensive Management of Benign and
Malignant Diseases. Vol. 1. Philadelphia: WB Saunders; 1991:715-729.
3. Rohrich RJ, Thornton JF, Sorokin ES. Recurrent mammary hyperplasia: current
concepts. Plast Reconstr Surg. 2003;111:387-393.

A 20-year-old woman who comes to the office for consultation regarding


reduction mammaplasty inquires about ability to breast-feed after surgery. The
most appropriate response to this patient is that breast-feeding is possible after
each of the following techniques EXCEPT

(A) free nipple


(B) inferior pedicle
(C) liposuction
(D) McKissock bipedicle
(E) vertical

The correct response is Option A.

In the reduction mammaplasty technique using free nipple grafting, the nipple is
totally removed from the breast and placed as a skin graft in a new, superior
location. Therefore, lactation is no longer possible because the milk ducts are no
longer connected to the nipple.

If suction lipectomy or pedicled techniques are used, breast tissue is preserved


behind the nipple and the collecting ducts are still attached to the nipple; therefore,
lactation is possible. Even with these techniques, the success of breast-feeding still
varies with societal influences, patient desires, amount of tissue resected, and
hormonal production of engorgement and milk.

In one study of 78 patients having babies after reduction, 29% breast-fed their
children, 18% attempted breast-feeding and were unsuccessful, and 52% did not
wish to breast-feed. In a study of 49 Brazilian women who had undergone
reduction mammaplasty (breast-feeding is more common in Brazil), 58% breast-
fed compared with 94% of nonoperated control subjects. The length of time that
children were breast-fed was shorter in the reduction mammaplasty patients. In a
third study of 30 women, 93% wished to breast-feed and were successful, although
many of the babies required complementary formula feedings.

References:
1. Harris L, Morris SF, Freiburg A. Is breast feeding possible after reduction
mammaplasty? Plast Reconstr Surg. 1992;89(5):836-839.
2. Souto GC, Giugliani ER, Giugliani C, et al. The impact of breast reduction on
breastfeeding performance. J Hum Lact. 2003;19(1):43-49.
3. Marshall DR, Callan PP, Nicholson W. Breastfeeding after reduction
mammaplasty. Br J Plast Surg. 1994;47(3):167-169.

Burns

According to the American Burn Association guidelines, which of the following


patients has a major burn injury requiring triage to a specialized burn center?
(A) 4-year-old boy with partial-thickness burn on the dorsal aspect of the forearm
(B) 16-year-old girl with partial-thickness burn over the shoulders and upper back
(C) 40-year-old man with type 1 diabetes mellitus and full-thickness perineal burn
(D) 55-year-old woman with full-thickness burns of the volar aspect of the right
arm

The correct response is Option C.

The major burn criteria of the American Burn Association identify individuals,
such as those with diabetes mellitus, who need specialized treatment because they
are at high risk for postburn morbidity and mortality. The criteria for triage to a
specialized burn center include:

$ second- or third-degree burn over more than 10% of the total body surface area
(BSA) in a patient younger than 10 years or older than 50 years
$ second- or third-degree burn over more than 20% of the total BSA in a patient of
any other age
$ significant burn of the face, hands, feet, genitalia, perineum, or skin over major
joints
$ third-degree burn over more than 5% of the total BSA in a patient of any age
$ burn with concomitant inhalation injury, significant electrical injury including
lightning strike, or significant chemical injury
$ burn with a significant preexisting medical disorder that complicates
management, such as diabetes mellitus or heart disease
$ burn with concomitant trauma
$ burn in a patient who might have special social or emotional needs or require
long-term support, as in a child who has been abused or neglected

References:
1. Minanov OP, Peterson P. Burn injury. In: Georgiade GS, Riefkohl R, Levin LS,
eds. Georgiade Plastic, Maxillofacial, and Reconstructive Surgery. 3rd ed.
Baltimore: Williams & Wilkins; 1997:198.
2. Pruitt B, Goodwin C, Mason A. Epidemiological, demographic and outcome
characteristics of burn injury. In: Herndon D, ed. Total Burn Care. 2nd ed.
Philadelphia: WB Saunders; 2001.
A 24-year-old man has a mentosternal contracture (shown above) one year after
sustaining burns over 90% of the total body surface area. Which of the following
reconstructive interventions is the most appropriate management of the
contracture?

(A) Scar release and coverage with a dorsal scapular island flap
(B) Scar release and coverage with a free scapular flap
(C) Scar release and skin graft coverage with a thin split-thickness skin graft
harvested from the scalp
(D) Scar release and use of the dermal regeneration template (Integra)

The correct response is Option D.

A mentosternal contracture usually requires a wide scar release and extensive


tissue coverage. However, this patient who has burns over nearly the total body
surface area has limited donor sites. Therefore, the dermal regeneration template
should be used with thin split-thickness grafting. This reconstructive intervention
provides an acceptably low rate of long-term recurrence of contracture and it uses
available donor sites.

Scar release and coverage with a thin split-thickness skin graft have an
unacceptable rate of recurrence of contracture. Scar release and coverage with a
free scapular flap or dorsal scapular island flap are ideal options for reconstruction.
However, unburned scapular skin is not likely to be available in a patient with
burns over 90% of the total body surface area.

References:
1. Frame JD, Still J, Lakhel-LeCoadou A, et al. Use of dermal regeneration
template in contracture release procedures: a multicenter evaluation. Plast
Reconstr Surg. 2004;113(5):1330-1338.
2. Angrigiani C, Grilli D, Karanas YL, et al. The dorsal scapular island flap: an
alternative for head, neck, and chest reconstruction. Plast Reconstr Surg.
2003;111(1):67-78.

Topical silver sulfadiazine may produce which of the following sequelae?

(A) Carbonic anhydrase inhibition


(B) Granulocyte reduction
(C) Methemoglobinemia
(D) Staining of the skin on contact

The correct response is Option B.

Silver sulfadiazine (Silvadene) is a commonly used topical burn agent. It may


result in granulocyte reduction (neutropenia and thrombocytopenia). Carbonic
anhydrase inhibition may occur with mafenide acetate (Sulfamylon), resulting in
metabolic acidosis. Silver nitrate is an excellent topical agent and has no gram-
negative resistance; however, brown staining of skin and equipment is common
and methemoglobinemia may rarely occur.

References:
1. Heggers J, Hawkins H, Edgar P, et al. Treatment of infections in burns. In:
Herndon D, ed. Total Burn Care. 2nd ed. Philadelphia: WB Saunders; 2001.
2. Moncrief JA, Lindberg RB, Switzer WE, et al. The use of a topical sulfonamide
in the control of burn wound sepsis. J Trauma. 1966;6(3):407-414.

A 30-year-old man sustains burns over 42% of the total body surface area and is
resuscitated using the Parkland (Baxter) burn formula. Five percent albumin is
added to the resuscitation fluid 24 hours after the injury to achieve which of the
following?

(A) Maintenance of intravascular volume


(B) Normalization of intravascular pH
(C) Nutritional support
(D) Provision of coagulation cofactors
The correct response is Option A.

Acute burn resuscitation using the Parkland (Baxter) formula is based on the
patient’s physiologic response to injury. Burns cause a leak in the capillary
endothelium, which results in excessive protein loss. By 24 hours after the burn
injury, the capillary leak is largely resolved. At that time, 5% albumin is added to
the resuscitation fluid to help maintain intravascular volume.

References:
1. Baxer CR. Fluid volume and electrolyte changes in early postburn period. Clin
Plast Surg. 1974;1:693.
2. Kao CC, Garner WL. Acute burns. Plast Reconstr Surg. 2000;105(7):2482-
2492.

An 87-year-old man sustained a third-degree scald injury to the proximal forearm


one hour ago. Early excision of the wound is performed, and the resultant 15 H 15-
cm defect is covered using the dermal regeneration template (Integra). On which
postoperative day should removal of the top silicone layer and placement of an
autograft be performed?

(A) 1
(B) 7
(C) 14
(D) 21
The correct response is Option D.

Integra consists of a collagen-glycosaminoglycan layer covered by a silicone


occlusive layer and can be used as a dermal substitute until it is replaced by the
host’s own fibroblasts and endothelial cells. After the collagen-glycosaminoglycan
layer is fully revascularized, the silicone layer can be removed and an autograft
can be used for epidermal closure. This typically occurs at three weeks (on
postoperative day 21) but may be done later, when full revascularization is
evident.

Before postoperative day 21, revascularization is not adequate. In fact, the


Moiemen, Staiano, and Ojeh study found that full revascularization was more
likely to occur at four weeks.
References:
1. Moiemen NS, Staiano JJ, Ojeh NO, et al. Reconstructive surgery with a dermal
regeneration template: clinical and histologic study. Plast Reconstr Surg.
2001;108(1):93-103.
2. Dantzer E, Braye FM. Reconstructive surgery using an artificial dermis
(Integra): results with 39 grafts. Br J Plast Surg. 2001;54(8):659-664.

A 45-year-old farmer has worsening ulceration of the right cornea two days after
anhydrous ammonia was splashed in his eyes. Immediately after this accident, the
patient’s eyes were washed with saline for 45 minutes. Which of the following is
the most appropriate explanation for the worsening of this patient=s condition?

(A) Bacterial infection


(B) Heat from the examining light
(C) Liquefaction necrosis
(D) Scar tissue

The correct response is Option C.

Liquefaction necrosis is the most likely cause of the worsening eye injury.
Because anhydrous ammonia is an alkaline solution, it can denature and dissolve
proteins and lyse cell membranes. This increases the penetration of the alkaline
solution into the eye, furthering the damage.

If a bacterial infection occurs, it usually begins more than two days after the
injury. Examination of the eyes with a light is unlikely to cause further damage.
Scar tissue is unlikely to develop within two days.

References:
1. Sanford AP, Herndon DN. Chemical burns. In: Herndon D, ed. Total Burn Care.
2nd ed. Philadelphia: WB Saunders; 2001:475-480.
2. Wright KW. Ocular trauma. In: Wright KW, ed. Textbook of Ophthalmology.
Baltimore: Williams & Wilkins; 1997:889-897.
A 35-year-old man is brought to the emergency department after sustaining burns
covering 40% of the total body surface area (TBSA). Physical examination shows
burns to the face and chest, facial edema, and singed nasal hairs. He has stridor;
respirations are 24/min. With the patient receiving 100% oxygen, pulse oximetry
is 98%; arterial blood gas analysis shows a carbon monoxide level of 30%. Which
of the following is the most appropriate next step in management?

(A) Bronchoscopy
(B) Cricothyroidotomy
(C) Escharotomy of the chest
(D) Hyperbaric oxygen therapy
(E) Intubation

The correct response is Option E.

Smoke inhalation and potential carbon monoxide poisoning should be suspected in


any patient who has sustained facial burns and has carbon deposits within the
oropharynx, singeing of facial hair, or carbonaceous sputum. If these findings are
present, 100% oxygen should be administered immediately using a non-rebreather
mask. Because pulse oximetry cannot differentiate between oxyhemoglobin and
carboxyhemoglobin, artificially high readings are often obtained even in patients
with carbon monoxide toxicity. The half-life of carboxyhemoglobin at an inspired
oxygen fraction (FIO2) of 0.21 is 240 minutes, but at an FIO2 of 1.0, the half-life
improves to 30 minutes.

Any patient who has a carbon monoxide level of 30% or higher on arrival in the
emergency department, even after receiving supplemental oxygen, has suffered
severe toxicity. Intubation should be performed immediately because coma and
respiratory depression may occur with levels of 40% to 50%, and death can occur
at levels of 50% or higher. Patients who have dyspnea, stridor, tachypnea, and/or
swelling of the tongue or oropharynx, or who are using accessory respiratory
muscles, should also undergo immediate intubation. If progressive swelling of the
airway is a concern during fluid resuscitation, intubation should be considered.

Hyperbaric oxygen may be administered to patients with severe carbon monoxide


poisoning, but, in emergency cases, this time-consuming process is not advocated.

Bronchoscopy or ventilation scanning should be performed to confirm smoke


inhalation, but is less urgent than management of the airway in this patient.
Cricothyroidotomy is indicated only in patients with severe swelling of the airway
when intubation is not possible.

Chest escharotomy is indicated in patients with respiratory compromise and


circumferential chest burns, but should not be performed before breathing has
improved.

References:
1. Kao CC, Garner WL. Acute burns. Plast Reconst Surg. 2000;105:2482-2492.
2. Yowler CJ, Fratianne RB. Current status of burn resuscitation. Clin Plast Surg.
2000;27:1-10.

In a patient who sustained burns over 35% of the total body surface area four
hours ago, which of the following is the most important factor in development of
shock?

(A) Dilation of the peripheral vasculature


(B) Hypovolemia
(C) Myocardial depression
(D) Paralytic ileus
(E) Renal failure

The correct response is Option B.

In an untreated major thermal burn, the most important factor in the initial
development of shock is hypovolemia. Thermal injury disrupts capillary
endothelial integrity and alters membranes. In a major burn, these actions occur
even in unburned tissue because of circulatory and microcirculatory dysfunction.
They lead to plasma leakage from the circulation, which results in decreased
plasma volume, cardiac output, and urine output and increased systemic vascular
resistance. Local and systemic inflammatory mediators may play a role in these
processes.

Dilation of the peripheral vasculature can cause shock later if sepsis occurs but is
not a factor in the initial development of shock. Myocardial depression can
develop later in shock if a systemic inflammatory response, severe hypovolemia,
or sepsis occurs. However, it is not a factor initially. Paralytic ileus is a systemic
result of a major burn but is not a factor in the initial development of shock. Renal
shutdown is an effect of initial shock, not a factor in its development, but can
occur from sustained hypovolemia.

References:
1. Kramer GC, Lund T, Herndon DN. Pathophysiology of burn shock and burn
edema. In: Herndon D, ed. Total Burn Care. 2nd ed. Philadelphia: WB Saunders;
2001:78-87.
2. Press B. Thermal, electrical and chemical injuries. In: Aston SJ, Beasley RW,
Thorne CH, eds. Grabb & Smith’s Plastic Surgery. 5th ed. Philadelphia: Lippincott
Williams & Wilkins; 1997:161-190.

A 55-year-old man who weighs 90 kg (198 lb) is brought to the emergency


department eight hours after sustaining first-degree burns to the head and neck and
second- and third-degree burns to the entire anterior trunk and both lower
extremities. According to the Parkland formula, the most appropriate management
is fluid resuscitation with administration of lactated Ringer’s solution for the next
eight hours at a rate of how many milliliters per hour?

(A) 607 mL/hr


(B) 709 mL/hr
(C) 1215 mL/hr
(D) 1823 mL/hr
(E) 2127 mL/hr

The correct response is Option D.

In a patient who has a second- and/or third-degree burn injury that covers more
than 20% of the total body surface area (TBSA), acute fluid resuscitation should
be performed with administration of lactated Ringer’s solution during the initial 24
hours after injury. The Parkland formula is used to estimate the amount of fluid
required. According to this formula, lactated Ringer’s solution (4 mL/kg/% TBSA
burned) should be administered during the first 24 hours. A total of 50% of the
solution should be administered during the first eight-hour period and the
remaining 50% over the next 16 hours.

The TBSA involved in a burn can be calculated using the “rule of nines.”
According to this rule, the anterior trunk, the posterior trunk, and each lower
extremity are assigned values of 18%. Each upper extremity and the head have
values of 9%, and the neck has a value of 1%. In this patient, the burn of the
anterior trunk is assigned a value of 18%, and each lower extremity burn is 18%,
for a TBSA burn of 54%. First-degree burns, such as those of the head and neck,
are not included in the TBSA calculation.

A 90-kg patient who has burns involving 54% TBSA will require 19,440 mL of
fluid during the first 24 hours: 9720 mL during the first eight hours and 4860 mL
in both the second and third eight-hour periods. Because he received no fluid
during the first eight hours immediately after injury, 14,580 mL of lactated
Ringer’s solution (9720 mL + 4860 mL) should be administered over the next
eight hours to adequately resuscitate this patient. Divided into eight-hour totals,
the solution is infused at a rate of 1823 mL/hr.

References:
1. Salisbury RE. Thermal burns. In: McCarthy JG, May JW, Littler JW, eds.
Plastic Surgery. Vol 1. Philadelphia: WB Saunders; 1990:791-795.
2. Warden GD. Fluid resuscitation and early management. In: Herndon D, ed.
Total Burn Care. 2nd ed. Philadelphia: WB Saunders; 2001:90-91.

A 38-year-old man sustained frostbite of the right hand four days ago.
Examination shows necrosis distal to the metacarpophalangeal (MP) joints. Three-
phase bone scanning shows viability of the proximal phalanx of each finger.
Which of the following is the most appropriate management?

(A) Amputation at the level of the distal interphalangeal joints and primary closure
(B) Amputation at the level of the MP joints and primary closure
(C) Amputation at the level of the MP joints and radial forearm flap reconstruction
(D) Amputation at the level of the proximal interphalangeal (PIP) joints and groin
flap reconstruction
(E) Amputation at the level of the PIP joints and second metacarpal artery flap
reconstruction

The correct response is Option D.


Because frostbite injuries can cause devastating loss of tissue, aggressive
management is often needed to salvage and provide vascularized coverage over
viable bone. The level of skin loss does not always correlate with the level of bone
viability. A three-phase bone scan can be used to determine at which level the
bone is viable. Amputation of nonviable bone is performed along with amputation
of the nonviable soft tissue. To salvage the bone, well-vascularized coverage is
required, with either a pedicle flap or a free flap.

Amputation at the level of the distal interphalangeal joints and primary closure
would leave the middle phalanx nonvascularized. Amputating at the MP joint
would sacrifice viable bone. Amputation at the level of the MP joints and radial
forearm flap reconstruction would sacrifice viable bone, and the radial forearm
flap would not be needed for coverage. Amputation at the level of the PIP joints
and second metacarpal artery flap reconstruction is inappropriate because a second
metacarpal artery flap would be of insufficient length to cover the open area.

References:
1. Leonard LG, Daane SP, Sellers DS, et al. Salvage of avascular bone from
frostbite with free tissue transfer. Ann Plast Surg. 2001;46(4):431-433.
2. Greenwald D, Cooper B, Gottlieb L. An algorithm for early aggressive
treatment of frostbite with limb salvage directed by triple-phase scanning. Plast
Reconstr Surg. 1998;102(4):1069-1074.

Cleft Lip

A 6-year-old boy with velopharyngeal incompetence is scheduled to undergo


sphincter pharyngoplasty. He underwent cleft palate repair in infancy. In the
sphincter pharyngoplasty procedure, which of the following muscles is elevated in
the posterior tonsillar pillar?

(A) Levator veli palatini


(B) Palatoglossus
(C) Palatopharyngeus
(D) Stylopharyngeus
(E) Superior pharyngeal constrictor

The correct response is Option C.

The levator veli palatini originates from the petrous portion of the temporal bone
and the eustachian tube. It travels alongside the eustachian tube to enter the soft
palate. This muscle elevates the velum toward the posterior pharyngeal wall to
close the velopharyngeal mechanism and pull the eustachian tube open.

The palatoglossus muscle arises from the lateral margin of the tongue. It travels in
the anterior tonsillar pillar to enter into the soft palate and functions to pull the soft
palate downward.

The posterior tonsillar pillar is created by the palatopharyngeus. The


palatopharyngeus muscle arises from fibers that commingle with the superior
pharyngeal constrictor. It enters the soft palate, where its fibers intermingle with
those of the levator. The palatopharyngeus functions to depress the soft palate and
provide inward motion of the lateral pharyngeal wall. In a sphincter
pharyngoplasty, superiorly based flaps are elevated from the posterior tonsillar
pillar, sutured together, and inset into the posterior pharyngeal wall. These flaps
contain the palatopharyngeus muscle.
The superior pharyngeal constrictor originates from the posterior pharyngeal raphe
and courses downward and forward to insert into the medial pterygoid plate, the
pterygomandibular raphe, and the posterior part of the hyoid. The stylopharyngeal
muscle arises from the styloid process. It inserts between the fibers of the superior
and middle pharyngeal muscles in the pharyngeal wall. These muscles play a role
in swallowing.

References:
1. Clemente CD. Anatomy: A Regional Atlas of the Human Body. 4th ed.
Baltimore: Williams & Wilkins; 1997:435-576.
2. Sloan GM. Posterior pharyngeal flap and sphincter pharyngoplasty: the state of
the art. Cleft Palate Craniofac J. 2000;37:112-122.
3. Wexler A. Anatomy of the head and neck. In: Ferraro JW, ed. Fundamentals of
Maxillofacial Surgery. New York: Springer-Verlag; 1997:53-113.

In neonates with isolated cleft palate, which of the following percentages best
represents the incidence of additional anomalies?

(A) 10%
(B) 30%
(C) 50%
(D) 70%
(E) 90%

The correct response is Option C.

Isolated cleft palate has an incidence of 1:2000 without ethnic preference and has a
higher incidence of associated anomalies than cleft lip and palate (CL/P).
Approximately half of the cases of cleft palate with associated anomalies will fall
into established syndromes. Patients with CL/P are more common than those with
cleft palate alone; the incidence is 1 in 1000 Caucasians, 1 in 2000 African-
Americans, and 1 in 500 Asians. Associated anomalies in children with CL/P is
approximately 10% to 15%. The type of associated anomalies would include
musculoskeletal, cardiac, and neurological.

Both cleft lip and cleft palate are controlled by multifactorial inheritance.
Therefore, there is no distinctive pattern of inheritance within a single family. The
risk to first-degree relatives can be estimated as the square root of the population
risk. The risk is much lower for a second-degree relative. Higher chance of
occurrence is also seen with greater degree of severity in the affected relative. The
risk for development in a sibling of an affected child increases if more than one
family member has the condition.

References:
1. Vander Kolk CA. Cleft palate. In: Achauer BM, Eriksson E, Vander Kolk CA,
et al, eds. Plastic Surgery: Indications, Operations, and Outcomes. Vol 2. St Louis,
MO: Mosby; 2000:799-807.
2. Marazita ML. Genetic etiologies of facial clefting. In: Mooney MP, Siegel MI,
eds. Understanding Craniofacial Anomalies: The Etiopathogenesis of
Craniosynostoses and Facial Clefting. New York: Wiley-Liss; 2003:147-161.

A 10-month-old infant with cleft palate is scheduled to undergo repair via the
Veau-Wardill-Kilner V-Y technique. Which of the following structures will NOT
be directly manipulated during this repair procedure?

(A) Levator veli palatini muscles


(B) Musculus uvulae
(C) Nasal lining
(D) Palatopharyngeus muscles
(E) Tensor veli palatini muscles
The correct response is Option D.

Because cleft palate does not disrupt the palatopharyngeus muscles, they do not
need to be repaired. Cleft palate repair aims to eliminate the oronasal fistula and
optimize the function of the soft palate. The fistula is closed by creating a lining
for the nasal and oral sides of the fistula. The abnormal attachments of the levator
veli palatini and tensor veli palatini muscles to the hard palate are released, and the
muscles are repaired in the midline, giving them a more functional transverse
orientation. Finally, the musculus uvulae is repaired in the midline. This muscle
provides bulk on the upper surface of the soft palate during velopharyngeal
closure. Although the palatopharyngeus muscles are involved in velopharyngeal
closure, they are not disrupted by cleft palate.

References:
1. Bauer BS, Patel PKK. Cleft palate. In: Georgiade GS, Riefkohl RR, Levin LS,
eds. Georgiade Plastic, Maxillofacial and Reconstructive Surgery. 3rd ed.
Baltimore: Williams & Wilkins; 1997:239-246.
2. Vander Kolk CA. Cleft palate. In: Achauer BM, Eriksson E, Guyuron B, et al,
eds. Plastic Surgery Indications, Operations, and Outcomes. Vol. 2. St. Louis, MO:
Mosby; 2000:799-807.

Cosmetic Blepharoplasty, Browlift and Fillers


A 42-year-old woman has diplopia and pain in the left eye six days after
undergoing bilateral upper eyelid blepharoplasty. Which of the following findings
in this patient’s left eye confirms a suspected injury to the left superior oblique
muscle?

(A) Absent adduction


(B) Absent depression, extorsion, and adduction
(C) Absent elevation, intorsion, and adduction
(D) Absent extorsion, elevation, and abduction
(E) Absent intorsion, depression, and abduction

The correct response is Option E.

In this patient, an attempt to resect orbital fat caused injury to the left superior
oblique muscle. Because the superior oblique muscle is primarily responsible for
intorsion, depression, and abduction of the eye (see below), an absence of these
actions confirms suspected injury to this muscle.

Intraocular Primary
Secondary Action
Muscle Action
depression,
superior oblique intorsion
abduction
adduction,
superior rectus elevation
intorsion
medial rectus adduction none
lateral rectus abduction none
adduction,
inferior rectus depression
extorsion
elevation,
inferior oblique extorsion
abduction

References:
1. Doxanas MT, Anderson KL. Clinical Orbital Anatomy. Baltimore: Williams &
Wilkins; 1984.
2. Putterman AM. Cosmetic Oculoplastic Surgery. 3rd ed. Philadelphia: WB
Saunders; 1998.

A 52-year-old man has tearing, burning, and sensation of presence of a foreign


body six months after undergoing bilateral transcutaneous lower eyelid
blepharoplasty. Examination shows normal position of the lower eyelid of the
right eye; examination of the left eye shows 5 mm of inferior scleral show with lid
retraction and lateral ectropion of the lower eyelid. Which of the following is the
most appropriate surgical procedure to correct this patient’s deformity?

(A) Lateral canthoplasty


(B) Lateral tarsal strip with full-thickness skin grafting
(C) Lateral tarsorrhaphy of the left lower eyelid
(D) Tarsal strip with reattachment of the left lower eyelid retractors
(E) Wedge resection of the left lower eyelid

The correct response is Option B.

In this patient, examination shows cicatricial ectropion of the left lower eyelid,
which is associated with shortening of the anterior lamella of the eyelid and
horizontal laxity of the lower eyelid. A tarsal strip with skin grafting is the
treatment of choice for this disorder because it corrects the horizontal and vertical
laxities seen with cicatricial ectropion.

The other surgical procedures are not appropriate for this patient. Lateral
tarsorrhaphy is useful in treating patients with exposure keratopathy and paralytic
ectropion, such as in facial (VII) nerve palsy. Neither lateral tarsorrhaphy nor
lateral canthoplasty addresses the horizontal laxity of the eyelid. Wedge resection
would address the horizontal laxity of the left lower eyelid but not the vertical
shortening of the anterior lamella of the eyelid. Use of a tarsal strip with
reattachment of the lower eyelid retractors is the treatment of choice for
involutional or senile entropion of the lower eyelid, which this patient does not
have.

References:
1. McCord CD, Boswell CB, Hester TR. Lateral canthal anchoring. Plast Reconstr
Surg. 2003;112(1):222-236.
2. Glat P, Jelkes GW, Jelks EB, et al. Evolution of the lateral canthoplasty. Plast
Reconstr Surg 1997;100(6):1396-1405.

Which of the following best differentiates the Asian upper eyelid from the
Occidental upper eyelid?

(A) Fibers of the levator apparatus insert into the orbicularis muscle closer to the
superior tarsal border in the Asian eyelid
(B) Müller’s muscle inserts into the tarsal plate more inferiorly in the Asian eyelid
than in the Occidental eyelid
(C) Orbital septum fuses with the levator aponeurosis cephalad to the superior
tarsal border in the Asian upper lid
(D) Preaponeurotic fat rests in a more caudal position in the Asian, creating the
appearance of a fuller eyelid
(E) Upper eyelid crease is higher in the Asian than the Occidental

The correct response is Option D.

There are several differences in anatomy in the Asian orbital region compared with
the Occidental orbit. These include more shallow orbits, prominent globes, and
epicanthal folds. In the upper eyelid, the orbital septum fuses with the levator
aponeurosis caudal to the superior tarsal border. This allows the preaponeurotic fat
to lie in a more caudal position in the lid, with some fat lying superficial to the
tarsal plate, giving the impression of a fuller upper lid. In addition, the levator
muscle fibers insert into the orbicularis muscle closer to the inferior tarsal border
in the Asian upper lid, causing the lid crease to be much closer to the inferior tarsal
border compared with the Occidental upper lid. There is no racial difference in the
Müller’s muscle.

References:
1. Doxanas MD, Anderson RL. Oriental eyelids. Arch Ophthalmol.
1984;102:1232-1235.
2. Shamoun JM, Ellenbogen R. Blepharoplasty, forehead and eyebrow lift. In:
Georgiade GS, Riefkohl R, Levin LS, eds. Georgiade Plastic, Maxillofacial, and
Reconstructive Surgery. 3rd ed. Baltimore: Williams & Wilkins; 1997:575.

Which of the following anatomic structures is incised during transconjunctival


blepharoplasty?

(A) Arcus marginalis


(B) Capsulopalpebral fascia
(C) Orbital septum
(D) Tarsal plate
(E) Tenon’s capsule

The correct response is Option B.


During transconjunctival blepharoplasty, the incision is placed in the lower
conjunctiva and divides the capsulopalpebral fascia (lower lid retractors).
Dissection proceeds posterior to the orbital septum and tarsal plate. The
transconjunctival approach provides access to deeper structures without a visible
scar and eliminates the need to transect the orbicularis muscle and orbital septum.
It can be useful for cases of steatoblepharon (fat herniation) and exposure to the
orbital floor. However, the resultant loss of orbital and septal tone may contribute
to scleral show and ectropion formation.

The other anatomic structures are not involved in transconjunctival blepharoplasty.


The arcus marginalis is the junction at which the orbital septum and periosteum
meet. An incision just caudal to the arcus marginalis can facilitate exploration of
the orbital floor. The orbital septum is not transected in transconjunctival
blepharoplasty. The capsulopalpebral fascia originates from the inferior rectus
muscle and inserts into the inferior aspect of the tarsal plate. The tarsal plate
provides the structural framework of the eyelids. Tenon’s capsule (bulbar fascia) is
a fascial structure that surrounds the globe and divides the bony orbit in halves.
The anterior half contains the globe; the posterior half consists of fat, nerves,
muscles, and vessels that supply the globe and extraocular muscles.

References:
1. Zide BM, Jelks GW. Surgical Anatomy of the Orbit. New York: Raven Press;
1985:28-30.
2. Tomlinson FB, Hovey LM. Transconjunctival lower lid blepharoplasty for fat
removal. Plast Reconstr Surg. 1975;56:314-318.
3. McCord CD, Shore J. Avoidance of lower lid blepharoplasty. Ophthalmology.
1983;90:1039-1046.

A 57-year-old woman is scheduled to undergo coronal brow lift because she has
deep transverse creases at the level of the radix of the nose as well as glabellar
creases and brow ptosis. For effective reduction of the transverse creases at the
level of the radix, which of the following muscles should be addressed during the
procedure?

(A) Corrugator supercilia


(B) Frontalis
(C) Nasalis
(D) Orbicularis oculi
(E) Procerus

The correct response is Option E.


Transverse creases at the level of the radix of the nose are caused by contraction of
the procerus muscle, which runs perpendicular to these creases from the level of
the nasal bone to the skin of the forehead. Therefore, for effective reduction of this
patient’s transverse creases, this muscle should be addressed.

The corrugator supercilii muscles give rise to oblique and longitudinal lines at the
glabella. The fibers of the procerus muscle blend with the fibers of the frontalis
muscle at the glabella and with the nasalis muscle at the tip of the nose. The
frontalis muscle causes prominent transverse lines on the forehead. The nasalis
muscle runs transverse from the lower nose to the maxilla and affects the lower
nose. The orbicularis oculi muscles cause crow’s feet.

References:
1. Koch RJ, Troell RJ, Goode RI. Contemporary management of the aging brow
and forehead. Laryngoscope. 1997;107(6):710-715.
2. Williams PL, Warwick R, Dyson M, et al, eds. Gray’s Anatomy. 37th ed. New
York: Churchill Livingstone, 1989:572.

A 55-year-old man requests cosmetic blepharoplasty. On examination, the eyelid


skin exhibits moderate thickness, hooding of the upper eyelid skin bilaterally, and
pseudoherniation of intraorbital fat at the lower eyelids. Eyelid excursion is
normal, and the “snap test” is unremarkable. Which of the following is the most
likely diagnosis?

(A) Blepharochalasis
(B) Blepharoptosis
(C) Dermatochalasis
(D) Hypotonia
(E) Proptosis

The correct response is Option C.

Traditionally, the terms “blepharochalasis” and “dermatochalasis” have been used


interchangeably to characterize the baggy eyelid deformity. In reality, they are
different disorders. Blepharochalasis describes the condition resulting form
recurrent bouts of nonspecific inflammatory edema of the eyelids that results in
thinning and redness of the overlying skin. Affected individuals have repeated
episodes of eyelid swelling and thin, excess skin of the upper lids, blepharoptosis,
pseudoepicanthal folds, and disinsertion of the lateral canthal tendon.
Dermatochalasis of the eyelids describes the cosmetic deformity of baggy eyelids
and is common in middle age, when loss of elasticity resulting from the aging
process leads to eyelid skin redundancy, and is usually more pronounced in the
upper eyelids.

The resting position and excursion of the eyelids should be determined during
evaluation of a patient for blepharoplasty. A resting-level difference of 1 mm or
more is usually visible and may require repair. The method of ptosis repair
depends on the degree of the deficit, levator function, and strength of Müller’s
muscle and the levator aponeurosis.

Forward projection of the cornea normally varies less than 3 mm between eyes.
Unilateral proptosis may indicate a retrobulbar mass. Bilateral proptosis may
indicate thyroid disease. In the proptotic eye, skin resection must be more
conservative to avoid excessive widening of the palpebral fissure and increasing
corneal exposure.

The “snap test” can be used to determine the resting tone of the lower lid. If
diminished, even limited skin resection in older patients with weak or stretched
pretarsal orbicularis can produce ectropion if the lid suspensory tone is not
augmented.
References:
1. Callahan MA. Congenital ptosis. In: Nesi FA, Levine MR, Lisman MD, eds.
Smith’s Ophthalmic Plastic and Reconstructive Surgery. 2nd ed. St. Louis, MO:
Mosby-Year Book; 1998:375.
2. Jelks GW, Jelks EB. Preoperative evaluation of the blepharoplasty patient:
bypassing the pitfalls. Clin Plast Surg. 1993;20:213.

A 35-year-old woman comes to the office for evaluation and consultation


regarding removal of wrinkles around the eyes and forehead, especially in the
glabellar region. Physical examination shows that the lateral brow is slightly low.
To raise the lateral brow using botulinum toxin (Botox) therapy, which of the
following muscles must be treated?

(A) Corrugator
(B) Depressor supracilii
(C) Frontalis
(D) Orbicularis oculi
(E) Procerus
The correct response is Option D.
The orbicularis oculi muscle is the primary depressor of the lateral brow.
Therefore, paralysis of this muscle with botulinum toxin raises the lateral brow.
Paralysis of the other muscles listed does not produce this effect. The corrugator
muscle is responsible for producing vertical wrinkles in the glabellar area. The
depressor supracilii muscle is the primary depressor of the medial brow. The
frontalis muscle is the primary elevator of the brow. The procerus muscle is
responsible for producing transverse wrinkles in the glabellar area.

References:
1. Fagien S. Bolox for facial aesthetic enhancement. Plast Reconstr Surg.
2003;112:65-185.
2. Ahn M, Catter M, Maas C. Temporal browlift using Botox. Plast Reconstr Surg.
2003;112:985.

Which of the following characteristics of hyaluronic acid dermal fillers is


responsible for more prolonged results?

(A) Cross-linking
(B) Nonanimal source
(C) Particle size
(D) Viscosity

The correct response is Option A.

The ideal injectable filler is biocompatible, readily available, and easy to inject,
with long-lasting effect and minimal complications. Hyaluronic acid fillers have
less risk of immunogenicity because, in contrast to collagen, hyaluronic acid is
chemically identical across all species. Stabilization (cross-linking) of the
molecule results in improved resistance to degradation without compromising its
biocompatibility. Hyaluronic acid fillers are available from a nonanimal source
(streptococcus) and animal sources (rooster comb). No skin testing is required for
nonanimal sources. The product produced via fermentation results in less
likelihood of contamination with antigenetic proteins, decreasing the risk for
hypersensitivity.
Particle size is important when evaluating dermal fillers and relates more to depth
of injection rather than persistence. Larger particles must be injected deeper into
the dermis or subcutaneously to avoid visibility. Viscosity of the filler has more to
do with ease of injection and pain at the injection site than soft-tissue persistence.
Complications include bruising, redness, swelling, pain, tenderness, and itching.

References:
1. Narins RS, Brandt F, Leyden J, et al. A randomized, double-blind, multicenter
comparison of the efficacy and tolerability of Restylane versus Zyplast for the
correction of nasolabial folds. Dermatol Surg. 2003;29(6):588-595.
2. Duranti F, Salti G, Bovani B, et al. Injectable hyaluronic acid gel for soft tissue
augmentation: a clinical and histological study. Dermatol Surg. 1998;24(12):1317-
1325.
3. Saylan Z. Facial fillers and their complications. Aesth Surg J. 2003;23(3):221-
224.

Which of the following soft-tissue fillers is most effective in achieving permanent


results?

(A) Artecoll
(B) Cymetra
(C) Fascian
(D) Isolagen
(E) Restylane

The correct response is Option A.

Artecoll (Artefill) is a permanent soft-tissue filler composed of small, smooth


microbeads of polymethylmethacrylate (PMMA) in a collagen matrix. The smooth
PMMA beads elicit a soft-tissue response that leads to a permanent soft-tissue
augmentation effect. Artecoll has been used for many years in Europe; however, it
is now recommended for approval in the U.S. marketed under the name Artefill.

Although a permanent result may seem a desirable trait, the use of any permanent
soft-tissue filler must be performed with great care, because any potential
deformity will also be permanent. Furthermore, there may also be long-term issues
including granuloma formation and other delayed soft-tissue responses that may
not be clinically apparent for five to six years postimplantation.

Isolagen is an allogeneic nonpermanent filler. Fascian is a nonpermanent


allogeneic fascial particulate injectable. Restylane is a biologically prepared
hyaluronic acid nonpermanent filler. Cymetra is made of human dermis in
injectable form.

References:
1. Cheng JT, Perkins SW, Hamilton MM. Collagen and injectable fillers.
Otolaryngol Clin North Am. 2002;35:73-85.
2. Saylon Z. Facial fillers and their complications. Aesth Surg J. 2003;23(3):221-
229.

Cosmetic Face Lifts

Which of the following layers of the scalp is analogous to the SMAS layer?

(A) Deep temporal fascia


(B) Galea
(C) Innominate fascia
(D) Parotid-masseteric fascia
(E) Pericranium

The correct response is Option B.

The galea is analogous to the SMAS layer because the galea-frontalis, temporal
parietal fascia, SMAS, orbicularis oculi, and platysma form a continuous single
layer. Awareness of this anatomic relationship is essential to avoiding injury to the
facial (VII) nerve during dissection for a facelift and browlift. The nerve lies just
under the layer of the SMAS, facial muscles, and galea.
The deep temporal fascia, innominate fascia, parotid-masseteric fascia,
pericranium, and cervical fascia are all part of an analogous, deeper anatomic
grouping beneath the more superficial SMAS system.

References:
1. Thorne CHM, Aston SJ. Aesthetic surgery of the aging face. In: Aston SJ,
Beasley RW, Thorne CH, eds. Grabb & Smith’s Plastic Surgery. 5th ed.
Philadelphia: Lippincott Williams & Wilkins; 1997:636.
2. Williams PL, Warwick R, Dyson M, et al, eds. Gray’s Anatomy. 37th ed. New
York: Churchill Livingstone, 1989:570-573.

A 54-year-old woman has had inability to depress the right side of the lower lip for
the past month. This symptom began shortly after she underwent subcutaneous
rhytidectomy with SMAS plication along with submental suction lipectomy.
Which of the following is the most appropriate next step in management?

(A) Exploration of the surgical site for possible transection of a nerve


(B) Follow-up examination in one month
(C) Marginal mandibular nerve grafting
(D) Physical therapy with nerve stimulation
(E) Temporalis muscle transfer to the right oral commissure for facial reanimation

The correct response is Option B.

Because nerves generally are not severed during rhytidectomy, SMAS plication,
and suction lipectomy, this patient=s deficit is most likely the result of
neurapraxia. With neurapraxia, function normally returns spontaneously within
three months. Therefore, a follow-up examination in one month is the most
appropriate step at this time.

Because the facial (VII) nerve travels deep to the muscles of facial animation,
rhytidectomy with dissection in the subcutaneous plane does not pose a risk to the
facial nerve branches. Deeper dissection can be more dangerous and should be
performed with a clear understanding of the anatomy of the facial nerve. The
frontal branch of the facial nerve becomes very superficial as it crosses the
zygomatic arch. The other branches become superficial as they exit from within
the parotid gland. SMAS surgery superficial to the parotid gland is generally safe.
The anterior edge of the parotid gland lies no less than 3.5 cm from the tragus.
Submental suction lipectomy is also very safe and poses minimal risk if the
cannula stays superficial to the platysma. If the cannula passes beneath the
platysma, the marginal mandibular branch of the facial nerve is at risk. Large
studies of patients who have had suction lipectomy have shown a low rate (<1%)
of nerve injury. Nerves and blood vessels are generally not severed during suction
lipectomy.

Physical therapy with nerve stimulation is not necessary because nerve function
will return spontaneously. Surgical exploration is not warranted because of the
very low probability that the nerve has been severed.

Marginal mandibular nerve grafting and temporalis muscle transfer are not
indicated for facial reanimation because the deficit is likely to improve
spontaneously. Also, temporalis muscle transfer to the right oral commissure
would help elevate the commissure but not depress the lip.
References:
1. Dellon AL. Peripheral nerve injuries. In: Georgiade GS, Riefkohl R, Levin LS,
eds. Georgiade Plastic, Maxillofacial, and Reconstructive Surgery. 3rd ed.
Baltimore: Williams & Wilkins; 1997:1011-1013.
2. Dillerud E. Suction lipoplasty: a report on complications, undesired results, and
patient satisfaction based on 3511 procedures. Plast Reconstr Surg. 1991;88:239-
246.
3. Wilhelmi BJ, Mowlavi A, Neumeister MW. The safe face lift with bony
anatomic landmarks to elevate the SMAS. Plast Reconstr Surg. 2003;111(5):1723-
1726.
A 56-year-old woman has a 3-cm area of preauricular skin slough 10 days after
undergoing sub-SMAS rhytidectomy. Which of the following interventions is the
most appropriate initial management?

(A) Observation
(B) Debridement
(C) Flap advancement
(D) Full-thickness skin grafting
(E) Split-thickness skin grafting
The correct response is Option A.

After rhytidectomy, skin slough requires careful observation. The injured skin
forms an eschar that should be left in place until it begins to separate. The
separated eschar may be trimmed as the wound epithelializes, which may take
three to four weeks.

The other interventions are not needed initially. Debridement is indicated if


infection develops beneath the eschar. Skin grafting is used only for large areas of
slough that do not close in a reasonable period of time. After skin laxity has
returned, scar excision and flap advancement may be indicated to improve the
appearance of the scar.

References:
1. Rees TD, Aston SJ, Thorne CHM. Blepharoplasty and facialplasty. In:
McCarthy JG, May JW, Littler JW, eds. Plastic Surgery. Vol 4. Philadelphia: WB
Saunders; 1990:2358-2435.
2. Rees TD. Postoperative considerations and complications. In: Rees TD, ed.
Aesthetic Plastic Surgery. Philadelphia: WB Saunders; 1980:708-727.

Cosmetic Hair

Which of the following terms represents the primary active phase of hair growth?

(A) Anagen
(B) Anaphase
(C) Metaphase
(D) Telogen
(E) Telophase
The correct response is Option A.

Normal hair growth involves two primary phases. The active phase, anagen, is the
phase of hair growth. In this phase, which can last three to five years,
approximately 85% of hair follicles produce hair. The resting phase, telogen,
heralds the loss of the hair shaft and affects approximately 15% of hair follicles at
any given time. Balding occurs when the anagen phase is shortened and the
telogen phase is prolonged.

Anaphase, metaphase, and telophase are all phases in cell division and the
replication of deoxyribonucleic acid. They are not specifically related to hair
growth.

References:
1. Orentreich N. Advances in Biology of Skin. Vol 9. New York: Pergamon Press;
1969.
2. Orentreich N, Durr NP. Biology of scalp hair growth. Clin Plast Surg.
1982;9:197.

Which of the following is the most common cause of male pattern baldness?

(A) Cyclical hair loss


(B) Decreased plasma androgen level
(C) Decreased plasma testosterone level
(D) Increased plasma estrogen level
(E) Inheritance

The correct response is Option E.

The only cause of male pattern baldness is inheritance of an X-linked autosomal


dominant gene. Male pattern baldness is also influenced by other minor genetic
factors. The pattern and timing of baldness and the age of onset of hair loss are
influenced primarily by all of these genetic factors.

Cyclical hair loss is characteristic of normal hair growth. Plasma androgen,


testosterone, and estrogen levels have not been shown to influence the rate or
timing of baldness. However, increased levels of 5_-reductase have been an
isolated finding in the hair follicles of balding patients.
References:
1. Ayres S. Hair transplantation. In: Epstein E, Epstein E Jr, eds. Skin Surgery. 6th
ed. Philadelphia: WB Saunders; 1987.
2. Orentreich N, Durr NP. Biology of scalp hair growth. Clin Plast Surg.
1982;9:197.

Cosmetic Liposuction, Abdomen and Thigh

A healthy 60-year-old woman undergoes suction lipectomy of the lateral thighs


during which a total volume of 2 L is aspirated. During the procedure, 1.5 L of
infiltrate and 1.5 L of crystalloid are administered intravenously. Which of the
following is the most appropriate additional intervention for fluid management?

(A) Administer a 1-L bolus of crystalloid


(B) Administer a 3-L bolus of crystalloid
(C) Administer a diuretic
(D) No further hydration

The correct response is Option A.

Hydration during suction lipectomy is very important to prevent complications.


Too little fluid causes hypotension and too much fluid can cause pulmonary
edema. As larger amounts of infiltrate are used with the tumescent technique, it is
difficult to know how much intravenous fluid to give to the patient.

One study concludes that the combination of infiltrate and intravenous fluids (1.5
L and 1.5 L in this case) should add up to twice the aspirate removed (2 _ 2 L).
Therefore, this patient should receive 1 L of fluid in the recovery area. Careful
monitoring of blood pressure, tissue turgor, and urine output will confirm that the
patient is euvolemic.

Patients should not be discharged before adequate fluids are given to maintain
normal urine output. Large-volume liposuctions may require overnight
monitoring. Administration of a diuretic is inappropriate because the patient is not
overloaded with fluid. A 3-L bolus may overhydrate the patient and lead to
pulmonary edema.

References:
1. Pitman G. Liposuctioning and body contouring. In: Aston SJ, Thorne CHM,
Beasley RW, eds. Grabb and Smith’s Plastic Surgery. 5th ed. Philadelphia:
Lippincott Williams & Wilkins; 1997:669-690
2. Pitman GH. Discussion on “The Role of Subcutaneous Infiltration in Suction
Assisted Lipoplasty: A Review.” Plast Reconstr Surg. 1997;99(2):523-526.
The incidence of complications is highest following abdominoplasty when the
procedure is combined with suction lipectomy of which of the following areas?

(A) Central infraumbilical


(B) Epigastric
(C) Flank
(D) Lateral thigh
(E) Medial thigh

The correct response is Option A.

Suction lipectomy of the central portion of the abdominoplasty flap can lead to
loss of skin in the central and inferior portions of the flap and should be avoided.
A critical step in avoiding skin loss is leaving intact the subcutaneous layer of fat
between the skin and the fascia of Scarpa. Only very limited, cautious suction
lipectomy should be attempted in the central portion of the abdominoplasty flap
and should be limited to globular fat deep to the fascia of Scarpa. The lateral
portions of the abdominoplasty flap can be suctioned deep to the fascia of Scarpa.
The epigastrium may also be suctioned carefully without complications. The hips
and thighs can be suctioned aggressively or defatted directly without concern for
skin loss in the abdominoplasty flap.

References:
1. Matarasso A. Liposuction as an adjunct to a full abdominoplasty. Plast Reconstr
Surg. 1995;95:829.
2. Mladick RA. Body contouring of the abdomen, thighs, hips and buttocks. In:
Georgiade GS, Riefkohl R, Levin LS, eds. Georgiade Plastic, Maxillofacial, and
Reconstructive Surgery. 3rd ed. Baltimore: Williams & Wilkins; 1997:674.

A healthy 30-year-old man is scheduled to undergo suction lipectomy of the trunk


using the superwet technique. When injected into subcutaneous fat with solutions
containing epinephrine, which of the following is the maximum recommended
dose of lidocaine?

(A) 7 mg/kg
(B) 14 mg/kg
(C) 21 mg/kg
(D) 28 mg/kg
(E) 35 mg/kg
The correct response is Option E.

The maximum recommended dose of lidocaine is 35 mg/kg when injected into


subcutaneous fat with solutions containing epinephrine. Doses of up to 50 mg/kg
have been used, but are not generally recommended because of toxicity concerns.

Various individual anesthetic agents and anesthetic combinations are appropriate


for suction lipectomy, depending on the patient=s health, the estimated volume of
aspirate to be removed, and the postoperative discharge plan. For suction
lipectomy, anesthetic agents are added to the wetting solution to provide
preemptive and prolonged postoperative local analgesia. In smaller-volume
suction lipectomy cases, anesthetic infiltration solutions alone may provide
adequate pain relief. In larger-volume suction lipectomy cases, the superwet and
tumescent solutions are often used for sedation, general, or epidural anesthesia to
ensure adequate patient comfort.

Lidocaine with epinephrine is used most often as the anesthetic agent in the
wetting solution. Historically, the recommended dose of lidocaine was less than 7
mg/kg. However, this dose did not take into consideration the slow absorption of
lidocaine from fat, the persistent vasoconstriction caused by epinephrine, and the
lidocaine removed in the suction lipectomy aspirate. All these factors decrease the
risk of systemic toxicity from lidocaine and allow higher doses to be used safely.

References:
1. Iverson RE, Lynch DJ, and the ASPS Committee on Patient Safety. Practice
advisory on liposuction. Plast Reconstr Surg. 2004;113(5):1478-1490.
2. Grazer FM, Grazer JM, Sorenson CL. Suction-assisted lipectomy. In: Achauer
BM, Eriksson E, Vander Kolk C, et al, eds. Plastic Surgery: Indications,
Operations, and Outcomes. Vol 5. St. Louis, MO: Mosby; 2000:2859-2887.

A 45-year-old man who has achieved substantial weight loss from massive obesity
is scheduled to undergo belt lipectomy for circumferential truncal excess. Which
of the following is the most likely postoperative complication?

(A) Deep venous thrombosis


(B) Dehiscence
(C) Infection
(D) Seroma
(E) Skin necrosis
The correct response is Option D.

Compared with traditional abdominoplasty, belt lipectomy (which combines


abdominoplasty with circumferential excision of skin and fat) can provide more
optimal contouring for patients with circumferential truncal excess because it
addresses the trunk as a unit. Patients who have had a substantial weight loss
commonly are candidates for this procedure. After belt lipectomy, care must be
undertaken to monitor for and treat complications. Seroma management is a
significant part of postoperative care for up to one third of patients who undergo
belt lipectomy, and patients should be made aware of the significant morbidity
associated with the procedure.

The other complications listed affect less than 10% of patients. Procedures that
increase intraabdominal pressure, such as hernia repair, increase the risk of deep
venous thrombosis and pulmonary emboli. Deep venous thrombosis and
pulmonary emboli are always possible with long-term anesthesia and
immobilization, but these are fortunately rare. Wound dehiscence is a concern in
belt lipectomy; when the patient flexes at the waist to relieve anterior tension, the
back incision is strained and vice versa. Infection rates are less than 5%. Factors
that may increase the risk of skin necrosis involve interruption of the lateral
intercostal blood supply by lateral skin resection, lack of vascularization across the
midline of the anterior abdominal flap, and excess tension on the abdominal flap.

References:
1. Aly AS, Cram AE, Chao M, et al. Belt lipectomy for circumferential truncal
excess: the University of Iowa experience. Plast Reconstr Surg. 2003;111:398-
413.
2. Lockwood T. Contouring of the arms, trunk, and thighs. In: Achauer BM,
Eriksson E, Vander Kolk C, et al, eds. Plastic Surgery: Indications, Operations,
and Outcomes. Vol 5. St. Louis, MO: Mosby; 2000:2839-2858.

During a medial thigh lift procedure, the incision over the femoral triangle is
dissected more superficially to avoid injury to which of the following?

(A) Femoral artery


(B) Femoral nerve
(C) Femoral vein
(D) Inguinal ligament
(E) Lymphatic plexus

The correct response is Option E.


The lymphatics of the lower extremity travel to the superficial inguinal nodes and
deep inguinal nodes. The deep inguinal nodes are situated just medial to the
femoral vein and receive lymphatics that accompany the femoral vessels, lymph
vessels from the genitalia, and efferents from the superficial nodes. The superficial
nodes receive vessels from the gluteal region, infraumbilical abdominal wall,
perianal region, and external genitalia as well as most of the superficial lymph
vessels of the lower limb. Because superficial lymph vessels are small, they are
prone to injury, which can lead to wound-healing problems such as lymphoceles
and seromas as well as problems associated with recurrent peripheral edema. The
femoral vein, artery, and nerve run deeper, beneath the inguinal ligament at the
level of the incision, significantly deeper than the fragile lymphatics of the
superficial inguinal nodes.

References:
1. Lockwood TE. Body contouring, trunk and thigh lifts. In: Cohen M, ed. Mastery
of Plastic and Reconstructive Surgery. Vol 3. Philadelphia: Lippincott Williams &
Wilkins; 1994:2201-2218.
2. Pitman G. Liposuctioning and body contouring. In: Aston SJ, Beasley RW,
Thorne CH, eds. Grabb & Smith’s Plastic Surgery. 5th ed. Philadelphia: Lippincott
Williams & Wilkins; 1997:669-690.
3. Williams PL, Warwick R, Dyson M, et al, eds. Gray’s Anatomy. 37th ed. New
York: Churchill Livingstone, 1989:812-813, 848-849.

When performing suction lipectomy using the superwet technique, the amount of
blood loss in the suction aspirate is closest to which of the following?

(A) 0%
(B) 10%
(C) 20%
(D) 30%
(E) 40%

The correct response is Option A.

With the superwet technique, blood loss is approximately 1% of the suction


aspirate. This technique uses a 1:1 ratio of subcutaneous infiltrate to aspirate. The
infiltrate consists of saline or Ringer’s lactate solution, epinephrine, and in some
cases lidocaine.

The first method of suction lipectomy, the dry technique, was associated with
blood loss of 20% to 45% in the suction aspirate as well as substantial swelling
and discoloration. It was performed under general anesthesia without infiltration of
subcutaneous solutions before insertion of the suction lipectomy cannula. Except
in limited applications, this approach has been abandoned.

The wet technique is associated with blood loss of 4% to 30% of the aspirate. In
this technique, 200 to 300 mL of infiltrate or wetting solution, with or without
additives, is injected into the operative field before insertion of the suction
lipectomy cannula. Small doses of the vasoconstrictor epinephrine are added to the
infiltrate.

Like the superwet technique, tumescent suction lipectomy is associated with blood
loss of approximately 1% in the suction aspirate. However, it uses more infiltrate,
up to 3 or 4 mL of infiltrate for each planned milliliter of aspirate.

References:
1. Iverson RE, Lynch DJ, and the APSP Committee on Patient Safety. Practice
advisory on liposuction. Plast Reconstr Surg. 2004;113(5):1478-1490.
2. Grazer FM, Grazer JM, Sorenson CL. Suction-assisted lipectomy. In: Achauer
BM, Eriksson E, Vander Kolk C, et al, eds. Plastic Surgery: Indications,
Operations, and Outcomes. Vol 5. St. Louis, MO: Mosby; 2000:2859-2887.

Which of the following structures is LEAST likely to be injured during


brachioplasty?

(A) Basilic vein


(B) Cephalic vein
(C) Intercostobrachial nerve
(D) Medial cutaneous nerve of the forearm

The correct response is Option B.

Because the cephalic vein runs anterior and superior to the dissection planes used
in brachioplasty, it is not likely to be injured during this procedure. The basilic
vein, intercostobrachial nerve, and medial cutaneous nerve of the forearm are
aligned slightly medially and posteriorly along the arm in the area of dissection for
a standard brachioplasty. Therefore, they are susceptible to injury during the
procedure.
References:
1. Vogt P, Baroudi R. Brachioplasty and brachial suction assisted lipectomy. In:
Cohen M, ed. Mastery of Plastic and Reconstructive Surgery. Vol 3. Philadelphia:
Lippincott Williams & Wilkins; 1994:2224-2228.
2. Netter FH. Atlas of Human Anatomy. 2nd ed. East Hanover, NJ: Novartis,
1997:410.
3. Williams PL, Warwick R, Dyson M, et al, eds. Gray’s Anatomy. 37th ed. New
York: Churchill Livingstone, 1989:806, 1132.

Cosmetic Rhinoplasty / Chin

Two months after cosmetic rhinoplasty, a patient has numbness of the nasal tip.
The most likely explanation is injury to which of the following nerves?

(A) Descending branch of the infraorbital


(B) Descending branch of the lesser palatine
(C) External branch of the anterior ethmoidal
(D) Medial branch of the infratrochlear
(E) Medial branch of the nasopalatine

The correct response is Option C.

Knowledge of the innervation of the nose is necessary for local anesthesia in


rhinoplasty and nasal reconstruction.

The anterior ethmoidal nerve enters the nose near the crista galli and has two
branches. The external branch emerges between the nasal bone and the lateral
nasal cartilage and supplies the skin of the nasal tip and alae. It is vulnerable
during tip cartilage dissection. The internal branch of the anterior ethmoidal
supplies sensation to the septum and the internal nasal walls.

The infraorbital nerve supplies sensation to the cheek, lip, lower eyelid, and the
upper gingiva. The nasopalatine runs anteroinferiorly on the nasal septum in a
groove in the vomer. It supplies sensation to the septum and the hard palate.

The lesser palatine innervates the uvula, tonsil, and soft palate. The infratrochlear
nerve supplies the skin of the radix. All of these nerves are branches of the fifth
cranial nerve.

References:
1. Bannister LH, Berry MM, Collins P, et al, eds. Gray=s Anatomy. 38th ed. New
York: Churchill Livingstone, 1995:1233-1234.
2. Oneal R, Izenberg P, Schlesinger J. Surgical anatomy of the nose. In: Daniel
RK, Ed. Rhinoplasty. Boston, MA: Little Brown; 1993:10-11.

A 19-year-old man has numbness of the left lower lip four weeks after undergoing
transoral placement of a Silastic chin implant. Physical examination shows
superior displacement of the left wing of the implant. Which of the following is
the most appropriate management?

(A) Injection of a corticosteroid


(B) Massage
(C) Observation
(D) Reoperation
(E) Taping

The correct response is Option D.

Reoperation should be done as soon as possible. The implant should be surgically


revised to remove pressure on the mental nerve caused by superior displacement
of the implant. In a patient with numbness, leaving an implant in place for eight
weeks or more may lead to permanent loss of sensation due to fascicular pressure
and may require nerve repair.

Observation without intervention could lead to permanent injury. Taping of the


chin is unlikely to be effective and may put additional pressure on the mental
nerve. Massage and corticosteroid injection are not appropriate because they
would not correct the underlying problem and would delay surgery, which could
lead to permanent injury.

References:
1. Zide BM, Pfeifer TM, Longaker MT. Chin surgery: I. AugmentationCthe
allures and the alerts. Plast Reconstr Surg. 1999;104(6):1843-1853.
2. Yaremchuk MJ. Improving aesthetic outcomes after alloplastic chin
augmentation. Plast Reconstr Surg. 2003;112(5):1422-1434.

A 25-year-old woman comes to the office for postoperative follow-up 10 days


after undergoing aesthetic rhinoplasty with rasping of a dorsal hump without the
performance of cartilage grafts or osteotomy. Physical examination shows dorsal
prominence with erythema but no fluctuance. Which of the following interventions
is the most appropriate initial management?

(A) Needle aspiration and irrigation


(B) Observation
(C) Open excision
(D) Oral administration of an antibiotic
(E) Topical administration of an antibiotic

The correct response is Option D.

Because this patient has periostitis of the nasal dorsum, the most appropriate initial
management is oral administration of an antibiotic to treat the infection. After the
erythema resolves, the dorsal prominence can be surgically excised in 8 to 12
months. Studies show that shavings retained after dorsal rasping or saw osteotomy
provide a nidus for periostitis. To reduce the risk of periostitis, all debris should be
evacuated from the dorsum at the conclusion of dorsal rasping or saw osteotomy.

The other interventions are not appropriate for this patient. Observation delays
treatment, which could lead to worsening infection. Topical administration of an
antibiotic is ineffective in treating periostitis. Needle aspiration and irrigation and
open excision are appropriate interventions to allow drainage and obtain cultures
in the presence of fluctuance, which this patient does not have.

References:
1. Sheen J, Peebles Sheen A. Aesthetic Rhinoplasty. 2nd ed. St. Louis, MO:
Mosby-Year Book; 1987:568-577.
2. Rees TD. Postoperative considerations and complications. In: Rees TD, ed.
Aesthetic Plastic Surgery. Philadelphia: WB Saunders; 1980:708-727.

Resection of the cephalic borders of the alar cartilages and caudal septum during
rhinoplasty is most likely to have which of the following effects?

(A) Decrease the alar flare


(B) Lengthen the nose
(C) Lower the columella
(D) Move the tip cephalad
(E) Shorten the nasal bones

The correct response is Option D.

Resection of the cephalic borders of the alar cartilages and caudal septum is
frequently done by directly accessing anatomic structures during open rhinoplasty
or by intracartilaginous, infracartilaginous, marginal, or transfixion incisions when
an intranasal approach is used. Cephalad resection of the lateral alar crus moves
the tip of the nose cephalad, decreases its fullness, and increases the definition of
the projecting points of the dome. During this surgery, care should be taken to
avoid weakening the support of the nostril arch by overresecting.
The other effects listed do not occur with resection of the cephalic borders of the
alar cartilages and caudal septum during rhinoplasty. Alar wedge (Weir) resection
is commonly used to decrease alar flare. Resection of the caudal septum usually
shortens the nose by allowing the tip of the nose to move cephalad with minimal
change in the nasolabial angle. This maneuver also raises the columella relative to
the alar margin and makes the upper lip appear longer. The nasal bones are not
affected by manipulation of the soft-tissue tip-lobule complex.

References:
1. Rohrich RJ, Muzaffar AR. Primary rhinoplasty. In: Achauer BM, Eriksson E,
Vander Kolk C, et al, eds. Plastic Surgery: Indications, Operations, and Outcomes.
Vol 5. St. Louis, MO: Mosby; 2000:2631-2672.
2. Guyuron B. Dynamic interplays during rhinoplasty. Clin Plast Surg.
1996;23:223-231.
3. Tebbets JB. Shaping and positioning of the nasal tip without surgical disruption:
a new, systematic approach. Plast Reconstr Surg. 1994;94(1):61-77.

A 21-year-old man comes to the office for consultation regarding rhinoplasty


because of a large dorsal hump. Effective surgical interventions to eliminate this
patient’s deformity include each of the following EXCEPT

(A) augmentation of a saddle-nose deformity


(B) augmentation of the radix with a dorsal implant
(C) rasping of the hump
(D) resection of the hump, followed by osteotomy and infracturing of the nasal
bones
(E) separation of the upper lateral cartilages from the nasal septum using a
transmucosal incision

The correct response is Option E.


The nasal dorsal hump is predominently cartilaginous (57%) rather than bony
(43%). Humps are classically resected by osteotomy. If an open-roof deformity is
created, infracture should be additionally performed. Some surgeons prefer rasping
the hump for fine control and shaping.

The cartilaginous portion of the hump consists of the nasal septum and the upper
lateral cartilages. These structures can be resected as a unit sharply. Separating the
upper lateral cartilages from the septum is not necessary and can compromise the
support of the nose.

Some humps are prominent due to lack of height at the radix or the supratip area.
For patients with a low caudal nasofrontal junction, a dorsal implant can give the
illusion of a reduced hump. Similarly, if a patient has a saddle nose, correction of
this defect will make a hump less conspicuous.

References:
1. Daniel RK, Lessard ML. Rhinoplasty: a graded aesthetic-anatomical approach.
Ann Plast Surg. 1984;13:436
2. Constantian MD. An alternate strategy for reducing the large nasal base. Plast
Reconstr Surg. 1989;83:41.
3. Peck GC. Basic primary rhinoplasty. Clin Plast Surg. 1988;15(1):15-27.

Each of the following is a general characteristic of the Asian nose EXCEPT

(A) alar flare


(B) bulbous nasal tip
(C) columellar show
(D) thick subcutaneous tissue
(E) wide flat dorsum

The correct response is Option C.

Columellar show is not a usual characteristic of the Asian nose. Although no two
noses are alike, common anatomic characteristics among Asian patients include
alar flare, a bulbous nasal tip, a short retracted columella, thick subcutaneous
tissue, and wide flat nasal dorsum. The base view of the nose commonly shows a
flat columella-alar triangle with hanging ala and a poorly projecting nasal tip. All
of these characteristics should be considered when evaluating an Asian patient for
rhinoplasty.

Asian rhinoplasty is one of the most commonly performed cosmetic procedures in


the world. Such surgeries usually focus on augmentation of the nasal dorsum with
an alloplastic implant. Different alloplastic materials, sizes, and shapes of implants
have been used with varying degrees of success. Although silicone implants for
nasal augmentation have been popular throughout Asia, such implants are not as
popular in the United States. An important consideration in implant surgery is the
thickness of the overlying nasal soft tissues to minimize implant extrusion.
References:
1. Matsunaga R. Augmentation rhinoplasty of Asian noses. Facial Plast Surg Clin
North Am. 1996;4:75-85.
2. Fernandez MO. Silicone implants for augmentation rhinoplasty. Facial Plast
Surg Clin North Am. 1996;4:55-62.
3. McCurdy JA. Augmentation rhinoplasty: implant selection and design. Facial
Plast Surg Clin North Am. 1996;4:87-92.

Cranio-Facial

Which of the following craniofacial anomalies is associated with orbital


hypotelorism?

(A) Crouzon syndrome


(B) Frontonasal dysplasia
(C) Frontonasal encephalocele
(D) Trigonocephaly

The correct response is Option D.

Hypotelorism, a decrease in intraorbital distance, occurs with trigonocephaly


caused by metopic synostosis and holoprosencephaly. In patients with metopic
synostosis, growth is inhibited perpendicular to the synostotic metopic suture,
resulting in decreased interorbital distance as well as narrowing of the forehead.
Holoprosencephaly results from a breakdown in the formation of prechordal
mesoderm, leading to deficiencies of the structures located in the midline and
narrowing of interorbital distance.

Hypertelorism, an increase in interorbital distance, is associated with numerous


craniofacial disorders, including frontonasal encephalocele and median facial
clefts occurring secondary to incomplete fusion of the medial orbital walls.
Patients with more severe forms of craniosynostosis, such as Apert or Crouzon
syndrome, may also have orbital hypertelorism. The hypertelorism seen in patients
with frontonasal dysplasia occurs as a result of excess bone formation.

References:
1. Kawamoto HK. Craniofacial clefts. In: Aston SJ, Beasley RW, Thorne CH, eds.
Grabb & Smith’s Plastic Surgery. 5th ed. Philadelphia: Lippincott Williams &
Wilkins;1997:349-363.
2. Ortiz-Monasterio F, Molina F. Orbital hypertelorism. Clin Plast Surg.
1994;21:599-612.
3. Tessier P. Anatomical classification of facial, craniofacial and latero-facial
clefts. J Maxillofac Surg. 1969;4:69.

In patients with Binder syndrome, the most likely physical finding is hypoplasia of
which of the following structures?

(A) Anterior cranial base


(B) Anterior nasal floor
(C) Anterior wall of the maxilla
(D) Medial orbital wall
(E) Nasal septum

The correct response is Option B.

A patient with Binder syndrome, or maxillonasal dysplasia, typically has a


shortened nose with flattening of the nasal bridge and perialar regions. The
columella is shortened, the nasolabial angle is acute, and the upper lip is convex.
The anterior nasal spine and frontonasal angle are absent. Occlusion is Angle class
III. Binder syndrome is caused by hypoplasia of the anterior nasal floor (fossa
praenasalis) and localized symmetric maxillary hypoplasia in the region of the alar
rims. Nasal views show a retracted columellar-lip junction, a perpendicular alar-
cheek junction, a convex upper nasal tip with a wide, shallow philtrum, crescent-
shaped nostrils without a sill, a low-set and flat nasal tip, and a stretched and
shallowed cupid’s bow. The triangular flair typically seen at the base of the nose is
instead absent.

In patients with Binder syndrome, the primary goal of surgery is increasing the
length of the nose and the projection of the nasal tip. This can be achieved by
performing Le Fort I osteotomy, Le Fort II osteotomy, or a combination of both
procedures, as well as compensatory orthodontic treatment. Autogenous bone and
cartilage grafts may be required to reconstruct the nose.

References:
1. Holmstrom H. Clinical and pathologic features of maxillonasal dysplasia
(Binder’s syndrome): significance of prenasal fossa on etiology. Plast Reconstr
Surg. 1986;78:559-567.
2. Posnick JC, Tompson B. Binder syndrome: staging of reconstruction and
skeletal stability and relapse patterns after Le Fort I osteotomy using miniplate
fixation. Plast Reconstr Surg. 1997;99:961-973.
Mutation of the fibroblast growth factor receptor (FGFR) has been most
commonly associated with which of the following single-suture synostoses?

(A) Lambdoid
(B) Metopic
(C) Sagittal
(D) Squamosal
(E) Unicoronal

The correct response is Option E.

Mutation of FGFR3, located at chromosome 4p16, has been found to cause


unicoronal synostosis. This suggests a genetic basis for certain forms of synostotic
frontal plagiocephaly. FGFRs regulate cell growth and bony proliferation.
Mutations in FGFRs have been associated with syndromic craniosynostoses, such
as in Pfeiffer, Crouzon, and Jackson-Weiss syndromes.

Evaluation of patients with unicoronal synostosis showed more severe cranial


dysmorphology and a higher number of surgical revisions in those with FGFR3
mutation for facial dysmorphology compared with those without the mutation.
This finding has led to genetic screening for all patients with unicoronal synostosis
to better counsel patients and anticipate surgical outcomes.

References:
1. Cassileth LB, Bartlett SP, Glat PM, et al. Clinical characteristics of patients
with unicoronal synostosis and mutations of fibroblast growth factor receptor 3: a
preliminary report. Plast Reconstr Surg. 2001;108(7):1849-1854.
2. Gripp KW, Stolle CA, McDonald-McGinn DM, et al. Phenotype of the
fibroblast growth factor receptor 2 Ser351Cys mutation: Pfeiffer syndrome type
III. Am J Med Genet 1998;78(4):356-360.
3. Gripp KW, McDonald-McGinn DM, Gaudenz K, et al. Identification of a
genetic cause for isolated unilateral coronal synostosis: a unique mutation in the
fibroblast growth factor receptor 3. J Pediatr. 1998;132(4):714-716.

Which of the following findings is commonly caused by nonsyndromic unicoronal


synostosis?

(A) Anterior displacement of the ipsilateral ear


(B) Deviation of the root of the nose to the contralateral side
(C) Flattening of the ipsilateral aspect of the occiput
(D) Occlusal cant up on the ipsilateral side
(E) Recession of the contralateral forehead

The correct response is Option A.

Nonsyndromic unicoronal synostosis commonly causes anterior displacement of


the ipsilateral ear toward the affected suture. It also typically results in ipsilateral
flattening and contralateral bossing of the forehead and deviation of the root of the
nose to the ipsilateral side toward the affected suture. Unicoronal synostosis does
not typically affect occlusion on either side and is not likely to cause significant
change in occipital shape.

References:
1. Bruneteau RJ, Mulliken JB. Frontal plagiocephaly: synostotic, compensational,
or deformational. Plast Reconstr Surg. 1991;89:21-31.
2. Biggs WS. Diagnosis and management of positional head deformity. Am Fam
Physician. 2003;67:1953-1958.

A 2-month-old infant has a facial cleft extending from the upper lip through the
nasal ala and into the medial canthal region. Which of the following is the most
appropriate Tessier classification of this cleft?

(A) No. 1
(B) No. 2
(C) No. 3
(D) No. 4
(E) No. 5

The correct response is Option C.

As shown below, the most appropriate classification for this cleft is Tessier
number 3 because this is the only classification in which the cleft involves the
nasal ala and medial canthus. A cleft classified as number 0 involves the midline
of the nose. A cleft classified as number 1 or 2 involves the nasal ala but is medial
to the eye. A cleft classified as number 4 or 5 is lateral to the nose and typically
involves the lower eyelid.
References:
1. Kawamoto HK. The kaleidoscopic world of rare craniofacial clefts: order out of
chaos (Tessier Classification). Clin Plast Surg. 1976;3:529-572.
2. Hunt JA, Hobar PC. Common craniofacial anomalies: facial clefts and
encephaloceles. Plast Reconstr Surg. 2003;112:606-616.

In patients with Treacher Collins syndrome, which of the following is a


characteristic skeletal finding?

(A) Brachycephaly
(B) Hypertelorism
(C) Macrogenia
(D) Malar hypoplasia
(E) Preaxial polysyndactyly

The correct response is Option D.


The characteristic skeletal finding in patients with Treacher Collins syndrome is
hypoplasia of the malar bones, which often occurs in conjunction with clefting
through the zygomatic arches. Patients also have hypoplasia of the maxilla and
mandible and antegonial notching of the angle of the mandible. Occlusion is Angle
class II; there is an anterior open bite and clockwise rotation of the occlusal plane.
Effects on the temporomandibular joint are varied.

Brachycephaly, macrogenia, preaxial polysyndactyly, and hypertelorism do not


occur in patients with Treacher Collins syndrome.

References:
1. Marsh JL, Celin SE, Vannier MW, et al. The skeletal anatomy of
mandibulofacial dysostosis (Treacher Collins syndrome). Plast Reconstr Surg.
1986;78:460.
2. Posnik JC. Treacher Collins syndrome. In: Aston SJ, Beasley RW, Thorne CH,
eds. Grabb & Smith’s Plastic Surgery. 5th ed. Philadelphia: Lippincott Williams &
Wilkins; 1997:313.

A 4-month-old infant has had a mass on the central nasal bridge (shown above)
since birth. Physical examination shows a mass measuring 0.5 _ 1 cm that is soft
and minimally mobile with a central pore. MRI of the head is shown above. Based
on these clinical findings, which of the following is the most likely diagnosis?

(A) Dermoid cyst


(B) Encephalocele
(C) Glioma
(D) Lipoma
(E) Vascular malformation

The correct response is Option A.

This infant has a nasoglabellar dermoid cyst with an intracranial component,


which requires transcranial and local excision.

In pediatric patients, orbitofacial dermoid cysts vary in presentation and location.


They typically fall into three subgroups: brow region, orbital region, and
nasoglabellar region. Most dermoid cysts are located in the lateral brow region.
Typically, dermoid cysts in the brow and orbital regions do not have intracranial
extension and can be excised locally. However, 10% to 45% of nasoglabellar
dermoid cysts have sinus tracts and occasional intracranial extension. For dermoid
cysts in this region, further studies with MRI or CT are recommended to exclude
intracranial extension.

Recent reports have shown frontotemporal dermoid cysts with sinus tracts that
have intracranial extension. If a sinus tract with extension is encountered during
local excision, further radiographic evaluation is warranted. Frontotemporal
dermoid cysts may represent a distinct entity from dermoid cysts in the brow
region.

References:
1. Bartlett SP, Lin KY, Grossman R, Katowitz J. The surgical management of
orbitofacial dermoids in the pediatric patient. Plast Reconstr Surg.
1993;91(7):1208-1215.
2. Lacey M, Gear AJ, Lee A. Temporal dermoids: three cases and a modified
treatment algorithm. Ann Plast Surg. 2003;51(1):103-109.

For each patient with coronal synostosis, select the most likely diagnosis (ABD).

(A) Apert syndrome


(B) Crouzon syndrome
(C) Pfeiffer syndrome
(D) Saethre-Chotzen syndrome

46 A 1-year-old infant has midface hypoplasia and axial skeletal deformities; the
digits are normal
47 A 10-year-old boy has generalized acne and symmetric complex syndactyly

The correct response for Item 46 is Option B and for Item 47 is Option A.

Each of the patients described above has acrocephalosyndactyly, or a syndrome of


craniofacial dysostosis. Patients with these syndromes have been shown to have
“gain in function” anomalies of the fibroblastic growth factor receptors.
The 1-year-old infant has Crouzon syndrome, or acrocephalosyndactyly type II.
This condition is characterized by hypoplasia of the midface, exorbitism, and axial
skeletal deformities. The fingers and toes are typically unaffected.

The 10-year-old boy has findings consistent with Apert syndrome, or


acrocephalosyndactyly type I, including bilateral complex syndactyly of the hands
and feet and generalized acne. Mental deficiencies are also common.

Characteristic findings in patients with Pfeiffer syndrome, also known as


acrocephalosyndactyly type V, include simple syndactyly and large, broad thumbs
and great toes.

Saethre-Chotzen syndrome is also known as acrocephalosyndactyly type III.


Affected patients typically have a low frontal hairline with backward sloping of
the forehead, low-set ears, beaking of the nose, and ptosis of the eyelids. Simple
syndactyly may also be seen.

References:
1. Bartlett SP, Mackay GJ. Craniosynostosis syndromes. In: Aston SJ, Beasley
RW, Thorne CH, eds. Grabb & Smith’s Plastic Surgery. Philadelphia: Lippincott
Williams & Wilkins; 1997:295-304.
2. Buchman SR, Muraszko KM. Syndromic craniosynostosis. In: Lin KY, Ogle
RC, Jane JA, eds. Craniofacial Surgery: Science and Surgical Technique.
Philadelphia: WB Saunders; 2002:252-271.

Ear Reconstruction - 2004


For each deformity of the ear, select the corresponding anatomic site in the
photograph shown on page 44 (A-E).

(A) Concha cavum


(B) Helical rim
(C) Scapha
(D) Superior crus of the antihelix
(E) Triangular fossa

1 Effacement of this structure most commonly results in prominence of the ear.

2 Hypertrophy of this structure most commonly results in prominence of the


middle third of the ear.

The correct response for Item 1 is Option D and for Item 2 is Option A.

In children with congenital ear prominence, the superior and middle thirds of the
ear are most likely to be affected. The most likely cause of a prominent superior
third of the ear is absence or effacement of the superior crus of the antihelix. As a
result, the conchoscaphal angle is greater than 90 degrees and the helix is
positioned more than 12 to 15 mm from the temporal region. The cephaloauricular
angle is also increased, typically measuring more than 25 degrees. Appropriate
management involves scoring and suturing of the cartilage to recreate the natural
roll of the antihelix.

In contrast, prominence of the middle third of the ear is most likely caused by
hypertrophy of the concha cavum. In affected patients, the concha cavum has a
depth of more than 1.5 cm. The middle third of the ear is located more than 16 to
18 mm from the mastoid region. Options for correction include excision and/or
reduction of the concha or setback with concha-mastoid sutures.
The scapha is the concave region between the helical rim and antihelix, and the
triangular fossa is the concave area between the superior and inferior crura of the
antihelix. Although the helical rim, scapha, and triangular fossa lie in the superior
third of the ear, they do not typically cause prominent ears.

References
1. Bauer BS, Patel PK. Congenital deformities of the ear. In: Bentz ML, ed.
Pediatric Plastic Surgery. Stamford, Conn: Appleton & Lange; 1998:359.
2. Spira M. Otoplasty: what I do now – a 30-year perspective. Plast Reconstr Surg.
1999;104:834-840.

A 77-year-old man has a 12-mm squamous cell carcinoma on the lateral margin of
the right helix. He is scheduled to undergo excision of the lesion with confirmation
of margins by frozen section, followed by immediate reconstruction. Which of the
following flaps is most appropriate for ear reconstruction?

(A) Antia-Buch flap


(B) Postauricular flap
(C) Temporoparietal fascial flap
(D) Temporalis muscle flap

The correct response is Option A.

The Antia-Buch flap is most appropriate for reconstruction of this patient’s ear
defect. The lesion can be excised easily because of its location on the lateral rim
and of the size of the auricle. Following excision, the resultant defect is effectively
reconstructed using the Antia-Buch flap, which is a local flap that uses tissue from
the helical rim based on the postauricular skin to reconstruct the helical margin. It
is a reliable, single-stage procedure that is acceptable aesthetically. The surgeon
may need to excise a “dog ear”-shaped area of tissue from the conchal bowl and
incise and advance the helical margins separately. However, because the two ears
are not viewed simultaneously, moderate differences in ear size are frequently
unnoticed.

A postauricular flap does not provide thin, contoured, helical-type tissue and
requires several procedures for adequate coverage. The temporoparietal fascial
flap provides thin, pliable soft-tissue coverage for a cartilage or alloplastic
framework, as in patients undergoing microtia reconstruction. A temporalis muscle
flap is excessively bulky and is not appropriate for ear reconstruction because it
would obliterate the intricate detailing of the ear.

References
1. Antia NH, Buch VI. Chondrocutaneous advancement flap for the marginal
defect of the ear. Plast Reconstr Surg. 1967;39:472.
2. Elsahy NI. Reconstruction of the ear after skin and cartilage loss. Clin Plast
Surg. 2002;29:201-212.
3. Low DW. Modified chondrocutaneous advancement flap for ear reconstruction.
Plast Reconstr Surg. 1998;102:174-177.
4. Park C, Lew DH, Yoo WM. An analysis of 123 temporoparietal fascial flaps:
anatomic and clinical considerations in total auricular reconstruction. Plast
Reconstr Surg. 1999;104:1295-1306.

A 14-year-old boy sustains an avulsion injury involving the entire pinna when he
is bitten by a dog. The amputated part has been preserved on iced saline gauze.
Following administration of antibiotics, tetanus toxoid, and rabies prophylaxis,
microsurgical replantation of the ear is to be performed. Anastomosis of the
arteries is most appropriate at which of the following anatomic locations on the
ear?

(A) Anterior surface


(B) Inferior surface
(C) Posterior surface
(D) Superior surface

The correct response is Option C.

In patients undergoing microsurgical replantation of the ear, the tissues are


debrided first, and dissection is performed to locate the appropriate vessels for
replantation, with visualization provided by an operating microscope. Because the
large arteries to the ear enter on the posterior aspect of the pinna, anastomosis is
most appropriate on the posterior surface. These arteries include branches of the
external carotid artery, the anterior auricular branch of the superficial temporal
artery, and a branch of the occipital artery. In contrast, the smaller branches are
located on the anterior surface.
References
1. Park C, Lineaweaver WC, Rumly TO, et al. Arterial supply of the anterior ear.
Plast Reconstr Surg. 1992;90:38-44
2. Pick TP, Howden R, eds. Gray’s Anatomy: Descriptive and Surgical. New
York, NY: Bounty Books; 1977.

A 15-year-old boy undergoes reconstruction of a 15-mm2 traumatic defect of the


right ear with a graft harvested from the contralateral ear. On examination two
days after the procedure, the graft appears dusky. A photograph is shown above.
Which of the following is the most appropriate next step in management?

(A) Hyperbaric oxygen therapy


(B) Application of leeches
(C) Release of the sutures
(D) Debridement of the graft

The correct response is Option A.

In this 15-year-old boy who exhibits duskiness at the graft site two days after
undergoing composite grafting of the ear, the most appropriate next step is
initiation of hyperbaric oxygen therapy. This will provide oxygenation during the
critical ischemia period for the graft and thus is likely to improve the outcome.
Hyperbaric oxygen therapy enhances antimicrobial activity by facilitating the
oxidative burst of polymorphonuclear neutrophils. It increases the
hyperoxygenation of tissue to a level that is 10 to 15 times greater than normal. In
addition, it stimulates angiogenesis and blunts the ischemia-reperfusion injury
response.

Application of leeches is appropriate if arterial input is adequate but venous


outflow is insufficient, as in patients undergoing microsurgical replantation who
demonstrate thrombosis of the vein, or if a suitable vein does not exist for
anastomosis. However, duskiness of the ear is an indication of arterial
insufficiency, and leeches would fail to attach if they were applied.

Because a composite graft receives its vascularity through diffusion from the
surrounding wound bed, releasing the sutures would inhibit the “take” of the graft
to the bed. Similarly, performing debridement two days after grafting is excessive.
Instead, the composite graft should be left in place for a minimum of two weeks in
order to demonstrate healing and incorporation, as long as infection does not
develop.

References
1. McClane S, Renner G, Bell PL, et al. Pilot study to evaluate the efficacy of
hyperbaric oxygen therapy in improving the survival of reattached auricular
composite grafts in the New Zealand White rabbit. Otolaryngol Head Neck Surg.
2000;123:539-542.
2. Nichter LS, Morwood DT, Williams GS, et al. Expanding the limits of
composite grafting: a case report of successful nose replantation assisted by
hyperbaric oxygen therapy. Plast Reconstr Surg. 1991;87:337-340.
3. Renner G, McClane SD, Early E, et al. Enhancement of auricular composite
graft survival with hyperbaric oxygen therapy. Arch Facial Plast Surg.
2002;4:102-104.
4. Zhang F, Cheng C, Gerlach T, et al. Effect of hyperbaric oxygen on survival of
the composite ear graft in rats. Ann Plast Surg. 1998;41:530-534.

A 24-year-old man has pain and swelling of the left ear after injuring the ear in a
fight. Physical examination shows obliteration of the normal contours of the lateral
surface of the ear. Which of the following is the most appropriate management?

(A) Application of a pressure dressing for several days, followed by evacuation of


clotted blood
(B) Needle aspiration of the ear
(C) Needle aspiration of the ear and application of a pressure dressing
(D) Incision and drainage of the skin and perichondrium and application of a
pressure dressing
(E) Excision of thickened tissue and placement of suction-drainage catheters

The correct response is Option D.

Hematoma formation is the primary complication of blunt trauma to the ear. The
mechanism of injury involves disruption of blood vessels in the perichondrium,
leading to hemorrhage. The blood fills the space between the perichondrium and
cartilage, distorting the contour of the lateral ear into a convex shape and blocking
the vascular supply to the cartilage, which is derived from the perichondrium.
Necrosis or infection of the cartilage results.

Prompt treatment involves removing the accumulated blood while maintaining


pressure on the affected area for several days to prevent recurrence. To accomplish
this, an incision is made through the skin and perichondrium on the inner side of
and parallel to the antihelix, which will conceal the scar. The blood is drained and
the wound is inspected for further bleeding. When the surgeon is assured that the
bleeding has stopped, a pressure dressing and a head dressing are applied.

Late treatment of a cauliflower ear deformity involves excision of the thickened


tissue, including fibrous tissue and new cartilage, followed by application of a
pressure dressing.

Simple needle aspiration of the blood is likely to result in development of seroma.

References
1. Elsahy NI. Acquired ear defects. Clin Plast Surg. 2002;29:175-186.
2. Schuller DE, Dankle SD, Strauss RH, et al. A technique to treat wrestlers’
auricular hematoma without interrupting training or competition. Arch
Otolaryngol Head Neck Surg. 1989;15:202-206.

A 21-year-old man sustains a complete amputation of the right ear at the level of
the external auditory canal in a motor vehicle collision. There are no other injuries.
Which of the following procedures will provide the best aesthetic result?

(A) Delayed total ear reconstruction with a rib cartilage graft


(B) Dermabrasion of the epidermis of the amputated ear, burial of the ear in a
subcutaneous postauricular pocket, followed by removal and coverage with a skin
graft or flap
(C) Removal of the skin of the amputated ear, reattachment of the ear cartilage,
and immediate coverage with a temporoparietal fascial flap and skin graft
(D) Composite grafting of the amputated ear followed by surface cooling
(E) Microsurgical ear replantation

The correct response is Option E.

Successful microsurgical replantation of the ear provides superior aesthetic results


while eliminating the need for other complex reconstructive procedures. However,
this technique is associated with increased operative time and the need for multiple
blood transfusions. Hospitalization is typically prolonged, and failure rates
associated with the procedure are high.
Delayed reconstruction results in only moderate cosmetic improvement, and
secondary reconstruction does not sufficiently recreate the intricate architecture of
the external ear.

Primary nonvascularized replantation of the ear produces a good appearance


initially because of the survival of the avulsed cartilage; however, late distortion of
the cartilage frequently limits the overall aesthetic result. Techniques used for
nonvascularized replantation include primary reattachment of the ear with surface
cooling, dermabrasion of the ear, and partial or complete burial of the ear in a
postauricular skin pocket, followed by coverage of the filleted cartilage with a
temporoparietal fascial flap and skin graft.

References
1. Brent B. Reconstruction of the auricle. In: McCarthy JG, ed. Plastic Surgery.
Philadelphia, Pa: WB Saunders Co; 1990;3:2094.
2. King GM. Microvascular ear transplantation. Clin Plast Surg. 2002;29:233-248.
3. Turpin IM. Microsurgical replantation of the external ear. Clin Plast Surg.
1990;17:397.

A 6-year-old boy has prominent ears. Physical examination shows an obtuse


concha-mastoid angle. The antihelical fold is normal. Which of the following is
the most appropriate management?

(A) Use of a headband splint at night


(B) Use of Mustardé sutures
(C) Setback of the concha using concha-mastoid sutures
(D) Excision of excess skin
(E) Resection of the concha

The correct response is Option C.

Because prominent ears can be caused by an enlarged conchal bowl, an obtuse


concha-mastoid angle, or loss of the antihelical fold, appropriate management
should be based on the cause of the deformity. This 6-year-old boy has ear
prominence caused by an obtuse concha-mastoid angle. Conchal setback is
recommended in children to correct the enlarged conchal bowl and obtuse concha-
mastoid angle deformities. This is accomplished using concha-mastoid sutures,
which are mattress sutures placed between the posterior conchal wall and the
mastoid periosteum to create a more acute concha-mastoid angle and decrease the
height of the protruding concha.
In contrast, elliptical conchal excision may be required to perform conchal setback
in adults, whose ear cartilage is typically stiff.

Ear prominence resulting from loss of the antihelical fold is best corrected by
abrading or scoring the antihelix and placing Mustardé mattress sutures between
the conchal eminence and the scaphoid eminence.

Splinting is effective only in infants because of the pliability of the ear cartilage in
this age group.

Excess skin may need to be excised following placement of concha-mastoid


sutures, but this is unlikely to produce adequate setback if performed alone. This
technique is appropriate instead to decrease the prominence of the lobule or
superior helix.

References
1. Furnas DW. Otoplasty for prominent ears. Clin Plast Surg. 2002;29:273-288.
2. Yotsuyanagi T, Yokoi K, Sawada Y. Nonsurgical treatment of various auricular
deformities. Clin Plast Surg. 2002;29:327-332.

Each of the following deformities is commonly associated with microtia EXCEPT

(A) cervical spine abnormalities


(B) inner ear abnormalities
(C) macrostomia
(D) mandibular hypoplasia
(E) preauricular pits

The correct response is Option B.

Patients with microtia have partial or complete absence of the external ear
structures due to abnormal embryologic development of portions of the first, or
mandibular, and second, or hyoid, branchial arches. This typically occurs during
the fourth to twelfth week of intrauterine development and affects the auditory
ossicles, external auditory canal, middle ear cavity, and tympanic membrane.
Several abnormalities can occur in conjunction with microtia. Orbital auricular
vertebral syndrome, also known as Goldenhar syndrome, and the Tessier No. 7
cleft also result from abnormalities in the development of the first and second
branchial arches. Orbital auricular vertebral syndrome is characterized by microtia,
cervical spine abnormalities, mandibular hypoplasia, preauricular pits and sinuses,
and hemifacial microsomia. The Tessier No. 7 cleft manifests as microtia,
macrostomia, and preauricular sinuses.
Because the external auditory meatus and internal ear are derived from different
structures, the internal ear is usually well constructed in patients with microtia.
Likewise, patients with orbital auricular vertebral syndrome have abnormalities of
the middle and external ear but not the inner ear.

References
1. Kawamoto HK Jr, Patel PK. Atypical facial clefts. In: Bentz ML, ed. Pediatric
Plastic Surgery. Stamford, Conn: Appleton & Lange; 1998:175-225.
2. Kurihara K. Congenital deformities of the external ear. In: Cohen M, ed.
Mastery of Plastic and Reconstructive Surgery. Boston, Mass: Little, Brown & Co;
1994;1:776-779.

Eye Reconstruction

A 2-year-old boy is brought to the office by his parents for evaluation of ptosis of
the upper eyelids. On examination, there are no abnormalities of the right upper
eyelid. Examination of the left eye shows 3.5 mm of ptosis of the upper lid,
absence of the eyelid crease, and 3 mm of levator muscle function. Which of the
following is the most appropriate management of the left eye?

(A) Observation
(B) Application of a patch
(C) Frontalis sling, upper eyelid
(D) Levator resection/advancement, upper eyelid
(E) Resection of Müller’s muscle, upper eyelid

The correct response is Option C.

The patient has 3.5 mm of ptosis of the left upper eyelid, which partially obstructs
vision. Absence of the eyelid crease and 3 mm of levator function are highly
suggestive of congenital ptosis. A frontalis sling of the left upper eyelid can
correct these abnormalities by resetting the position of the affected eyelid.

Observation is not appropriate because the partial obstruction of vision could lead
to amblyopia in the left eye. Application of a patch is inappropriate because it
would deprive the left eye of all visual input, exacerbating the problem. Resection
of Müller’s muscle from the left upper eyelid is an acceptable treatment of a small
amount of ptosis (such as 1 mm) in an adult but would not correct this severe
ptosis. Levator resection and advancement of the upper eyelid are the most
common method of treating levator dehiscence in adults but would not be
appropriate to treat a child with minimal levator function.

References:
1. Caraway JA. Reconstruction of the eyelid and correction of ptosis of the eyelid.
In: Aston SJ, Beasley RW, Thorne CH, eds. Grabb & Smith’s Plastic Surgery. 5th
ed. Philadelphia: Lippincott Williams & Wilkins;1997:529-544.
2. Ramirez OM, Pena G. Frontalis muscle advancement: a dynamic structure for
the treatment of severe congenital eyelid ptosis. Plast Reconstr Surg.
2004;113(6):1841-1851.

A 57-year-old woman comes to the office for evaluation of drooping of the


eyelids. Physical examination shows ptosis of the upper eyelids, high eyelid
crease, margin to reflex distance (MRD) of 0 mm, and excellent levator function.
Which of the following techniques is most appropriate for surgical correction of
this patient=s abnormality?

(A) Fasanella-Servat
(B) Plication of the levator muscle
(C) Reanastomosis of the dehisced levator aponeurosis
(D) Resection of the orbicularis muscle
(E) Unilateral frontalis suspension using autogenous fascia lata

The correct response is Option C.

This patient has classic signs of blepharoptosis secondary to acquired dehiscence


of the levator aponeurosis, such as high eyelid crease, excellent levator function,
and an MRD of 0 mm. The MRD quantifies the distance between the lid margin
and the pupillary reflex. To correct this blepharoptosis in the most anatomically
correct fashion, surgery should be done to reanastomose the dehisced end of the
levator aponeurosis to the superior aspect of the tarsus.

The other surgical options are used for different indications. A Fasanella-Servat
procedure takes a posterior conjunctival approach to correct mild ptosis without
levator disinsertion. Plication of the levator muscle alone does not provide long-
lasting results and is a valid option only if the distal end of the muscle is not
dehisced. Resection of the orbicularis muscle will not correct the ptosis. Unilateral
frontalis suspension is the procedure of choice for patients with absent levator
function and severe ptosis.

References:
1. Caraway JA. Reconstruction of the eyelid and correction of ptosis of the eyelid.
In: Aston SJ, Beasley RW, Thorne CH, eds. Grabb & Smith’s Plastic Surgery. 5th
ed. Philadelphia: Lippincott Williams & Wilkins; 1997:529-544.
2. Gonnering RS. Upper eyelid blepharoplasty. In: Tse DT, Wright KW, eds.
Color Atlas of Ophthalmic Surgery: Oculoplastic Surgery. Philadelphia: JB
Lippincott; 1992:175-188.
A 6-year-old child who has had chronic bilateral epiphora since birth has been
treated with corneal lubrication for the past year. Slit-lamp examination by the
child’s pediatric ophthalmologist one week ago showed bilateral corneal staining.
On physical examination, the lashes on both lower eyelids rub against the inferior
cornea (shown above). Which of the following is the most appropriate
management?

(A) Daytime taping of the margin of the lower lid


(B) Initiate nighttime lubrication of the eyes
(C) Lateral tarsal strip with repositioning of the eyelid margin
(D) Reinsertion of the retractors at the base of the tarsus and subtotal excision of
the preseptal orbicularis
(E) Resection of redundant pretarsal skin and orbicularis muscle

The correct response is Option E.

The most common cause of epiblepharon is excess pretarsal skin and orbicularis
oculi muscle at the lower eyelid margin. In this congenital anomaly, a fold of skin
and underlying orbicularis muscle override the eyelid margin, often pushing the
cilia against the globe. The eyelid margin and tarsus are stable and maintain the
proper orientation. Epiblepharon usually affects the lower eyelids, is more
common among Asians, and may be accentuated on downward gaze. Most cases
resolve with facial growth during childhood. Surgical correction is needed when
the lashes cause significant corneal injury. Epiblepharon requires resection of the
redundant pretarsal skin and orbicularis muscle as well as placement of sutures
between the tarsal plate and the subcutaneous tissue to create adhesions.
Taping of the lower eyelid is not practical in a child. Nighttime lubrication of the
eyes is not sufficient treatment because the lashes continue to cause damage
during the day, which may cause permanent corneal scarring. Lateral tarsal strip
and repositioning of the eyelid margin are a common treatment for ectropion.
Reinsertion of the retractors at the base of the tarsus and subtotal excision of the
preseptal orbicularis are the treatment of choice for involutional entropion with
horizontal laxity from the tarsus, vertical laxity due to attenuation or disinsertion
of the lower eyelid retractors or orbital septum, and migration of the preseptal
orbicularis in the pretarsal position.

References:
1. Woo KI, Yi K, Kim YD. Surgical correction for lower lid epiblepharon in
Asians. Br J Ophthalmol. 2000;84(12):1407-1410.
2. Jeon S, Park H, Park YG. Surgical correction of congenital epiblepharon: low
eyelid crease reforming technique. J Pediatr Ophthalmol Strabismus.
2001;38(6):356-358.

A 44-year-old woman has the defect shown above three days after undergoing
excision of a squamous cell carcinoma from the lower eyelid. The resultant defect
of the lower lid is 60%. Which of the following reconstructions is the most
appropriate management?
(A) Composite contralateral lower lid graft
(B) Cutler-Beard flap
(C) Direct closure with cantholysis
(D) Hughes tarsoconjunctival flap
(E) Skin graft

The correct response is Option D.

This defect requires prompt surgical correction, because allowing the wound to
granulate would result in a severely dysfunctional eyelid. With such a defect, a
Hughes tarsoconjunctival flap is best used for reconstruction. This reconstruction
involves taking a tarsoconjunctival flap from the upper eyelid, leaving 3 to 4 mm
of the upper tarsus intact, and advancing the remaining tarsus and conjunctiva to
the lower eyelid defect. This provides the posterior lamella of the eyelid. The
anterior lamella is reconstructed by advancement of a skin-muscle flap from the
lower eyelid or cheek.

Direct closure with cantholysis is appropriate for defects of less than 25% of the
eyelid. The Cutler-Beard flap, which uses skin and muscle from the lower eyelid,
is inappropriate for a patient with a lower eyelid defect. Because this patient=s
defect affects more than 50% of the lower eyelid and because the eyelid requires a
supportive base, a skin graft or composite contralateral lower eyelid graft would
not provide adequate support.
References:
1. Caraway JA. Reconstruction of the eyelid and correction of ptosis of the eyelid.
In: Aston SJ, Beasley RW, Thorne CH, eds. Grabb & Smith’s Plastic Surgery. 5th
ed. Philadelphia: Lippincott Williams & Wilkins; 1997:529-544.
2. Rohrich RJ, Zbar RIS. The evolution of the Hughes tarsoconjunctival flap for
lower eyelid reconstruction. Plast Reconstr Surg. 1999;104:518-523.
.
Flaps

Which of the following muscles is included in a facial artery musculomucosal


flap?

(A) Buccinator
(B) Depressor anguli oris
(C) Levator labii superioris
(D) Orbicularis oris
(E) Zygomaticus major

The correct response is Option A.


Because the buccinator muscle is sandwiched between the facial artery and the
oral mucosa, it must be included in a facial artery musculomucosal flap. The
buccinator muscle originates from the pterygomandibular raphe and inserts into
the orbicularis oris muscle and mucosa of the lateral lip elements.
The other muscles listed are supplied by the facial nerve but are not included in a
facial artery musculomucosal flap because of their location. The depressor anguli
oris and zygomaticus major muscles lie superficial to the plane of dissection for
this flap. The levator labii superioris muscle inserts into the lateral half of the lip,
but its origin is from the infraorbital margin of the maxilla. The orbicularis oris
muscle is anterior to the plane of dissection for the facial artery musculomucosal
flap.

References:
1. Pribaz J, Stephens W, Crespo L, Gifford G. A new intraoral flap: facial artery
musculomucosal (FAMM) flap. Plast Reconstr Surg. 1992;90:421-429.
2. Wexler A. Anatomy of the head and neck. In: Ferraro JW, ed. Fundamentals of
Maxillofacial Surgery. New York: Springer-Verlag; 1997:53-114.

A 65-year-old man undergoes operative removal of a basal cell carcinoma at the


junction of the upper cheek and temporal region, followed by coverage of the
resultant 10 _ 5-cm defect with a submental myocutaneous flap. This flap derives
its blood supply from a branch of which of the following arteries?

(A) Facial
(B) Inferior thyroid
(C) Lingual
(D) Superior thyroid
(E) Transverse cervical

The correct response is Option A.

The submental flap is a myocutaneous flap that is useful in head and neck
reconstruction. This flap provides a contour, color, and tissue texture that is
suitably matched to the face. The flap is elevated below the level of the platysma
muscle and includes the submental artery and vein, which are direct branches of
the facial artery and vein. The flap can be transposed to cover defects in the lower
and central thirds of the face and into the inferior aspect of the upper third of the
face.
The submental artery is a consistent branch of the facial artery and gives off one or
two cutaneous perforators to the submental skin. The submental artery runs in
relation to the anterior belly of the diagastric muscle. Of the choices listed, the
facial artery is the most superior branch of the external carotid artery. The lingual
artery provides the blood supply to the tongue, the superior thyroid and inferior
thyroid arteries provide the blood supply to the thyroid gland, and the transverse
cervical artery gives off a descending branch, which provides the blood supply to
the trapezius muscle.

References:
1. Faltaous AA, Yetman RJ. The submental artery flap: an anatomic study. Plast
Reconstr Surg. 1996;97:56-60.
2. Pistre V, Pelissier P, Martin D, et al. Ten years of experience with the
submental flap. Plast Reconstr Surg. 2001;108:1576-1581.

Which of the following arteries is the basis of the major blood supply to the
pectoralis major myocutaneous flap for head and neck reconstruction?

(A) Internal mammary


(B) Lateral thoracic
(C) Superior thoracic
(D) Thoracoacromial
(E) Transverse cervical

The correct response is Option D.

The major blood supply to the pectoralis major myocutaneous flap is the
thoracoacromial artery. This flap has been the workhorse for head and neck
reconstruction. It remains a lifeboat flap for microvascular flap failures in the area.
It arises from the second part of the axillary artery (continuation of the subclavian
artery) and divides into four branches with the pectoral branch supplying the
pectoral muscles and the flap. The superior thoracic artery arises from the first part
of the axillary artery and also supplies some part of the pectoral muscles. The
lateral thoracic artery also arises from the axillary artery but does not supply the
flap. The internal mammary artery does supply the pectoralis major muscle and its
accompanying skin; however, it cannot be pedicled on this axis for head and neck
reconstruction. The transverse cervical artery originates from the subclavian artery
and supplies the muscles of the neck and scapula.

References:
1. Gabella G. Cardiovascular. In: Bannister LH, Berry MM, Collins P, et al, eds.
Gray’s Anatomy. 38th ed. New York: Churchill Livingstone, 1995:1504-1574.
2. Ariyan S. Pectoralis major muscle and musculocutaneous flaps. In: Strauch B,
Vasconez LO, Hall-Findlay EJ, eds. Grabb’s Encyclopedia of Flaps. Vol 1.
Philadelphia: Lippincott Williams & Wilkins; 1998:470-473.

Which of the following interventions is most appropriate to improve the viability


of an ischemic skin flap?

(A) Apply medicinal leeches to the flap


(B) Apply nitroglycerin paste to the flap
(C) Elevate the affected area
(D) Ensure adequate fluid resuscitation
(E) Perform hyperbaric oxygen therapy

The correct response is Option D.

The initial 24-hour period is critical to flap viability. In cases of compromised


arterial inflow, steps that can be taken to improve arterial inflow are of primary
concern. Ensuring adequate postoperative fluid resuscitation is paramount. This
ensures adequate cardiac output and optimizes tissue perfusion.

Medicinal leeches are indicated for venous congestion and would have
questionable value in an ischemic setting. Although hyperbaric oxygen therapy
and elevation may both have beneficial effects in this setting, they are less
appropriate than fluid resuscitation. Some clinicians will use 2% nitroglycerin
ointment to ischemic areas every four to six hours or silver sulfadiazine cream
twice daily. Either of these techniques is believed to be useful in reducing the risk
and extent of full-thickness skin loss but would probably not be as important as
ensuring adequate hydration.

References:
1. Lockwood T. Body contouring with excisions. In: Goldwyn RM, Cohen MN,
eds. The Unfavorable Result in Plastic Surgery: Avoidance and Treatment. 3rd ed.
Philadelphia: Lippincott Williams & Wilkins; 2001:1148-1149.
2. Zamboni WA, Roth AC, Russel RC, et al: The effect of hyperbaric oxygen on
reperfusion of ischemic axial skin flaps: a laser Doppler analysis. Ann Plast Surg.
1992;28:339.

Hand Congen - 2004


A 6-month-old boy has the deformities shown in the photograph above. There are
no associated syndromes. His parents would like to have more children and inquire
about the risk for similar deformities. The risk for development of this deformity
in future offspring is closest to

(A) 0%
(B) 5%
(C) 10%
(D) 25%
(E) 50%

The correct response is Option A.

This 6-month-old boy has amniotic constriction band syndrome, a congenital hand
deformity that manifests as intrauterine amputation of the digits. Amniotic
constriction band syndrome has no known genetic transmission and is considered
to be an intrauterine accident. Although there are several theories regarding the
cause of this condition, it is believed to be related to rupture of the amniotic
membrane caused by oligohydramnios; when this occurs, the digits and/or
extremities are constricted by amniotic tissue bands created from the edges of the
ruptured amniotic sac. Amniotic bands may result in amputations, not only of the
fingers (as in this patient) or the extremities, but sometimes other parts of the
body. The degree of constriction is variable and is classified according to its
severity.

Amniotic constriction band syndrome is the fourth most common congenital


anomaly of the hand; only polydactyly, syndactyly, and trigger thumb occur more
frequently. Currently, its incidence is estimated at approximately 1 in 5000 to
15,000 births, with variations among different ethnic groups.

Anomalies frequently associated with amniotic constriction band syndrome


include club foot, cleft lip and/or palate, craniofacial defects, hemangioma, and
meningocele.
In this infant, appropriate management of the hand deformity is deepening of the
web space using a standard dorsal flap followed by full-thickness skin grafting.

References
1. Doyle JR. Constriction ring reconstruction. In: Blair WF, ed. Techniques in
Hand Surgery. Baltimore, Md: Williams & Wilkins; 1996:1106-1111.
2. Ezaki M. Amnion disruption sequence. In: Green DP, Hotchkiss RN, Pederson
WC, eds. Operative Hand Surgery. 4th ed. New York, NY: Churchill Livingstone,
Inc; 1999;1:429-431.
3. Upton J. Congenital anomalies of the hand and forearm. In: McCarthy JG, ed.
Plastic Surgery. Philadelphia, Pa: WB Saunders Co; 1990;8:5373-5378.
4. Wiedrich TA. Congenital constriction band syndrome. Hand Clin. 1998;14:29-
37.

During embryologic development of the hand, which of the following structures is


most likely to affect normal differentiation of the limb?

(A) Apical ectodermal ridge


(B) Fourth branchial cyst
(C) Germinal matrix
(D) Second branchial cleft
(E) Wolffian ridge

The correct response is Option A.

The apical ectodermal ridge is the critical structure that defines the growth and
differentiation of the new limb during embryologic development; if the apical
ectodermal ridge were to be surgically removed, the developing limb would be
truncated. The apical ectodermal ridge arises from the Wolffian ridge, which
protrudes from the main trunk of the embryo. A zone of polarizing activity
determines the anterior-posterior morphology of the limb.

The upper extremities begin to develop during the first four weeks of gestation. By
the completion of the fifth week, the hand becomes a recognizable structure. As
the apical ectodermal ridge becomes flattened, the hand appears initially as a
paddle. Each of the phalanges is then formed by a process of physiologic cell
death that occurs within the web spaces. The digits become defined by the end of
the eighth week, and fingernails can be identified by 17 weeks’ gestation.

References
1. Daluiski A, Yi SE, Lyons KM. The molecular control of upper extremity
development: implications for congenital hand anomalies. J Hand Surg.
2001;26A:8-22.
2. McCarroll HR. Congenital anomalies: a 25-year overview. J Hand Surg.
2000;25A:1007-1037.
3. Watson S. The principles of management of congenital anomalies of the upper
limb. Arch Dis Child. 2000;83:10-17.

A 6-year-old girl has camptodactyly of the small finger of the right hand. The
deformity has not improved with splinting and passive stretching exercises. On
physical examination, there is loss of 20 degrees of terminal extension of the
proximal interphalangeal (PIP) joint that is unaffected by flexion of the
metacarpophalangeal joint.

Which of the following is the most appropriate management?

(A) Observation
(B) Injection of a corticosteroid into the PIP joint
(C) Release of the lumbrical tendon
(D) Release of the superficialis tendon
(E) Zancolli-lasso procedure

The correct response is Option A.

In this 6-year-old girl with camptodactyly who has a loss of 20 degrees of


extension of the proximal interphalangeal (PIP) joint, observation is most
appropriate. Surgical correction is rarely required in patients who have lost less
than 30 degrees of extension. Minimal extension deficits that require operative
management can be corrected by transferring the lumbrical tendon into the central
slip.

In patients who have a terminal extension deficit of 30 degrees or more, surgical


release of the abnormal lumbrical and superficialis tendons is most likely to
improve the deformity. Release of the attachment of the accessory collateral
ligaments to the volar plate may also be necessary. Full-thickness skin grafts or
local skin flaps can be used to cover the resultant volar skin deficits. However, the
patient should be informed of the likelihood for a residual flexion deformity,
especially if there are abnormalities of the articular joint surfaces on preoperative
radiographs.

Corticosteroids should not be injected into the affected joints of patients with
camptodactyly. In the Zancolli-lasso procedure, slips of the flexor digitorum
superficialis tendon are looped through the A2 pulley. This technique is
appropriate for correction of digital clawing associated with ulnar palsy in a
patient who demonstrates improved extension of the PIP joint with flexion of the
metacarpophalangeal joint.

References
1. Ezaki M, Kay SP, Light TR, et al. Congenital hand deformities. In: Green DP,
Hotchkiss RN, Pederson WC, eds. Operative Hand Surgery. 4th ed. New York,
NY: Churchill Livingstone, Inc; 1999;1:325-551.
2. McFarlane RM, Curry GI, Porte AM, et al. The anatomy and treatment of
camptodactyly of the small finger. J Hand Surg. 1992;17A:35-44.

Q174
PIC

A 10-month-old infant with simple, complete syndactyly involving the third web
space of the left hand undergoes reconstruction using a dorsal flap and multiple
full-thickness skin grafts. A postoperative photograph is shown above. Which of
the following is the most appropriate next step in management?

(A) Wound cleansing using dilute peroxide and application of antibiotic ointment
daily
(B) Application of soft dressings and an elastic wrap
(C) Application of a short arm splint
(D) Application of a short arm cast
(E) Application of a long arm cast

PIC

The correct response is Option E.

In infants and children who undergo reconstructive procedures for correction of


syndactyly, the most appropriate management postoperatively is application of a
long arm cast. The success of the procedure depends on adequate immobilization
of the entire upper extremity following surgery. When applying the cast, the elbow
must be flexed before padding and fiberglass material are applied in order to
prevent restriction or pressure in the antecubital region. Flexion of the cast at the
elbow will prevent the dressings around the skin grafts and flaps from becoming
loosened.

The photograph above shows the properly applied above-elbow cast of a patient
who has undergone syndactyly reconstruction. All of the fingertips are visible, but
the fingers are immobilized securely. This infant previously underwent two
reconstructive attempts that were unsuccessful because of improper postoperative
immobilization. The third procedure, involving release of severe contractures of
the digits followed by repeat full-thickness skin grafting, was successful because
the arm was immobilized correctly.

If the arm is not immobilized, the skin grafts and flaps will ultimately fail, and
significant wound problems will occur. Soft dressings do not provide the needed
immobilization. Short arm splints and casts frequently become dislodged in infants
in children because of their high level of activity and the problems associated with
fitting a short arm device to the cone-shaped forearm of a young child.

References
1. Ezaki M. Syndactyly. In: Green DP, Hotchkiss RN, Pederson WC, eds.
Operative Hand Surgery. 4th ed. New York, NY: Churchill Livingstone, Inc;
1999;1:426.
2. Upton J. Congenital anomalies of the hand and forearm. In: McCarthy JG, ed.
Plastic Surgery. Philadelphia, Pa: WB Saunders Co; 1990;8:5292.

A 3-year-old boy has total absence of the long and ring fingers of both hands. This
finding is most consistent with which of the following conditions?

(A) Camptodactyly
(B) Clinodactyly
(C) Ectrodactyly
(D) Macrodactyly
(E) Polysyndactyly

The correct response is Option C.

This 3-year-old boy has ectrodactyly, a partial or total absence of the fingers that
suggests a central hand deficiency. Central hand deficiencies can be classified as
typical or atypical. Patients with typical central hand deficiencies have absence of
the third ray (infrequently the second ray, rarely the fourth ray). The finding is
often bilateral. Other anomalies, including cleft lip and palate, congenital heart
disease, and significant deformities of the upper and lower extremities, can be
associated. Syndactyly and foot involvement are common. The inheritance is
familial.

In contrast, a patient with atypical central hand deficiency usually has several
deficient rays unilaterally. This deficiency is commonly known as
symbrachydactyly and is the opposite of true cleft hand. Inheritance is nonfamilial.
Other anomalies, such as syndactyly and foot involvement, are uncommon.
Affected patients typically have digital nubbins that are best removed at birth.
Functional hand grasp can be increased through surgical manipulation of the web
space or osteotomies.
Camptodactyly is a nontraumatic flexion deformity of the proximal
interphalangeal joint, usually bilateral and involving the small finger.

Patients with clinodactyly have either radial or ulnar angulation of the digit,
usually the small finger, at the distal interphalangeal joint. In severe clinodactyly,
a delta phalanx is common.

Macrodactyly involves overgrowth of one digit or hand segment. Overgrowth


appears to be under neurotrophic control. Correction is either reduction or
ablation.

Polysyndactyly is a type of complicated syndactyly, in which polydactyly and


syndactyly are seen in association.

References
1. Dobyns JH, Wood VE, Bayne LG. Congenital hand deformities. In: Green DP,
ed. Operative Hand Surgery. New York, NY: Churchill Livingstone, Inc;
1993;1:251-548.
2. Flatt AE. The Care of Congenital Hand Anomalies. Saint Louis, Mo: CV Mosby
Co; 1977.
3. Upton J. Congenital anomalies of the hand and forearm. In: McCarthy JG, ed.
Plastic Surgery. Philadelphia, Pa: WB Saunders Co; 1990;8:5213-5398.

Q190
PIC

A 9-month-old infant has the deformity shown in the photograph above. The
pregnancy and delivery were uncomplicated; the infant’s development is otherwise
normal. Which of the following is the most likely diagnosis?
(A) Brachysyndactyly
(B) Camptodactyly
(C) Clinodactyly
(D) Ectrodactyly
(E) Syndactyly

The correct response is Option E.

Syndactyly is a common congenital hand anomaly, occurring in one of every 2500


neonates. Familial inheritance ranges from 10% to 40%, and boys are twice as
likely as girls to be affected. Syndactyly can be simple or complex and complete
or incomplete. Patients with simple syndactyly have fusion of the skin of adjacent
digits. In complex syndactyly, the skin and bones are fused. Complete syndactyly
encompasses the entire web space. In patients with incomplete syndactyly, only
part of the web space is fused.

Based on the clinical photograph alone, incomplete syndactyly can be diagnosed


in this infant because the entire web space is not involved. However, because there
are no additional radiographs, a diagnosis of simple or complex syndactyly cannot
be determined.

Although most infants with syndactyly undergo release before 18 months of age,
some surgeons have performed surgery as early as 6 weeks of age.

Infants with brachydactyly have short, fused digits. Camptodactyly is


characterized by curvature of a digit in its plane of flexion; in contrast,
clinodactyly describes deviation of the digit in the plane of the hand. Patients with
ectrodactyly have partial or total absence of the digits.

References
1. Ezaki M, Kay SP, Light T, et al. Congenital hand deformities. In: Green DP,
Hotchkiss RN, Pederson WC, eds. Operative Hand Surgery. 4th ed. New York,
NY: Churchill Livingstone, Inc; 1999;1:325-551.
2. Smith P, Laing H. Syndactyly. In: Gupta A, Kay SP, Scheker LR, eds. The
Growing Hand. London: Mosby - Year Book, Inc; 2000:225-230.

Q197
PIC

A 7-month-old infant has the deformity shown in the photographs above. Which of
the following is the most likely mode of inheritance of this deformity?

(A) Autosomal dominant


(B) Autosomal recessive
(C) Sporadic
(D) X-linked dominant
(E) X-linked recessive

The correct response is Option A.


This 7-month-old infant has a typical, or true, cleft hand deformity, which is most
commonly inherited as an autosomal dominant condition. However, cleft hand
may also occur as a spontaneous mutation, as it did in this patient whose family
members were unaffected.

Typical cleft hand deformity is classified as a longitudinal deficiency because it


involves dysplasia of the central portion of the hand. This anomaly is stratified
into five categories, based on the degree of ray absence and hypoplasia of the
thumb space. Characteristics include a V-shaped cleft, suppression of the central
digit, and sometimes minor syndactyly of the ulnar border digits.

In contrast, atypical cleft hand is sporadic and is included in the teratologic


sequence of symbrachydactyly. Affected patients have a broad, flat cleft hand with
finger nubbins containing nail remnants.

Because typical cleft hand may be associated with several syndromes, any infant
who exhibits this anomaly should be evaluated for cardiac, pulmonary,
musculoskeletal, ocular, and renal defects. EEC syndrome, involving ectrodactyly,
ectodermal dysplasia, and cleft lip/palate, is most frequently associated. Many
patients with typical cleft hand deformity also have clefting of the feet (as does
this patient), which results from localization of the split hand foot gene (SHFM1)
to chromosome 7q21.

The degree of deformity of typical cleft hand varies greatly. Although most
patients adapt and ultimately function quite well, surgical reconstruction is often
recommended for functional as well as cosmetic reasons. The Snow-Littler
procedure, which involves transposition of a palmar-based cleft flap with the index
ray to close the cleft and create a useful first web space, is an appropriate
reconstructive option.

References
1. Kay SP. Cleft hand. In: Green DP, Hotchkiss RN, Pederson WC, eds. Operative
Hand Surgery. 4th ed. New York, NY: Churchill Livingstone, Inc; 1999;1:402-
413.
2. Netscher DT. Congenital hand problems. Hand Clin. 1998;25:544.
3. Upton J. Congenital anomalies of the hand and forearm. In: McCarthy JG, ed.
Plastic Surgery. Philadelphia, Pa: WB Saunders Co; 1990;8:5267-5273.

Hand FC Disloc- 2004

A 20-year-old college basketball player sustains a dorsal dislocation of the


metacarpophalangeal joint of the dominant index finger. Repeat attempts at closed
reduction are unsuccessful. Which of the following structures are the most likely
cause of the unsuccessful closed reduction?

(A) Extensor indices, lumbrical muscle, and flexor digitorum profundus


(B) Flexor digitorum profundus, volar plate, and sagittal bands
(C) Lumbrical muscle, volar plate, and flexor digitorum profundus
(D) Sagittal bands, extensor indices, and lumbrical muscle
(E) Volar plate, sagittal bands, and extensor indices
Pathology of irreducible (complex) dislocations of the index metacarpophalangeal
joint. The metacarpal head protrudes volarly between the lumbrical (radially) and
the flexor tendons (ulnarly). The proximal phalanx and volar plate displace dorsal
to the metacarpal head, and the plate is folded and impinged within the previously
congruous articular surfaces. The flexor tendons remaining in the fibrous sheath
attached to the volar plate are kept taut by this displacement and thus maintain the
tight tendon-lumbrical encirclement around the narrow metacarpal neck. Traction
applied to attempt reduction further tightens this entrapment and makes closed
reduction impossible.

Reproduced with permission of Green DP, Hotchkiss RN, Pederson WC, eds.
Operative Hand Surgery. 4th ed. New York, NY: Churchill Livingstone, Inc;
1999;1:784.

The correct response is Option C.


The lumbrical muscle, volar plate, and flexor digitorum profundus tendons are
most likely interfering with reduction of the index finger dislocation. Dorsal
dislocations are uncommon injuries that occur as a result of forced hyperextension.
The membranous proximal portion of the volar plate ruptures and becomes
interposed dorsally between the base of the proximal phalanx and the dorsal
metacarpal head. Reduction can be accomplished by flexing the wrist, which
loosens the flexor tendons, and then applying pressure, directed distally and
volarly, to the base of the proximal phalanx. In patients with complex dislocations,
open reduction is required because the volar plate lies in the joint space, blocking a
successful closed reduction. With dislocations of the metacarpophalangeal joint,
the flexor digitorum profundus tendon lies ulnar to the joint and the lumbrical
muscle lies radial to the joint. The sagittal bands and extensor indices are dorsal
structures that are not involved.

References
1. Glickel SZ, Barron A, Eaton RG. Dislocations and ligament injuries in the
digits. In: Green DP, Hotchkiss RN, Pederson WC, eds. Operative Hand Surgery.
4th ed. New York, NY: Churchill Livingstone, Inc; 1999;1:772- 808.
2. Kiefhaber TR. Phalangeal dislocations / periarticular trauma. In: Peimer CA, ed.
Surgery of the Hand and Upper Extremity. New York, NY: McGraw-Hill, Inc;
1996;1:939-972.

A 17-year-old boy has tenderness over the metacarpal shafts of the ring and small
finger four weeks after undergoing closed reduction of a hand injury followed by
use of a hand-based splint. On physical examination, there is no rotational
deformity of the involved digits. Current radiographs are shown above.

Which of the following is the most appropriate management?

(A) Repeat closed reduction followed by application of a long arm cast


(B) Closed reduction and transcutaneous fixation with Kirschner wires
(C) Closed reduction and external fixation
(D) Open reduction and lag screw fixation
(E) Open reduction and miniplate fixation

The correct response is Option E.

This 17-year-old boy has displaced, angulated transverse fractures of the


metacarpal shafts of the ring and small fingers. Closed reduction of the fractures
and use of a hand-based splint for four weeks has not resulted in healing.
Therefore, operative treatment is the most appropriate next step. This includes
exploration to debride any material surrounding the fragments and open reduction
of the fractures. In addition, miniplate fixation is indicated for transverse
metacarpal fractures.
Repeat closed reduction and application of a long arm cast are unlikely to result in
a successful reduction or lead to fracture healing.
Closed reduction and transcutaneous fixation with Kirschner wires may have been
considered at the time of injury but are not appropriate four weeks later.

Closed reduction and external fixation are indicated in patients with highly
comminuted fractures with or without bone loss or fractures associated with soft-
tissue loss.

Lag screw fixation is appropriate for treatment of spiral and oblique metacarpal
fractures in which the length of the fracture is at least twice the diameter of the
bone. This technique is advantageous because it minimizes periosteal stripping.

References
1. Freeland AE. Hand Fractures: Repair, Reconstruction, and Rehabilitation. New
York, NY: Churchill Livingstone, Inc; 2000:14-65.
2. Stern PJ. Fractures of the metacarpals and phalanges. In: Green DP, Hotchkiss
RN, Pederson WC, eds. Operative Hand Surgery. 4th ed. New York, NY:
Churchill Livingstone, Inc; 1999;1:711-771.

A 35-year-old woman has an open fracture of the index finger metacarpal after
sustaining a gunshot wound to the hand. Physical examination shows a 2.5-cm
defect of the skin; radiographs show a 2-cm segmental defect of the metacarpal.
The patient is to undergo single-stage reconstruction using a distally based
posterior interosseous flap with vascularized bone.

Harvest of the flap with a cuff of which of the following muscles will maintain the
blood supply to the bone?

(A) Extensor carpi radialis brevis


(B) Extensor carpi ulnaris
(C) Extensor digiti minimi
(D) Extensor pollicis longus
(E) Supinator

The correct response is Option D.

The vascular supply of the ulna can be maintained by harvesting the distally based
posterior interosseous flap with a cuff of the extensor pollicis longus muscle. In
this patient who has a 2-cm segmental defect of the metacarpal, vascularized bone
from the ulna is appropriate for single-stage reconstruction. Other reconstructive
options include the radius, scapula, fibula, humerus, and iliac crest. The radial
forearm flap may also be harvested as an osteocutaneous flap for reconstruction of
the metacarpal.
To determine the cutaneous portion of the posterior interosseous flap, a line is
drawn from the lateral epicondyle of the humerus to the ulnar head with the
forearm in full pronation. The cutaneous branch of the posterior interosseous
nerve, which must be incorporated in the flap, lies 1 cm distal to the midpoint of
this line. The posterior interosseous artery lies deep to the deep fascia, and the
septum passes between the extensor carpi ulnaris and extensor digiti minimi. After
the artery has been identified, the surgeon dissects distally to the supinator, taking
care to identify and preserve the posterior interosseous nerve. A 5- to 7-cm
segment of bone can be harvested by dissecting through of a portion of the
extensor pollicis longus while leaving a cuff of muscle attached to the bone.

Although free iliac crest corticocancellous grafts can be used to reconstruct


metacarpal defects, vascularized bone graft is often preferred instead if the defect
is large or the soft-tissue envelope has poor quality.

The extensor carpi radialis brevis and supinator muscles do not supply perforators
to the ulna.

References
1. Akin S, Ozgenel Y, Ozcan M. Osteocutaneous posterior interosseous flap for
reconstruction of the metacarpal bone and soft-tissue defects in the hand. Plast
Reconstr Surg. 2002;109:982.
2. Angrigiani C, Grilli D, Dominikow D, et al. Posterior interosseous reverse
forearm flap: experience with 80 consecutive cases. Plast Reconstr Surg.
1993;92:285.
3. Yajima H, Tamai S, Yamauchi T, et al. Osteocutaneous radial forearm flap for
hand reconstruction. J Hand Surg. 1999;24A:594.

Hand Nerves- 2004

A 34-year-old man has radial nerve palsy six months after undergoing open
reduction and plate fixation of a fracture of the humeral diaphysis. The integrity of
the radial nerve was confirmed at the time of the initial injury. Which of the
following is the most appropriate next step in management?

(A) Dynamic extension splinting


(B) Injection of a corticosteroid
(C) Tendon transfers
(D) Interpositional nerve grafting
(E) Neurolysis

The correct response is Option C.


In patients with high radial nerve palsy, the primary goal is restoration of
extension of the wrist, fingers, and thumb. If the nerve was intact at the time of the
initial surgery and there is subsequently no return of function six months later,
further improvement is unlikely, and tendon transfers are indicated at this time.

Transfer of the pronator teres to the extensor carpi radialis brevis is frequently
performed to recover wrist extension. To regain finger extension, the flexor carpi
radialis, flexor carpi ulnaris, or flexor digitorum superficialis of the long or ring
fingers is transferred into the distal extensor digitorum communis tendons.
Transfers to regain thumb extension include the palmaris longus or flexor carpi
radialis to the extensor pollicis longus. Additionally, some surgeons advocate end-
to-side tendon transfers.

The extensor carpi radialis brevis cannot be transferred to the extensor digitorum
communis because it is also affected by the radial nerve palsy. Transfer of the
flexor digitorum profundus tendon of the long finger is associated with significant
loss of function at the donor site.

Splinting is not indicated in a patient who has persistent radial nerve palsy six
months after the initial procedure.

Injection of a corticosteroid is inappropriate treatment of radial nerve palsy.

Because the nerve is shown to be intact, the presence of a neuroma in continuity


may be inhibiting the reinnervation process. EMG should be performed to
determine the potential for nerve grafting or neurolysis in this patient. However,
most nerve grafting procedures in adults provide only limited improvement in
motor and sensory functions six months after denervation.

References
1. Green DP. Radial nerve palsy. In: Green DP, Hotchkiss RN, Pederson WC, eds.
Operative Hand Surgery. 4th ed. New York, NY: Churchill Livingstone, Inc;
1999;2:1481-1496.
2. Wheeler DR. Reconstruction for radial nerve palsy. In: Peimer CA, ed. Surgery
of the Hand and Upper Extremity. New York, NY: McGraw-Hill, Inc; 1996:1363-
1379.

A 23-year-old man sustained a complete laceration of the ulnar nerve at the level
of the elbow four weeks ago when he was stabbed in the nondominant forearm
with a knife. After surgical nerve preparation, a 2-cm gap is present. Which of the
following is the most appropriate next step in management?
(A) Use of a vein conduit
(B) Tendon transfers
(C) Mobilization of the nerve 15 cm proximally and distally
(D) Sural nerve grafting
(E) Ulnar nerve transposition

The correct response is Option E.

In this 23-year-old man who has a 2-cm nerve gap after sustaining a clean
laceration of the ulnar nerve at the elbow four weeks ago, the most appropriate
next step in management is transposition of the ulnar nerve. Because transposition
of this nerve at the elbow provides as much as 4 cm of length, it is recommended
in this patient in whom primary coaptation cannot be performed because of nerve
retraction.

Ulnar nerve transposition may yield as much as 3 cm of length when performed in


the arm, 2 cm of length at the forearm, and 1 to 2 cm of length at the distal forearm
and wrist. Transposition is also appropriate for repair of median and high radial
nerve injuries.

Vein conduits should only be considered if direct repair and transposition are not
options.

Tendon transfers alone do not restore sensory function, and are recommended for
late reconstruction only when nerve repair is no longer an option.

Extensive mobilization of the ulnar nerve into the mid forearm may cause
devascularization and injury to distal nerve branches and ultimately worsen
functional outcome.

Sural nerve grafting may be considered if transposition of the ulnar nerve results in
tension following nerve coaptation.

References
1. Strauch B. Use of nerve conduits in peripheral nerve repair. Hand Clin.
2000;16:123-130.
2. Trumble TE, McCallister WV. Repair of peripheral nerve defects in the upper
extremity. Hand Clin. 2000;16:37-52.

In a patient with established Volkmann’s ischemic contracture, reconstruction via


free transfer of the gracilis involves decompression of which of the following
nerves?
(A) Anterior interosseous
(B) Median
(C) Posterior interosseous
(D) Radial
(E) Ulnar

The correct response is Option B.

In a patient with Volkmann’s ischemic contracture of the forearm, the median


nerve is most likely compressed, and is thus most likely to require decompression,
because it traverses the center of the scarred muscles. After muscle infarction
occurs, peripheral nerves can become compressed within a constricting scar at a
specific anatomic location. Because improvement in nerve function is related to
the severity and duration of compression, early decompression is required to
minimize further dysfunction. As a result, the median nerve should be
decompressed as soon as the patient’s condition permits. In order to be successful,
operative exploration is necessary at all points of compression.

In patients with compartment syndrome, high interstitial pressures are typically


measured in the deepest compartments of the forearm, especially those
compartments that lie adjacent to bone. The flexor digitorum profundus and flexor
pollicis longus muscles are affected most often; in more severe cases, the flexor
digitorum superficialis, flexor carpi radialis, and flexor carpi ulnaris are involved
also. The median nerve often lies at the center of the a constricting scar in the
forearm and is thus at risk for compression at the lacertus fibrosus, the two heads
of the pronator teres, and the proximal arch of the flexor digitorum superficialis, as
well as within the carpal tunnel.

The anterior interosseous nerve is a branch of the median nerve and thus is not
involved in Volkmann’s ischemic contracture of the forearm. The posterior
interosseous nerve and the radial nerve are rarely involved. The ulnar nerve is
implicated less frequently than the median nerve and is often compressed at or just
distal to the elbow.

References
1. Botte MJ, Keenan MA, Gelberman RH. Volkmann’s ischemic contracture of the
upper extremity. Hand Clin. 1998;14:483-497.
2. Tsuge K. Management of established Volkmann’s contracture. In: Green DP,
Hotchkiss RN, Pederson WC, eds. Operative Hand Surgery. 4th ed. New York,
NY: Churchill Livingstone, Inc; 1999;1:592-603.
A 15-year-old girl has absence of sensation of the long finger and radial side of the
ring finger 18 months after undergoing operative repair of a partial injury to the
median nerve in the wrist. Sensation is normal in the thumb, index, and small
fingers and in the ulnar side of the ring finger. An intraoperative photograph is
shown above.

Which of the following is the most appropriate management?

(A) Internal neurolysis of the median nerve


(B) Excision of the neuroma only
(C) Excision of the neuroma and epineural repair
(D) Excision of the neuroma and sural nerve grafting
(E) Resection of the median nerve and epineural repair

The correct response is Option D.

In this patient who has a neuroma in-continuity, the most appropriate management
is excision followed by sural nerve grafting. Neuroma in-continuity is often
difficult to diagnose and treat. Serial clinical examination and electrodiagnostic
testing are essential for diagnosis. Although operative exploration can improve
hand function and result in a good outcome, the functional fascicles that lie
adjacent to the neuroma are at risk for injury. Nerve conduction velocity studies
should be performed intraoperatively to identify the nonfunctioning fascicles that
lead into
and out of the neuroma. The surgeon should take great care during excision of the
neuroma to avoid damaging the functional fascicles. Following excision,
autogenous grafting with a donor nerve such as the sural nerve should be
performed.

Internal neurolysis would not re-establish the continuity of the involved fascicles.
Simple excision of the neuroma will result in recurrence. Excision and epineural
repair would place excessive tension on the neurorrhaphy and potentially lead to
the development of another neuroma. Resection of the median nerve is an
excessive procedure that would eliminate the functional portion of the nerve.

References
1. Kline DG. Timing for exploration of nerve lesions and evaluation of neuroma-
in-continuity. Clin Ortho. 1982;163:42.
2. MacKinnon SE, Glickman LT, Dagum A. A technique for the treatment of
neuroma-in-continuity. J Reconstr Microsurg. 1992;8:379.

A 50-year-old man is undergoing evaluation because he has had weakness of grip


in the right hand for the past two months that is affecting his golf swing. Physical
examination shows limited abduction and adduction of the fingers of the right
hand. He has difficulty crossing the fingers. Sensation is diminished over the volar
aspect of the small finger and volar-ulnar aspect of the ring finger. Sensation is
normal over the radial digits and dorsal aspect of the hand.

This patient most likely has an nerve lesion at which of the following sites?

(A) Arcade of Frohse


(B) Arcade of Struthers
(C) Guyon’s canal
(D) Origin of the flexor carpi ulnaris
(E) Vascular leash of Henry

The correct response is Option C.

This patient has symptoms consistent with ulnar nerve compression, including
weakness of the intrinsic muscles of the hand innervated by the ulnar nerve (which
manifests as a loss of finger adduction and abduction) and evidence of muscle
atrophy. There is also decreased sensation in the distribution of the ulnar nerve.
The ulnar nerve is compressed most commonly in the region of the cubital tunnel;
the second most common site of compression is at Guyon’s canal in the wrist.
Sensation over the dorsoulnar hand, which is supplied by the dorsal branches from
the ulnar nerve arising proximal to Guyon’s canal, is tested to determine the
location of compression. If sensation is altered dorsally, the lesion lies proximal to
the distal forearm, and is most likely to involve the cubital tunnel. However, if
sensation in the dorsoulnar hand is normal, the lesion lies distal to the distal
forearm, and most likely involves Guyon’s canal, as in this patient.

Other, less common sites of compression of the ulnar nerve include the arcade of
Struthers, which is a thin aponeurotic band extending from medial head of the
triceps to the medial intermuscular septum, located approximately 8 cm proximal
to the medial epicondyle, and occasionally the origin of the flexor carpi ulnaris.
The arcade of Frohse and vascular leash of Henry are potential sites of
compression of the radial nerve.

When compressed, peripheral nerves typically cause pain and specific nerve
deficits of sensation and strength. Some sensory branches divide from the nerve
proximal to the wrist or at the level of the carpal canal. The dorsal sensory branch
of the ulnar nerve divides approximately 6 cm proximal to the wrist. The motor
branches of the median and ulnar nerves can be separated into extrinsic and
intrinsic function.

Reference
1. Eversmann WW Jr. Entrapment and compression neuropathies. In: Green DP,
ed. Operative Hand Surgery. 3rd ed. New York, NY: Churchill Livingstone, Inc;
1982:1341-1385.

A 27-year-old woman has intense, burning pain in the right arm 10 days after
sustaining a fracture of the right radius. Conservative treatment of the pain with
oral administration of narcotic agents has not been effective. At the time of injury,
a long arm cast was applied in the emergency department.

Which of the following is the most appropriate diagnostic test?

(A) Three-view radiographs of the wrist


(B) Stellate ganglion block
(C) Thermography of the upper extremity
(D) Triple-phase bone scan
(E) MRI of the wrist

The correct response is Option A.

This patient has symptoms consistent with complex regional pain syndrome type I,
or reflex sympathetic dystrophy (RSD), a complex alteration of the pain response
following trauma. In contrast, patients with complex regional pain syndrome type
II, or causalgia, have posttraumatic pain resulting from an identifiable nerve
injury. While its exact cause is unknown, RSD is characterized by pain, stiffness,
limited function, atrophic changes, and vasomotor instability. Early diagnosis and
treatment are essential for optimal functional outcome; the surgeon must also
differentiate RSD from other, treatable conditions.

Although fractures of the distal radius are a common precipitating factor for RSD,
the pain may actually be caused by fracture nonunion or an excessively tight cast.
Therefore, appropriate initial management involves removing the cast and
obtaining three-view radiographs of the wrist to determine the adequacy of
fracture reduction. This should be performed before any of the other diagnostic
tests listed.

Stellate ganglion blocks are used in the treatment of RSD and can be performed
diagnostically, but simple radiographs should be obtained first. Thermography has
been advocated for diagnosis of this condition but has low sensitivity and
specificity. Triple-phase bone scans are important for evaluation and have a
relatively high specificity for diagnosis, but should not be performed initially. MRI
is ineffective in diagnosis and management of RSD.

References
1. Amadio PC, MacKinnon SE, Merritt WH, et al. Reflex sympathetic dystrophy
syndrome: consensus report of an ad hoc committee of the American Association
of Hand Surgery on the definition of reflex sympathetic dystrophy syndrome. Plast
Reconstr Surg. 1991;87:371.
2. Koman LA, Poehing GG, Smith BP, et al. RSD after wrist injury. In: Levin LS,
ed. Problems in Plastic and Reconstructive Surgery: The Wrist. Philadelphia, Pa:
JB Lippincott; 1992:300-321.

Hand Tendons- 2004

A 4-year-old girl has a fixed flexion deformity of the thumb of the dominant right
hand as well as a palpable nodule at the volar metacarpophalangeal joint that has
been present since birth. Which of the following interventions is the most
appropriate initial step?

(A) A1 pulley release


(B) Aspiration of the mass
(C) Biopsy of the nodule
(D) Injection of cortisone into the mass
(E) Transfer of the extensor indicis proprius tendon to the extensor pollicis longus
tendon

The correct response is Option A.

The first step in management of this deformity is A1 pulley release. The patient
has a congenital trigger thumb, the most common cause of congenital thumb
flexion deformity. The flexed position of the thumb can also be seen in patients
with congenital clasped thumb, absent or aberrant extensor tendons,
arthrogryposis, and spasticity. Patients with congenital trigger thumb commonly
have thickening of the tendon, referred to as “Notta’s node.” In patients younger
than age 3 years, the spontaneous resolution rate is 30%. Splinting and observation
are options when the condition is diagnosed early, but most patients require
surgery. During release of the pulley, no attempt is made to excise or reduce the
nodule in the tendon. Aspiration is not appropriate for the nodule in the flexor
tendon.

Aspiration can be used to manage retinacular cysts, which are ganglion cysts on
the tendon sheath seen at the volar metacarpophalangeal joint, but patients with
such cysts do not have flexion deformity. Biopsy is not necessary for Notta’s node,
which is a pathologic thickening of the flexor sheath. Such pathologic changes in
the flexor tendon are more common in children than in adults, who more
commonly have involvement of the tendon sheath.

Trigger digit injection is into the flexor sheath and not the mass. Risk of rupture is
higher with direct injection into the mass. Injecting a child would require at least
monitored deep sedation if not general anesthesia. Tendon transfer is appropriate
for treatment in patients with absent or aberrant thumb extensor tendons.

References:
1. Wolfe SW. Tenosynovitis. In: Green DP, Hotchkiss RN, Pederson WC, eds.
Green’s Operative Hand Surgery. Vol 2. 4th ed. New York: Churchill Livingstone,
1998:2022-2044.
2. Gropper PT. Small joint contractures. In: Peimer CA, ed. Surgery of the Hand
and Upper Extremity. Vol 2. New York: McGraw-Hill Professional Publishing;
1996:1583-1600.

A 16-year-old football player sustains an injury to the right ring finger when he
tries to tackle another player during a game. On the sideline, physical examination
shows tenderness along the proximal interphalangeal and distal interphalangeal
joints and inability to flex the distal phalanx. Radiographs show no abnormalities.
The most likely diagnosis is avulsion of which of the following?

(A) Flexor digitorum profundus tendon in Zone I


(B) Flexor digitorum profundus tendon in Zone II
(C) Flexor digitorum superficialis tendon in Zone I
(D) Flexor digitorum superficialis tendon in Zone II
(E) One slip of the flexor digitorum superficialis tendon in Zone II

The correct response is Option A.

This patient has sustained a “jersey finger” injury. The digital cascade is disrupted
because there is no flexion force at the distal phalanx of the ring finger. In this
patient, the flexor digitorum profundus tendon became avulsed from the distal
phalanx (Zone I) when the patient attempted to actively flex the distal
interphalangeal joint with the finger in forced extension. Three types of avulsion
injuries to the flexor digitorum profundus tendon have been described:

Type I: The flexor digitorum profundus tendon retracts into the palm
Type II: The flexor digitorum profundus tendon retracts to the proximal
interphalangeal joint
Type III: The flexor digitorum profundus tendon is entrapped at the A4 pulley

The patient is still able to flex the proximal interphalangeal joint, but the joint may
be tender if there is blood in the flexor sheath or if he has sustained a Type II
injury. The distal phalanx is drawn into extension because of the unopposed pull
of the terminal tendon.

Rupture of one slip of the flexor digitorum superficialis tendon would not result in
loss of function of the flexor digitorum profundus tendon or flexion of the distal
interphalangeal joint.

References:
1. Schneider LH. Flexor tendonsClate reconstruction. In: Green DP, Hotchkiss
RN, Pederson WC, eds. Green’s Operative Hand Surgery. Vol 2. 4th ed. New
York: Churchill Livingstone, 1998:1935.
2. Britton EN, Kleinhart JM. Acute flexor tendon injury: repair and rehabilitation.
In: Peimer CA, ed. Surgery of the Hand and Upper Extremity. New York:
McGraw-Hill Professional Publishing; 1996.

A 27-year-old woman has pain and instability on radial-directed stress in the right
thumb eight months after sustaining an injury to the ulnar collateral ligament.
Initial management of the injury consisted of thumb spica casting for six weeks. In
this patient, which of the following anatomic structures is most likely interposed
between the ulnar collateral ligament and the proximal phalanx?

(A) Abductor tendon


(B) Adductor aponeurosis
(C) First dorsal interosseous muscle
(D) Flexor pollicis longus
(E) Joint capsule

The correct response is Option B.


This patient has a classic Stener lesion, in which the adductor aponeurosis
becomes interposed between the ruptured ulnar collateral ligament and the base of
the proximal phalanx. This lesion cannot heal spontaneously because reattachment
of the ulnar collateral ligament to the proximal phalanx is blocked by the
interposed tendon. It is important to identify this condition at the time of the initial
injury so that operative repair can be performed without delay. Without surgical
correction, the patient will have painful instability to radial stress of the
metacarpophalangeal joint. The abductor tendon, the first dorsal interosseous
muscle, and the flexor pollicis longus are not involved because they are not in the
vicinity of the ulnar collateral ligament of the metacarpophalangeal joint of the
thumb. Tearing of the joint capsule is a potential complication of a Stener lesion,
and an unrepaired capsular tear can be associated with relative supination of the
proximal phalanx in relation to the metacarpal. Although the joint capsule should
always be inspected, and repaired if necessary, at the time of ligament repair, it is
not involved with blocking of the ulnar collateral ligament. In this injury, only the
adductor aponeurosis becomes interposed between the ulnar collateral ligament
and the proximal phalanx.

References:
1. Glickel SZ, Barron OA, Eaton RG. Dislocations and ligament injuries in the
digits. In: Green DP, Hotchkiss RN, Pederson WC, eds. Green=s Operative Hand
Surgery. Vol 1. 4th ed. New York: Churchill Livingstone, 1998:788-794.
2. Durham JW. Thumb metacarpophalangeal ulnar collateral ligament repair with
local tissues. In: Blair WF, ed. Techniques in Hand Surgery. Baltimore: Williams
& Wilkins; 1996:533-537.

Hand Tendons- 2004

A 12-year-old boy with spastic cerebral palsy and moderate mental retardation is
undergoing evaluation of hand function. On physical examination, the arms are
held in a reducible posture, with the elbows flexed, the forearms pronated, and the
wrists flexed. The ulnar digits are flexed tightly in the palm, and the thumb is held
against the index and long fingers. There is ulnar palmar maceration and
wounding. Moving two-point discrimination is 12 to 14 mm bilaterally.

Which of the following tendon transfers is most appropriate?

(A) Flexor carpi ulnaris to extensor carpi radialis brevis


(B) Flexor carpi ulnaris to flexor digitorum profundus
(C) Flexor digitorum profundus to extensor digitorum communis
(D) Flexor digitorum superficialis to flexor digitorum profundus
(E) Pronator teres to extensor carpi radialis longus

The correct response is Option D.


Patients with cerebral palsy typically have cognitive, developmental, and sensory
deficits that affect motor function. Cerebral palsy can be characterized as spastic,
with fluctuations in muscle tone, or as a motion disorder, involving ataxia,
dyskinesia, and tremors. Although improvement is difficult to achieve in those
affected by motion disorders, some children with spastic cerebral palsy have
enhanced hand function following surgery.

Important elements included in the preoperative testing of patients with cerebral


palsy are evaluation of cognition, hand placement, and sensibility. In patients who
fail to meet the minimum criteria in these areas, the primary goal of surgery is
improvement of hygiene and not necessarily of hand function.

Cognition is determined by intelligence quotient (IQ) testing; children can be


classified as normal (within a standard deviation of the norm), educable (IQ of 50
to 70), and trainable (IQ of 30 to 50).

Hand placement is measured by asking the child to place one hand on the head
initially and then on the opposite knee. This maneuver tests range of motion,
precision of placement, and time required to complete the task. Typically, only
those children who can perform this task within five seconds can be expected to
benefit from surgery.

Sensibility testing varies according to the age of the child. Texture discrimination
is the recommended test for children 2 to 3 years of age; object identification is
appropriate for those from 4 to 5 years of age, and graphesthesia is tested in
children ages 6 to 9 years. In children older than 9 years, sensibility is tested by
measuring moving two-point discrimination. Functional improvement following
surgery can only be expected in those children who can successfully discriminate
texture, identify objects, or exhibit graphesthesia, or those who have moving two-
point discrimination of less than 10 mm.

Because this 12-year-old boy with spastic cerebral palsy has both poor cognition
and sensibility, improvement of hygiene will be the primary goal of any surgical
procedure. Therefore, transfer of the flexor digitorum superficialis tendon to the
flexor digitorum profundus tendon is recommended to eliminate the clenched fist
deformity and relieve the maceration and disintegration of skin that is typically
associated with this deformity. This tendon transfer lengthens yet weakens the
finger flexors.

Transferring the flexor carpi ulnaris to the flexor digitorum profundus will only
increase the flexion force and aggravate the deformity. Transfer of the flexor
digitorum profundus to the extensor digitorum communis will not resolve the
clenched fingers. In patients who have a deformity of this severity, arthrodesis of
the wrist is recommended instead of tendon transfers to the wrist extensors.
References
1. Hoffer MM. Cerebral palsy. In: Green DP, ed. Operative Hand Surgery. 3rd ed.
New York, NY: Churchill Livingstone, Inc; 1982:215-223.
2. Koman LA, Gelberman RH, Toby EB, et al. Cerebral palsy: management of the
upper extremity. Clin Ortho. 1990;253:62-74.
3. Van Heest AE, House JH, Cariello C. Upper extremity surgical treatment of
cerebral palsy. J Hand Surg. 1999;24:323-330.

An 18-year-old college student sustains an injury to the nondominant small finger


when he falls while playing flag football. On physical examination, the finger is
swollen slightly and held in a flexed position. Active flexion and extension of the
finger are limited by pain. A radiograph is shown above. Closed reduction of the
finger in the emergency department is unsuccessful.

The most likely cause is interposition of which of the following structures within
the joint?

(A) Extensor tendon


(B) Flexor tendon
(C) Joint capsule
(D) Neurovascular bundle
(E) Volar plate
The correct response is Option A.

This 18-year-old college student has sustained a volar dislocation of the proximal
interphalangeal (PIP) joint, an injury so named because the middle phalanx is
dislocated volar to the proximal phalanx. In volar dislocations, which are far less
common than dorsal dislocations, the extensor tendon is torn by the distal condyle
of the proximal phalanx, as shown in the intraoperative photograph above. If the
condyle has pushed through the extensor tendon, the tendon may tighten and act as
a sling to prevent reduction of the dislocation.

Although some physicians may be able to perform closed reduction, operative


exploration and repair are recommended because of the possible injury to the
extensor tendon. Unsuccessful closed reduction is an absolute indication for
operative exploration, as repeat attempts at manipulation will only worsen edema
and may contribute to injury. In addition, volar dislocations can be associated with
complete avulsion of the central slip, requiring repair.

Although the flexor tendons and volar plate can become interposed within the joint
and interfere with closed reduction, this is a rare finding in patients with volar
dislocations and is more likely to be associated with irreducible dorsal
dislocations. The joint capsule and neurovascular bundle are not involved in volar
dislocations of the PIP joint.

References
1. Concannon MJ, Hurov J, eds. Hand Pearls. Philadelphia, Pa: Hanley & Belfus;
2002;146-149.
2. Dray GJ, Eaton RG. Dislocations and ligament injuries in the digits. In: Green
DP, ed. Operative Hand Surgery. 3rd ed. New York, NY: Churchill Livingstone,
Inc; 1982:773-774.
3. Liss FE, Green SM. Capsular injuries of the proximal interphalangeal joint.
Hand Clin. 1992;8:755-768.
4. Wang KC, Hsu KY, Shih CH. Irreducible volar rotatory dislocation of the
proximal interphalangeal joint. Orthop Rev. 1994;23:886-888.

Repair of flexor tendon injuries in which of the following zones is most commonly
associated with a good prognosis?

(A) Zone 1
(B) Zone 2
(C) Zone 3
(D) Zone 4
(E) Zone 5

The correct response is Option E.

Flexor tendon injuries in zone 5 have the best prognosis following repair. This
zone, one of five in the flexor tendon system, lies proximal to the carpal tunnel.
The generous space found proximal to the wrist allows for better tendon gliding
following repair.

In contrast, zone 2 is an especially tight region that contains both flexor tendons
within the fibro-osseous tunnel. There is an increased risk for development of
adhesions when flexor tendons are repaired in this zone.

Repair of flexor tendon injuries in zones 1, 3, and 4 typically results in


intermediate outcomes because of tendon vascularity and gliding potential in these
zones.

References
1. Schneider LH. Flexor tendons – late reconstruction. In: Green DP, Hotchkiss
RN, Pederson WC, eds. Operative Hand Surgery. 4th ed. New York, NY:
Churchill Livingstone, Inc; 1999;2:1898-1949.
2. Strickland JW. Flexor tendons – acute injuries. In: Green DP, Hotchkiss RN,
Pederson WC, eds. Operative Hand Surgery. 4th ed. New York NY: Churchill
Livingstone, Inc; 1999;2:1851-1897.

Hand Tip Amp Dup- 2004


A 25-year-old machinist sustains severe crush injuries to the index and long
fingers of the dominant right hand. Physical examination shows avulsion of the
volar skin pad of the index finger to the distal interphalangeal joint crease. Bone is
exposed, and the nail and nail bed are intact. The dorsal skin of the long finger is
avulsed, and the paratenon is exposed.

Which of the following is most appropriate for reconstruction of the index finger?

(A) Skin graft


(B) Moberg advancement flap
(C) Reverse cross-finger flap
(D) Thenar flap
(E) Volar V-Y advancement flap

The correct response is Option D.

The thenar flap is most appropriate for reconstruction of this patient’s defect. This
flap can be used to effectively reconstruct defects of the tips of the index and long
fingers, which flex comfortably into the thenar eminence. In contrast, because the
ring and small fingers have difficulty reaching the thenar crease, defects of these
fingertips can be covered instead using a hypothenar flap from the ulnar side of the
hand.

Split-thickness or full-thickness skin grafting is inappropriate over exposed bone,


and padding is required for coverage of any fingertip defect.

Moberg flaps are recommended for coverage of soft-tissue defects of the volar pad
of the thumb. The dorsal circulation of the thumb allows for the extensive soft-
tissue mobilization required with this flap. The neurovascular bundles are elevated
with the Moberg flap.

A reverse cross-finger flap is used to cover defects of the dorsal aspect of the
finger. With this flap, subcutaneous tissue is harvested from the dorsal and not the
volar aspect of the finger; therefore, the neurovascular bundles are not disrupted.
A cross-finger flap cannot be used in this patient because the dorsal skin of the
long finger is avulsed.
Likewise, a volar V-Y advancement flap is not possible because the volar skin pad
of the index finger has also been avulsed. Although the dorsal skin is intact, it
should not be used for coverage because the patient would like to preserve finger
length, and because the risk for development of a hook nail deformity would be
increased if the dorsal skin were transferred.

References
1. Browne EZ Jr. Skin grafts. In: Green DP, Hotchkiss RN, Pederson WC, eds.
Operative Hand Surgery. 4th ed. New York, NY: Churchill Livingstone, Inc;
1999;2:1759-1782.
2. Lister GD, Pederson WC. Skin flaps. In: Green DP, Hotchkiss RN, Pederson
WC, eds. Operative Hand Surgery. 4th ed. New York, NY: Churchill Livingstone,
Inc; 1999;2:1783-1850.

A 63-year-old man has the deformity of the left small finger shown in the
photographs above. It has worsened over the past two years, and he has limited
passive and active extension of the proximal interphalangeal (PIP) joint of the
finger. The most likely cause is contracture of which of the following cords?

(A) Central and lateral


(B) Central and pretendinous
(C) Natatory and lateral
(D) Natatory and pretendinous
(E) Natatory and spiral

The correct response is Option A.

In this 63-year-old man who has limited extension of the proximal interphalangeal
(PIP) joint of the left small finger that has occurred as a result of a Dupuytren
contracture, the central and lateral cords are the most likely cause. Diseased cords
evolve from the normal fascial bands of the hand in patients with Dupuytren
disease, leading to flexion deformities of the affected joints. Fascial structures of
the hand that may contribute to Dupuytren contracture include Grayson’s and
Cleland’s ligaments, the lateral digital sheath, the natatory ligament, the
pretendinous and spiral bands, and the superficial transverse ligament.
The central, lateral, and spiral cords cause flexion contractures of the PIP joint.
The central cord arises from the pretendinous band, and the lateral cord is formed
from the central digital sheath. The spiral cord is composed of the pretendinous
and spiral bands, lateral digital sheath, and Grayson’s ligament. Contraction of the
spiral cord results in medial and superficial displacement of the neurovascular
bundle.

In addition to the contracture of the PIP joint caused by the central and lateral
cords, this patient’s hand posture results from contracture of the
metacarpophalangeal joint caused by the action of the pretendinous cord. The
pretendinous cord does not contribute to contractures of the PIP joint.

The natatory cord is formed from the natatory ligament as it passes transversely
across the palm at the level of the web spaces. It causes adduction, not flexion,
contractures of the digits.

References
1. McFarlane R. Patterns of diseased fascia in the fingers of Dupuytren’s
contracture. Plast Reconstr Surg. 1974;54:31.
2. McFarlane RM. The anatomy of Dupuytren's disease. Bulletin Hosp Jt Dis
Orthop Inst. 1984;44:318-337.
3. McFarlane RM. Dupuytren’s contracture. In: Green DP, ed. Operative Hand
Surgery. 3rd ed. New York, NY: Churchill Livingstone, Inc; 1982:563-591.
4. Strickland JW, Leibovic SJ. Anatomy and pathogenesis of the digital cords and
nodules. Hand Clin. 1991;7:645-671.

Six months after sustaining a traumatic amputation of the right index finger at the
level of the distal interphalangeal joint, a 27-year-old machinist has extension of
the proximal interphalangeal joint of the index finger when he attempts to make a
fist. Revision amputation and primary closure were performed at the time of the
initial injury, and the patient has undergone occupational therapy for the past six
months.

Which of the following is the most appropriate management?

(A) Osteotomy of the middle phalanx


(B) Release of the sagittal bands
(C) Sectioning of the lumbrical muscle
(D) Tenolysis of the profundus tendon
(E) Transfer of the interosseous muscle

The correct response is Option C.


In this 27-year-old machinist who has a lumbrical-plus deformity secondary to
release of the flexor digitorum profundus tendon to the index finger, the most
appropriate management is sectioning or division of the lumbrical muscle.
Because the profundus tendon to the index finger is independent, proximal
retraction causes proximal retraction of lumbrical tendon, leading to increased
tension. With attempted flexion of the proximal interphalangeal (PIP) joint (ie, to
make a full fist), the lumbrical muscle migrates more proximally, exerting tension
on the extensor mechanism through the lateral band. Paradoxical extension of the
interphalangeal joints results. Sectioning of the muscle can be performed as an
outpatient procedure using a local anesthetic.

Osteotomy of the middle phalanx will not correct the muscle-tendon imbalance.
Release of the sagittal bands is most likely to result in subluxation of the extensor
tendons across the metacarpophalangeal joint. Tenolysis of the profundus tendon
is appropriate for management of flexion contractures with adhesions, and transfer
of the interosseous muscle is performed for correction of ulnar drift in patients
with rheumatoid arthritis.

References
1. Failla JM. Differential diagnosis of hand pain: tendinitis, ganglia, and other
syndromes. In: Peimer CA, ed. Surgery of The Hand and Upper Extremity. New
York, NY: McGraw-Hill, Inc; 1996;1:1223-1249.
2. Louis DS, Jebson PJ, Graham T. Amputations. In: Green DP, Hotchkiss RN,
Pederson WC, eds. Operative Hand Surgery. 4th ed. New York, NY: Churchill
Livingstone, Inc; 1999;1:48-94.

A 5-year-old girl sustains a stellate laceration of the sterile matrix of the nail bed
of the left long finger when she closes a door on the finger. Which of the following
is the most appropriate management?

(A) Allowing the nail bed to heal by second intention


(B) Primary repair of the nail bed
(C) Debridement of the nail bed and split-thickness skin grafting
(D) Debridement of the nail bed and split germinal matrix nail grafting
(E) Coverage of the nail bed with a free flap from the great toe

The correct response is Option B.

In this patient who has sustained a stellate laceration of the nail bed of the long
finger, the most appropriate management is primary repair of the nail bed.
Lacerations of the nail bed are common injuries that most frequently occur in the
long finger, as it is typically the last digit to be moved during a situation of
potential trauma to the hand. Injury to the nail bed can be classified as simple
laceration, stellate laceration, avulsion, crush injury, or amputation.

The most appropriate management of simple and stellate lacerations of the sterile
matrix is primary repair. These injuries are associated with the best prognosis; the
nail typically has a normal appearance after healing.

In patients with avulsion and crush injuries, the outcome is often variable, as
fracture of the distal phalanx may be associated. Any fracture that occurs must be
reduced initially to eliminate irregular bone contours, which often result in a nail
deformity. If there is contamination of the nail bed, the necrotic tissue is debrided.
Split matrix grafting, using sterile grafts obtained from an adjacent portion of
uninjured nail bed, can be performed for reconstruction. A split nail bed graft can
be harvested also from the great toe.

Healing by second intention would result in deformity of the nail bed.

Germinal matrix grafts are appropriate for repair of trauma to the germinal matrix
of the nail. These are full-thickness grafts that leave a deformity at the donor site
following harvest.

Reconstruction of the nail bed with a free flap is reserved for management of
chronic deformities of the nail and is not considered in patients with acute injuries.

References
1. Shepard GH. Nail grafts for reconstruction. Hand Clin. 1990;6:79-102.
2. Zook EG. Anatomy and physiology of the perionychium. Hand Clin.
2002;18:553-559.
3. Zook EG, Guy RJ, Russell RC. A study of nail bed injuries: causes, treatment,
and prognosis. J Hand Surg. 1984;9A:247-252.
4. Zook EG, Van Beek AL, Russell RC, et al. Anatomy and physiology of the
perionychium: a review of the literature and anatomic study. J Hand Surg.
1980;5:528-536.
A 46-year-old woman sustains a ring avulsion injury to the long finger when the
finger becomes caught in a machine. Emergent revascularization is performed; on
examination 10 days later, the patient has the findings shown in the photographs
above. Which of the following techniques is most likely to provide optimal
function?

(A) Resection of all nonviable soft tissue and coverage with a full-thickness skin
graft
(B) Resection of all nonviable soft tissue and coverage with a neurovascular island
flap from the ring finger
(C) Resection of all nonviable soft tissue and reconstruction with a toe-to-hand
transfer
(D) Revision amputation at the level of the mid proximal phalanx, with trimming
of the bone to a level at which it can be covered primarily by viable skin
(E) Ray amputation of the long finger, leaving the base of the metacarpal in place
The correct response is Option E.

Ring avulsion injuries are typically associated with the highest failure rates
following replantation, most likely because of the mechanism of injury, which
involves destruction of the intimal layer of the supporting vasculature. In this
patient, revascularization has failed, leaving a necrotic digit.

The most appropriate next step in management of this patient is ray amputation,
which involves removal of the entire digit and most or all of the metacarpal.
Completely removing the digit eliminates the segmental loss and greatly improves
both function and aesthetic appearance, as shown in the photographs above.

Resection of nonviable soft tissue results in exposure of the phalanges and


tendons. A full-thickness skin graft will not take over these exposed structures.

The Littler neurovascular island flap is based on the digital neurovascular bundle
of either the long or ring finger. This flap provides sensate coverage of smaller
digital defects, particularly the thumb, but would not cover the entire defect in this
patient.

Replacing the long finger with a toe-to-hand transfer is impractical, as the


transferred digit would be significantly shorter than the adjacent digits and would
ultimately impair their function. This technique is appropriate for patients who
have sustained amputations of the thumb or of multiple digits.

Simple revision amputation is the easiest method of skin closure but leaves a large
gap between the long and small fingers, allowing an area through which small
objects can fall, and thus limiting hand function.
References
1. Concannon MJ, Hurov J, eds. Hand Pearls. Philadelphia, Pa: Hanley & Belfus;
2002;141-145.
2. Levy HJ. Ring finger ray amputation: a 25-year follow-up. Am J Orthop.
1999;28:359-360.
3. Louis DL. Amputations. In: Green DP, ed. Operative Hand Surgery. 3rd ed.
New York, NY: Churchill Livingstone, Inc; 1982:62-72.
4. Peimer CA, Wheeler DR, Barrett A, et al. Hand function following single ray
amputation. J Hand Surg. 1999;24:1245-1248.

A 60-year-old mechanic is brought to the emergency department 12 hours after


sustaining a amputation of the nondominant left thumb at the level of the
metacarpophalangeal joint. At the time of injury, the amputated part was
immediately placed in a plastic bag over an ice slush. He underwent arthroplasty
of the carpometacarpal joint of the left hand five years ago.

Which of the following factors is most likely to influence the success of


replantation in this patient?

(A) Age of the patient


(B) Hematocrit of the patient
(C) Length of ischemia time
(D) Mechanism of injury
(E) Previous hand surgery

The correct response is Option D.

In addition to the experience of the surgeon, the mechanism of injury is most


likely to influence the success of replantation in any patient who has sustained an
amputation of a digit. Because the vessels of the amputated part are damaged in
patients who sustain avulsion and crush amputations, perfusion will be impeded,
decreasing the likelihood of successful replantation. In addition, cooling of the
amputated part contributes to the success of the replantation, as the amputated part
can be replanted as late as 24 hours after injury if it is well preserved.

The age of the patient does not affect the success rate of replantation in the
absence of other comorbid conditions or a history of cigarette smoking. Likewise,
hematocrit and a history of hand surgery generally do not influence the outcome of
replantation significantly. Because digits do not contain muscle, the length of
ischemia time is not an influential factor.
References
1. Goldner RD, Urbaniak JR. Replantation. In: Green DP, Hotchkiss RN, Pederson
WC, eds. Operative Hand Surgery. 4th ed. New York, NY: Churchill Livingstone,
Inc; 1999;1:1139-1157.
2. Soucacos PN. Indications and selection for digital amputation and replantation.
J Hand Surg. 2001;26B:572-581.

A 22-year-old woman has the split nail deformity shown in the photograph above.
On physical examination, the deformity involves the sterile and germinal matrices.
The patient does not want to lose the nail. Which of the following is the most
appropriate management?

(A) Excision of the scar and primary closure of the nail bed
(B) Split nail grafting from the same nail bed
(C) Split nail grafting from the toe
(D) Full-thickness nail grafting from the finger
(E) Full-thickness nail grafting from the toe

The correct response is Option E.

In this patient who has a split nail deformity, the most appropriate management is
full-thickness nail grafting from the toe. This deformity is caused by injury to the
nail bed, leading to scarring of the bed. The nail plate does not grow in the scarred
area, resulting in a split in the nail plate.

Because the deformity involves both the sterile and germinal matrices, only a full-
thickness nail will provide the sterile and germinal matrix components required for
reconstruction. Harvest of a full-thickness nail produces a significant cosmetic
defect at the donor site; therefore, a graft from the second toe is thought to provide
the least unsightly result.

In patients who have a small scar affecting the sterile matrix only, appropriate
management may include excision of the scar and re-approximation of the sterile
matrix; however, the sterile matrix is not usually mobilized and re-approximated
unless the affected area is narrow. In addition, the germinal matrix cannot tolerate
re-approximation.

As mentioned above, a split nail graft from either the same nail bed or another nail
bed will not provide the components needed for reconstruction of this defect. In
addition, using another finger as a donor will result in an unsightly donor defect in
the hand.

References
1. Zook EG, Brown RE. The perionychium. In: Green DP, Hotchkiss RN,
Pederson WC, eds. Operative Hand Surgery. 4th ed. New York, NY: Churchill
Livingstone, Inc; 1999;1:1353-1380.
2. Zook EG. Surgically treatable problems of the perionychium. In: McCarthy JG,
ed. Plastic Surgery. Philadelphia, Pa: WB Saunders Co; 1990;8:4499-4515.

A 57-year-old man has a flexion contracture involving the ring and small fingers
of the left hand. A photograph is shown on page 178. During partial fasciectomy
in this patient, the neurovascular bundle to these fingers is at risk for injury. Each
of the following structures is a normal component of the fascia surrounding the
neurovascular bundle EXCEPT

(A) Cleland’s ligament


(B) Grayson’s ligament
(C) lateral digital sheet
(D) pretendinous band
(E) retrovascular band
The correct response is Option D.

Fascial structures that encase the neurovascular bundles include Cleland’s and
Grayson’s ligaments, the lateral digital sheet, and the retrovascular band. Cleland’s
ligament is a thick fascial structure that lies deep to the neurovascular bundle; it
arises from the side of the phalanges and courses obliquely toward the skin.
Grayson’s ligament is thinner and more sheet-like than Cleland’s ligament, and is
positioned superficial to the neurovascular bundle; it arises from the tendon sheath
and extends to the skin. The lateral digital sheet is comprised of superficial fascia
on either side of the phalanges. It receives fibers from the natatory ligament and
the spiral band, and is found lateral to the neurovascular bundle. The retrovascular
band is a longitudinal structure situated dorsomedial to the neurovascular bundle.

The pretendinous cord is a pathologic component of Dupuytren disease. It is an


extension of the pretendinous band in the midline of the phalanges. The
pretendinous band is not a component of the fascia surrounding the neurovascular
bundles.

References
1. Boyer MI, Gelberman RH. Complications of the operative treatment of
Dupuytren’s disease. Hand Clin. 1999;15:161-166.
2. Rayan GM. Palmar fascial complex anatomy and pathology in Dupuytren’s
disease. Hand Clin. 1999;15:73-86.

Hand Wrist - 2004

The radiographs shown on page 140 are from a 45-year-old dock worker who has
had worsening pain and loss of motion and strength in the dominant right wrist
over the past two years. Which of the following is the most appropriate operative
procedure?
(A) Scapholunate ligament repair
(B) Radial styloidectomy
(C) Radial corrective osteotomy
(D) Ulnar shortening osteotomy
(E) Four-corner arthrodesis

The correct response is Option E.

This patient has scapholunate advanced collapse of the wrist, also known as SLAC
wrist, with radioscaphoid arthrosis and a dorsiflexed intercalated segment
instability deformity. This condition is caused by incompetency of the
scapholunate ligament, which leads to rotatory subluxation of the scaphoid.

SLAC is the most common cause of degenerative arthritis of the wrist. This
condition can be classified according to four stages, as shown in the table below.

Stage I Radioscaphoid
Stage II Radial midcarpal
Stage III Ulnar midcarpal
Stage IV Pancarpal

Surgical management is aimed at decreasing pain and optimizing wrist function,


using two unaffected articular surfaces if possible. In this patient, goals of surgery
include eliminating the radioscaphoid articulation and stabilizing carpal
kinematics. Therefore, the most appropriate procedure is scaphoid excision and
four-corner arthrodesis between the lunate, capitate, hamate, and triquetrum. None
of the other procedures addresses all of the problems that are present in this
patient’s wrist.

References
1. Blatt G, Tobias B, Lichtman DM. Scapholunate injuries. In: Lichtman DM,
Alexander AH, eds. The Wrist and Its Disorders. Philadelphia, Pa: WB Saunders
Co; 1997:268-306.
2. Weber ER, Hixson M, Frazier GT. Chronic wrist instability. In: Peimer CA, ed.
Surgery of the Hand and Upper Extremity. New York, NY: McGraw Hill, Inc;
1996:727-758.

A 39-year-old assembly line worker has chronic disabling pain of the


trapeziometacarpal joint of the dominant right hand that is exacerbated when he
uses his tools. Radiographs show significant degenerative arthritis that is limited to
the trapeziometacarpal joint.

Which of the following is the most appropriate operative management?


(A) Reconstruction of the palmar oblique (beak) ligament
(B) Trapeziectomy only
(C) Trapeziectomy with tendon interposition arthroplasty
(D) Trapeziectomy with Silastic interposition arthroplasty
(E) Trapeziometacarpal arthrodesis

The correct response is Option E.

In this 39-year-old assembly line worker who has significant degenerative arthritis
of the trapeziometacarpal joint (ie, carpometacarpal joint of the thumb) of the
dominant hand, the most appropriate management is trapeziometacarpal
arthrodesis. Arthrodesis is recommended for younger patients who have arthritis
that is limited to the trapeziometacarpal joint and who require strong grip and
pinch. This procedure may increase stresses across the peritrapezial joints, leading
to pain, laxity, and subsequent arthritis. In addition, some range of motion of the
thumb is sacrificed, but this may improve over time. Other potential options
include partial trapeziectomy and interposition of the palmaris longus, which is a
new arthroscopic technique that has demonstrated promising results but is not yet
used widely.

Trapeziectomy, with or without soft-tissue interposition arthroplasty, is most


commonly used to treat trapeziometacarpal arthritis because it provides relief of
pain and increased mobility. However, this technique results in a decrease of pinch
and grip strength to approximately 75% of normal. Silastic arthroplasty is
performed only in low-demand patients who have adequate bone stock.

Reconstruction of the palmar oblique (beak) ligament is indicated for treatment of


prearthritic, painful instability of the trapeziometacarpal joint, and may prevent the
development of arthritis. However, this procedure will not alleviate pain once
significant arthritic changes have occurred.

References
1. Klimo GF, Verma RB, Baratz ME. The treatment of trapeziometacarpal arthritis
with arthrodesis. Hand Clin. 2001;17:261-270.
2. Tomaino MM. Treatment of Eaton stage I trapeziometacarpal disease: ligament
reconstruction or thumb metacarpal extension osteotomy? Hand Clin.
2001;17:197-205.
A 45-year-old roofer has pain and swelling of the right wrist after falling off a roof
and landing on his outstretched right hand. Radiographs are shown above. This
patient most likely has which of the following types of fracture-dislocation?

(A) Carpometacarpal joint-hamate


(B) Radial styloid
(C) Scaphocapitate
(D) Scaphoid-perilunate
(E) Scaphotrapeziotrapezoid

The correct response is Option D.

This patient has a scaphoid-perilunate fracture-dislocation, also known as a


scaphoid-dorsal perilunate fracture-dislocation. Wrist dislocations can range from
simple injuries to extended fractures associated with dislocation of one or more
carpal bones. This patient has sustained one of the most severe forms of wrist
dislocation, as shown in the radiographs.

Fracture-dislocations of the wrist are identified according to the carpal bone(s)


involved and the type of lunate dislocation (ie, incomplete or complete). Lunate
dislocations are subdivided into midcarpal lunocapitate, complete lunate, and
perilunate injuries. A complete lunate injury can be differentiated from a
perilunate injury by the presence of a concomitant dislocation of the radiolunate
joint. Greater arc injuries are typically caused by high-energy trauma. The radial
styloid, scaphoid, capitate, hamate, triquetrum, and/or ulnar styloid may be
fractured. Lesser arc dislocations involve injury to the scapholunate, lunocapitate,
and/or lunotriquetral intervals; these injuries are rated as type I to type IV
according to Mayfield’s classification.
The radiographs do not depict a carpometacarpal joint-hamate fracture-dislocation.
A fracture-dislocation of the radial styloid would occur more proximally. The
scaphocapitate and scaphotrapeziotrapezoid lie distal to the level of injury.
References
1. Szabo RM, Sutherland TB. Acute carpal fractures and dislocations. In: Peimer
CA, ed. Surgery of the Hand and Upper Extremity. New York, NY: McGraw Hill,
Inc; 1996:711-726.
2. Yaghoubian R, Goebel F, Musgrave DS, et al. Diagnosis and management of
acute fracture-dislocations of the carpus. Ortho Clin North Am. 2001;32:295-305.

Which of the following scaphoid fracture patterns illustrated above has the highest
incidence of avascular necrosis?

(A) A
(B) B
(C) C
(D) D
(E) E

The correct response is Option A.


Fractures of the scaphoid comprise as many as 60% of all carpal bone fractures;
most of these fractures are caused by a fall onto a dorsally flexed wrist. Scaphoid
fractures can be displaced or nondisplaced. In patients with nondisplaced fractures,
application of a cast followed by mobilization results in a rate of union as high as
95%. A long arm thumb spica cast is typically applied first, then replaced with a
short arm thumb spica cast.

In contrast, displaced fractures are often associated with an unacceptably high rate
of nonunion if treated conservatively and a high incidence of avascular necrosis.
This complication has been reported to occur in 13% to 40% of patients with
scaphoid fractures; its incidence is dependent on the presence or absence of
displacement and the anatomic location of the fracture. Because perforators to the
scaphoid enter distally and proceed proximally, fractures that occur more
proximally are more likely to interrupt the blood supply to the scaphoid.
The distal pole of the scaphoid has a good, protective blood supply; as a result, the
risk for avascular necrosis is low. In contrast, fractures of the scaphoid waist are
associated with an incidence of avascular necrosis of approximately 30%, and
fractures of the proximal pole of the scaphoid have a rate of avascular necrosis that
may be as high as 100%. Therefore, internal fixation is generally advocated for
treatment of fractures of the proximal pole of the scaphoid. With rigid fixation of
the bone, revascularization of the scaphoid occurs almost uniformly.

References
1. Gasser H. Delayed union and pseudoarthrosis of the carpal navicular: treatment
by compression screw osteosynthesis: a preliminary report on 20 fractures. J Bone
Joint Surg. 1965;47A:249.
2. Szabo RM, Sutherland TB. Acute carpal fractures and dislocations. In: Peimer
CA, ed. Surgery of the Hand and Upper Extremity. New York, NY: McGraw-Hill,
Inc; 1996.

Head Neck Congenital - 2004

A neonate has a reddish 1.5-cm mass of the nasal root with overlying cutaneous
telangiectasias. A photograph is shown above. On physical examination, the mass
is firm, noncompressible, and nonpulsatile. It does not transilluminate or change
with Valsalva maneuver. Which of the following is the most likely diagnosis?

(A) Dermoid cyst


(B) Encephalocele
(C) Glioma
(D) Hemangioma
(E) Lipoma

The correct response is Option C.

The findings in this neonate are consistent with a glioma. Nasal gliomas are
thought to originate as encephaloceles but fail to maintain their intracranial
connections. They may be external, internal, or a combination of both. External
gliomas typically appear at or just lateral to the nasal root. They are reddish, firm,
noncompressible, lobular lesions that exhibit telangiectasias of the overlying skin,
but do not transilluminate or pulsate. Bony defects, intracranial connections, and
cerebrospinal fluid leakage occur only rarely. Histologic evaluation shows
astrocytic neuroglial cells and fibrous and vascular connective tissue that is
covered with skin or nasal mucosa.

A nasal dermoid cyst arises from a dermoid sinus, which is a cutaneous inward
passage lined with stratified squamous epithelium. These masses can also be
external or internal. An external nasal dermoid is a firm, noncompressible,
nonpulsatile lesion that does not transilluminate and may be lobulated. Although
bony defects are infrequent, cerebrospinal fluid leakage and meningitis may occur.
Nasal dermoid cysts are derived from ectoderm and mesoderm, lined with
squamous epithelium, and contain specialized adnexal structures such as hair
follicles, pilosebaceous glands, and smooth muscles.

Encephaloceles involve herniation of cranial tissue through a skull defect. They


may be classified as meningoceles (containing meninges only),
meningoencephaloceles (containing meninges and brain), or
meningoencephalocystoceles (containing meninges, brain, and part of the
ventricular system). External, or sincipital, encephaloceles are soft, bluish,
compressible, pulsatile masses that are located at the nasal root and
transilluminate. They typically enlarge with crying and Valsalva maneuver.

Hemangiomas are raised lesions that arise from a proliferation of endothelial cells.
Most appear shortly after birth and involute spontaneously after a period of rapid
growth. Discoloration of the overlying skin is often associated.
Lipomas are soft, skin colored, compressible lesions that do not have cutaneous
telangiectasias and do not transilluminate or pulsate. They may appear at the nasal
root, but are not predisposed to that location.

References
1. Bartlett SP, Lin KY, Grossman R, et al. The surgical management of
orbitofacial dermoids in the pediatric patient. Plast Reconstr Surg. 1993;91:1208-
1215.
2. Pensler JM, Ivescu AS, Ciletti SJ, et al. Craniofacial gliomas. Plast Reconstr
Surg. 1996;98:27-30.

A 25-year-old woman is scheduled to undergo lip augmentation using dermal


allograft. Blockade of the infraorbital and mental nerves will be performed
bilaterally. In this patient, direct infiltration of local anesthetic is most likely to be
required at which of the following sites?

(A) Central upper vermillion


(B) Central lower vermillion
(C) Commissures
(D) Cupid’s bow
(E) Philtrum

The correct response is Option C.

Because blockade of the infraorbital and mental nerves alone does not ensure
adequate anesthesia of the oral commissures, direct infiltration of local anesthetic
into the commissures is likely to be necessary. Although a portion of the
sensibility of this region is supplied by contributions from the infraorbital and
mental nerves, the greatest contributor is the buccal nerve, which is derived from
the mandibular branch of the trigeminal nerve (V3). The buccal nerve supplies
sensation to a large area of skin of the cheek that lies just lateral to the commissure
and overlaps the area of the modiolus and buccinator muscle, as well as to a
portion of the buccal mucosa on the opposite side of the buccinator. Although the
infraorbital nerve may reach the upper half of the commissure and extend to 1.5
cm laterally, the border of this region follows an upward curve away from the
commissure.

Bilateral blockade of the infraorbital nerve in the midline provides complete


anesthesia to all central components of the upper lip, including the vermillion,
cupid’s bow, and philtrum. In addition, bilateral mental nerve blocks effectively
anesthetize the central section of the lower lip because the regions supplied by
these nerves meet in the midline.

References
1. Moore KL, Dalley II AF. Summary of cranial nerves. In: Moore KL, Dalley II
AF, eds. Clinically Oriented Anatomy. 4th ed. Philadelphia, Pa: Lippincott
Williams & Wilkins; 1999:1082-1096.
2. Williams PL, Warwick R, Dyson M, et al, eds. Gray’s Anatomy. 37th ed.
Edinburgh, Scotland: Churchill Livingstone, Inc; 1989:570-575, 1098-1107.
3. Zide BM, Swift R. How to block and tackle the face. Plast Reconstr Surg.
1998;101:840-851.

Which of the following nerves supplies sensory innervation to the buccal mucosa?

(A) Trigeminal (V) nerve


(B) Facial (VII) nerve
(C) Glossopharyngeal (IX) nerve
(D) Vagus (X) nerve
(E) Lingual nerve

The correct response is Option A.

The buccal branch of the trigeminal (V) nerve provides sensation to the buccal
mucosa. It is important for the surgeon to know the anatomy of this nerve branch
to plan and perform neurotized free flap reconstruction and reinnervation of the
intraoral cavity.

The buccal branch of the facial (VII) nerve innervates the muscles surrounding the
buccal mucosa.

The glossopharyngeal (IX) and vagus (X) nerves do not provide sensory
innervation to the intraoral mucosa.

The lingual nerve provides sensation to a portion of the tongue.

References
1. Lutz BS, Wei FC. Microsurgical reconstruction of the buccal mucosa. Clin Plast
Surg. 2001;28:339.
2. Marieb EN, ed. Overview of the digestive system. In: Human Anatomy and
Physiology. Redwood City, Ca: Benjamin/Cummings Publishing; 1995.

A 38-year-old woman sustains an injury to the auriculotemporal nerve during


superficial parotidectomy for removal of a mixed tumor. Which of the following is
the most likely postoperative finding?

(A) Numbness of the concha, helix, lobule, and temporal skin


(B) Numbness of the tragus, external auditory canal, and temporal skin
(C) Numbness of the entire pinna and paralysis of the temporalis muscle
(D) Paralysis of the auricularis anterior, superior, and posterior muscles
(E) Paralysis of the temporalis muscle
The correct response is Option B.

Injury to the auriculotemporal nerve is most likely to result in numbness of the


tragus, external auditory meatus, tympanum, and temporal skin. The
auriculotemporal nerve is a branch of the mandibular division of the trigeminal
nerve (V3). The fascicles of this nerve divide soon after the nerve originates to
allow for passage of the middle meningeal artery. The nerve then courses between
the sphenomandibular ligament and the neck of the condyle, emerges from behind
the temporomandibular joint, and travels toward the posterior surface of the upper
portion of the parotid gland, where it may be vulnerable to injury during
parotidectomy. It ascends with the superficial temporal vessels over the posterior
zygomatic arch and divides into three branches, which provide cutaneous sensory
innervation to the tragus, external acoustic meatus, tympanic membrane, and
temporal region. Minor branches of the auriculotemporal nerve convey
secretomotor fibers to the parotid gland and articular fibers to the
temporomandibular joint.

The concha and antihelix receive sensory innervation from the auricular branch of
the vagus (X) nerve. Sensory innervation to the helix and lobule is supplied by the
great auricular nerve and lesser occipital nerve, which are derived from C2-3.

The auricularis anterior, superior, and posterior muscles receive motor innervation
from the temporal and posterior auricular branches of the facial (V) nerve.

The temporalis muscle is innervated by the deep temporal nerves, which are
derived from the anterior, or motor, branch of the mandibular division of the
trigeminal nerve (V3).

References
1. Netter FH, ed. Atlas of Human Anatomy. 2nd ed. East Hanover, NJ:
Novartis/Hoechstetter Printing Co, Inc; 1997:18-21.
2. Pegington J. The side of the mouth and parapharynx. In: Pegington J, ed.
Clinical Anatomy In Action – The Head and Neck. Edinburgh, Scotland: Churchill
Livingstone, Inc; 1986;2:150-153.
3. Williams PL, Warwick R, Dyson M, et al, eds. Gray’s Anatomy. 37th ed.
Edinburgh, Scotland: Churchill Livingstone, Inc; 1989:1098-1107.
4. Wright T. Anatomy and development of the ear and hearing. In: Ludman H,
Wright T, eds. Diseases of the Ear. 6th ed. London, England: Arnold; 1998:8-13.

Which of the following structures passes though the foramen ovale?


(A) Accessory nerves
(B) Optic (II) nerve
(C) Ophthalmic division of the trigeminal nerve (V1)
(D) Maxillary division of the trigeminal nerve (V2)
(E) Mandibular division of the trigeminal nerve (V3)

The correct response is Option E.

The mandibular division of the trigeminal nerve (V3) passes through the foramen
ovale. This foramen is located in the region of the superior orbital fissure, which
contains the nerves to the extraocular muscles, sympathetic fibers, and vessels and
is found within the middle cranial fossa.

The accessory nerves, glossopharyngeal (IX) nerve, and vagus (X) nerve pass
through the foramen jugulare.

The optic foramen transmits the optic (II) nerve and ophthalmic artery.

The ophthalmic division of the trigeminal nerve (V1) passes through the superior
orbital fissure.

The foramen rotundum transmits the maxillary division of the trigeminal nerve
(V2).

References
1. Hochman M. Reconstruction of midfacial and anterior skull base defects.
Otolaryngol Clin North Am. 1955;28:1269-1277.
2. Langstein HN, Chang DW, Robb GL. Coverage of skull base defects. Clin Plast
Surg. 2001;28:375.

The external auditory meatus develops from which of the following embryologic
structures?

(A) First branchial arch


(B) Second branchial arch
(C) Third branchial arch
(D) First branchial groove
(E) Second branchial groove

The correct response is Option D.


Development of the six branchial arches occurs within the walls of the anterior
foregut during the fourth week of gestation, as neural crest cells migrate into the
future head and neck region and alternating ridges and depressions develop. Each
branchial arch is composed of endoderm, ectoderm, and mesoderm. During
development, a series of clefts forms to create the branchial grooves externally and
the pharyngeal pouches internally. The branchial grooves are lined with surface
ectoderm and the pharyngeal pouches are lined with foregut endoderm.

During the sixth week of gestation, six hillocks appear on the first (mandibular)
and second (hyoid) branchial arches, which give rise to the auricle. The first
branchial arch gives rise to the anterior (first through third) hillocks, and the
second branchial arch gives rise to the posterior (fourth through sixth) hillocks. By
the end of the eighth week of gestation, the auricle assumes its characteristic shape
following differential growth and fusion of the hillocks.

The external auditory meatus develops from the dorsal aspect of the first branchial
groove, which is a cleft between the first and second branchial arches.

The second, third, and fourth branchial grooves are obliterated within the cervical
sinus during the later stages of embryologic development. The cervical sinus
develops as a result of caudal overgrowth of the second branchial arch.

References
1. Gosain AK, Moore OF. Embryology of the head and neck. In: Aston SJ,
Beasley RW, Thorne CH, eds. Grabb & Smith’s Plastic Surgery. 5th ed.
Philadelphia, Pa: Lippincott-Raven; 1997:223.
2. Moore KL, ed. The branchial apparatus and the head and neck. The Developing
Human. 4th ed. Philadelphia, Pa: WB Saunders Co; 1988:170.

Which of the following cranial nerves provides innervation to the temporalis


muscle?

(A) Ophthalmic division of the trigeminal nerve (V1)


(B) Maxillary division of the trigeminal nerve (V2)
(C) Mandibular division of the trigeminal nerve (V3)
(D) Abducens (VI) nerve
(E) Facial (VII) nerve

The correct response is Option C.

The temporalis muscle receives its innervation primarily from the branches of the
mandibular division of the trigeminal nerve (V3), which then exits the skull via the
foramen ovale. The motor branches of the buccal, masseteric, and mandibular
nerves are derived from V3 and act to innervate the temporalis muscle. This
muscle is a large, powerful muscle of mastication that originates along the
temporal crest of the skull and inserts into the coronoid process of the mandible.

The ophthalmic division of the trigeminal nerve (V1) provides sensation to the
forehead and anterior scalp; this nerve branch exits the skull through the
supraorbital foramen.

The maxillary division of the trigeminal nerve (V2) provides sensation to the
cheek and upper lip and to the upper teeth via the superior alveolar nerve. This
nerve branch is transmitted through the infraorbital foramen.

The abducens (VI) nerve provides motor innervation into the lateral rectus muscle
of the eye.

The facial (VII) nerve provides motor innervation to the muscles of facial
expression.

References
1. Burggasser G, Happak W, Gruber H, et al. The temporalis: blood supply and
innervation. Plast Reconstr Surg. 2002;109:1862-1869.
2. Chen CT, Robinson JB Jr, Rohrich RJ, et al. The blood supply of the reverse
temporalis muscle flap: anatomic study and clinical applications. Plast Reconstr
Surg. 1999;103:1181-1188.

Which of the following neural structures does NOT pass through the superior
orbital fissure?

(A) Optic (II) nerve


(B) Oculomotor (III) nerve
(C) Trochlear (IV) nerve
(D) Abducens (VI) nerve
(E) Sympathetic nerve fibers

The correct response is Option A.

The superior orbital fissure transmits the oculomotor (III), trochlear (IV), and
abducens (VI) nerves and sympathetic nerve fibers from the cavernous plexus. In
patients who sustain high-velocity fractures of the orbital roof, the fractures may
extend to involve the structures of the superior orbital fissure, resulting in a
condition known as superior orbital fissure syndrome. This syndrome manifests as
loss of ocular motion resulting from paralysis of the motor nerves that pass
through the superior orbital fissure, but does not affect vision.
The optic (II) nerve and ophthalmic artery pass through the optic foramen, which
is separated from the superior orbital fissure by the lesser wing of the sphenoid
bone. Orbital apex syndrome, which involves injury to the optic nerve resulting
from extension of the fracture into the optic canal, is characterized by loss of
vision.

References
1. Clemente CD, ed. Gray's Anatomy of the Human Body. 30th ed. Philadelphia,
Pa: Lea & Febiger; 1985:161-172.
2. Moore KL, Dalley AF, eds. Clinically Oriented Anatomy. 4th ed. Philadelphia,
Pa: Lippincott, Williams & Wilkins; 1999:845, 899.

Hemiogiomas - 2004

In a patient who has retinal hemangiomas and cerebellar hemangioblastomas,


which of the following is the most likely diagnosis?

(A) Klippel-Trénaunay syndrome


(B) Parkes-Weber syndrome
(C) Rendu-Osler-Weber syndrome
(D) Sturge-Weber syndrome
(E) von Hippel-Lindau disease

The correct response is Option E.

Patients with von Hippel-Lindau disease have hemangiomas affecting the retina
and hemangioblastomas of the cerebellum and visceral organs. Seizures and
mental retardation may also be associated.

Klippel-Trénaunay syndrome is characterized by a port-wine stain (typically


involving one extremity) overlying venous and lymphatic malformations.

Parkes-Weber syndrome is similar to Klippel-Trénaunay syndrome but is


differentiated by the presence of arteriovenous fistulas.

Rendu-Osler-Weber syndrome, or hereditary hemorrhagic telangiectasia, is an


autosomal dominant disorder characterized by multiple ectatic vessels involving
the skin, mucous membranes, and visceral organs. Epistaxis, hematuria,
hematemesis, and melena are frequently associated.

Sturge-Weber syndrome is associated with a large vascular malformation, usually


a port-wine stain, in the distribution of the first and second branches of the
trigeminal nerve (V1 and V2). Other findings include focal motor seizures,
hemiparesis, and visual field defects, especially glaucoma. Mental retardation may
result from leptomeningeal venous malformations.

References
1. Boyd JB, Mulliken JB, Kaban LB, et al. Skeletal changes associated with
vascular malformations. Plast Reconstr Surg. 1984;74:789.
2. Burns AJ, Mulliken JB. Cutaneous vascular anomalies, hemangiomas, and
malformations. In: Georgiade GS, Riefkohl R, Levin LS, eds. Textbook of Plastic,
Maxillofacial and Reconstructive Surgery. 3rd ed. Baltimore, Md: Williams &
Wilkins; 1997:178-197.
3. Mulliken JB. Cutaneous vascular anomalies. In: McCarthy JG, ed. Plastic
Surgery. Philadelphia, Pa: WB Saunders Co; 1990;5:3191-3274.
4. Young AE. Venous and arterial malformations. In: Mulliken JB, Young AE,
eds. Vascular Birthmarks: Hemangiomas and Malformations. Philadelphia, Pa:
WB Saunders Co; 1988:196-214.

A 26-year-old man has had progressive enlargement of the left arm over the past
10 years. A photograph is shown above. Neurapraxia of the median and ulnar
nerves developed recently. The right arm decompresses when he lifts his hand
over his head.

Which of the following is the most likely diagnosis?

(A) Arteriovenous malformation


(B) Capillary malformation
(C) Hemangioma
(D) Lymphatic malformation
(E) Venous malformation

The correct response is Option E.

This patient most likely has a venous malformation, which is characterized by


compressibility and a propensity to fill with blood when the patient changes body
positions. It is usually larger and extends deeper than its superficial structure
indicates, and it frequently intertwines with neurovascular components, which may
affect the function of the involved extremity. When a venous malformation is
associated with thrombosis, pain may result.

Arteriovenous malformations are high-flow lesions characterized by rapid blood


flow through multiple feeding vessels. Pulsations and temperature changes in the
affected area are associated, but dermal involvement is minimal. A bruit or thrill is
heard on auscultation.

Capillary malformations, or port-wine stains, have abnormalities limited to the


capillaries in the dermis. These lesions are present at birth and do not regress
spontaneously. If left untreated, cobblestoning, ectasia, and progressive
darkening may occur. Patients with facial port-wine stains involving the first
(ophthalmic) or second (maxillary) divisions of the trigeminal nerve (V1 and V2)
are at significantly greater risk for ocular or nervous system involvement.
Hemangiomas typically appear shortly after birth and proliferate for several
months before spontaneously regressing. They are characterized by proliferative
primitive endothelial cell nests in syncytial masses, with and without vessel
lumens.

Lymphatic malformations can be superficial or deep. Superficial lymphatic


malformations are comprised of anomalous lymphatic remnants that manifest as
clear vesicles. Deep lymphatic malformations appear similar to hemangiomas but
are soft and compressible on palpation. Bony overgrowth is associated.

References
1. Mulliken JB. Cutaneous vascular anomalies. In: McCarthy JG, ed. Plastic
Surgery. Philadelphia, Pa: WB Saunders Co; 1990;5:3191-3274.
2. Mulliken JB, Glowacki J. Hemangiomas and vascular malformations in infants
and children: a classification based on endothelial characteristics. Plast Reconstr
Surg. 1982;69:412-422.

Which of the following vascular lesions is characterized by an arteriole central


vessel?

(A) Macular stain


(B) Port-wine stain
(C) Pyogenic granuloma
(D) Spider angioma
(E) Strawberry hemangioma

The correct response is Option D.


Spider angioma, or arteriolar malformation, is a small vascular malformation that
contains a central blanching arteriole vessel. Spider angiomas occur on the face in
both children and adults. Obliteration of the central feeding arteriole is the goal of
treatment; otherwise, the lesion will continue to enlarge. Laser therapy or
electrocautery is recommended.

Macular stains are typically seen on the skin of neonates and are not true vascular
nevi.

Port-wine stains are capillary vascular malformations of the face that are seen at
birth and grow commensurately with the child.

Pyogenic granulomas are reparative vascular lesions commonly seen on the head
and neck in children. Because these lesions receive their blood supply primarily
from capillaries, they are not true vascular malformations.

A strawberry hemangioma is a venous malformation that is typically seen at birth


and enlarges during the first few months of life. These lesions do not grow
commensurately with the child.

References
1. Achauer BM, Vander Kam VM. Vascular lesions. Clin Plast Surg. 1993;20:43-
51.
2. Mulliken JB, Glowacki J. Hemangiomas and vascular malformations in infants
and children: a classification based on endothelial characteristics. Plast Reconstr
Surg. 1982;69:412-420.

Histopathologic examination of a proliferating hemangioma is most likely to show


each of the following characteristics EXCEPT

(A) decreased 17-beta estradiol binding sites


(B) increased collagenase activity
(C) increased quantity of mast cells
(D) multilaminate basement membrane
(E) plump endothelial cells

The correct response is Option A.

Hemangiomas are true benign neoplasms that have plump, rapidly dividing
endothelial cells and an increased quantity of mast cells on histopathologic
examination. These mast cells produce heparin, which is believed to be the
primarily stimulus for migration of capillary endothelial cells. The number of mast
cells begins to decrease as the hemangioma involutes.
During the proliferation phase of a hemangioma, the levels of circulating 17-beta
estradiol are increased. In addition, the number of binding sites for 17-beta
estradiol is actually increased, not decreased. Collagenase activity is increased in
all enlarging hemangiomas as the endothelial cells destroy the basement
membrane to allow for the formation of new capillary tubules. A multilaminate
basement membrane is also observed.

In contrast, vascular malformations exhibit flattening of the endothelium, thinning


of the basement membrane, and a normal quantity of mast cells.
References
1. Ausprunk DH, Folkman J. Migration and proliferation of endothelial cells in
preformed and newly formed blood vessels during tumor angiogenesis. Microvasc
Res. 1977;14:53.
2. Mulliken JB, Glowacki J. Hemangiomas and vascular malformations in infants
& children: a classification based on endothelial characteristics. Plast Reconstr
Surg. 1982;69:412.
3. Sasaki GH, Pang CY, Wittliff JL. Pathogenesis and treatment of infant skin
strawberry hemangiomas: clinical and in vitro studies of hormonal effects. Plast
Reconstr Surg. 1984;73:359.
4. Stal GH, Hamilton S, Spira M. Hemangiomas, lymphangiomas, and vascular
malformations of the head and neck. Otolaryngol Clin North Am. 1986;19:769.

A 38-year-old man has a pulsatile mass on the volar aspect of the distoradial
forearm three weeks after sustaining a stab wound to the forearm. At the time of
injury, he controlled the bleeding successfully by applying pressure to the wound
and did not seek treatment. Which of the following is the most likely diagnosis?

(A) Aneurysm
(B) Arteriovenous fistula
(C) Pseudoaneurysm
(D) Ulnar artery thrombosis
(E) Vascular malformation

The correct response is Option C.

The most likely diagnosis is pseudoaneurysm, which has occurred secondary to


trauma in the area of the radial artery. This patient sustained a stab wound to the
forearm that immediately began to bleed. Although he controlled the bleeding at
the time of injury, a pulsatile mass developed subsequently. Because obtaining a
biopsy specimen can result in significant hemorrhage in these patients, it is
important to carefully review preceding history, which is consistent with
pseudoaneurysm in this situation.
This patient’s history is not consistent with a true aneurysm, and examination
findings rule out an arteriovenous fistula.

Ulnar artery thrombosis is also known as hypothenar hammer syndrome because it


occurs secondary to localized trauma in construction workers or other persons who
strike objects with their hands. Typical symptoms include pain, paresthesia,
decreased temperature, color changes, pallor, and numbness of the ulnarmost
digits.

Vascular malformations are usually present at birth, but may develop and enlarge
over time. A thrill or bruit is frequently associated, and color changes may occur.

References
1. Angelides AC. Ganglions of the hand and wrist. In: Green DP, Hotchkiss RN,
Pederson WC, eds. Operative Hand Surgery. 4th ed. New York, NY: Churchill
Livingstone, Inc; 1999;2:2171-2183.
2. Koman LA, Ruch DS, Smith BP, et al. Vascular disorders. In: Green DP,
Hotchkiss RN, Pederson WC, eds. Operative Hand Surgery. 4th ed. New York,
NY: Churchill Livingstone, Inc; 1999;2:2254-2302.

A 4-year-old boy has a 4 ( 8-cm compressible mass of the soft tissue of the volar
forearm that has enlarged over the past year. He also has worsening pain in the
forearm. Physical examination shows intact skin over the forearm; a thrill can be
palpated over the mass. The mass does not decompress fully with elevation of the
arm.

Which of the following is the most appropriate diagnostic study?

(A) Plain radiographs


(B) Bone scan
(C) Contrast-enhanced CT scan
(D) MR angiography
(E) Contrast angiography

The correct response is Option D.

In this 4-year-old boy who has a high-flow vascular malformation of the volar
forearm, the most appropriate diagnostic study is magnetic resonance imaging
with intravenous gadolinium, also known as magnetic resonance angiography
(MRA). Noninvasive imaging studies are most appropriate in young patients with
suspected vascular malformations. MRA delineates baseline tissue involvement
and is used to distinguish between low-flow and high-flow lesions.
Plain radiographs are appropriate for detecting skeletal alterations resulting from
vascular malformations, but would not be diagnostic. Duplex ultrasonography is a
simple noninvasive technique that can define flow characteristics, but is
cumbersome in young patients and cannot be used to accurately assess the degree
of involvement of other soft-tissue structures. Contrast-enhanced CT scans do not
provide the necessary resolution for evaluation of vascular malformations.
Because contrast angiography is invasive, it is reserved for more extensive
evaluation of high-flow malformations, preoperative planning, and/or
superselective embolization.

References
1. Holder LE, Merine DS, Yang A. Nuclear medicine, contrast angiography, and
magnetic resonance imaging for evaluating vascular problems in the hand. Hand
Clin. 1993;9:85-113.
2. Upton J, Coombs CJ, Mulliken JB, et al. Vascular malformations of the upper
limb: a review of 270 patients. J Hand Surg. 1999;24A:1019-1035.

Laser

Which of the following treatment systems activates the topical formulation of 5-


aminolevulinic acid?

(A) Erbium laser


(B) Phenol
(C) Pulsed-dye laser
(D) Radiofrequency
(E) Tretinoin

The correct response is Option C.

5-Aminolevulinic acid (ALA) is a topical medication that, when placed on the


skin, penetrates the altered epithelial epidermis and is absorbed into the
keratinocytes, whereby it is converted enzymatically into protoporphyrin IX. The
application of light results in the release of cytotoxic radicals, which render this
drug useful for the treatment of cutaneous lesions. Protoporphyrin IX has a
maximum absorption at 410, 630, and 690 nm. Because of this, blue light systems
(400B450 nm), pulsed-dye systems (585B595 nm), and photo rejuvenation
systems (560B1200 nm) all may activate the drug. Recently, it has been used for
the treatment of acne vulgaris as well as aging. It requires up to one hour of
application time before initiation of therapy and may result in several days of
swelling, erythema, and exfoliation. Patients also tend to be photosensitive for up
to 48 hours after treatment. Radiofrequency and erbium (2940-nm) laser systems
are not within the visible spectrum and will not activate protoporphyrin IX.
Topical agents such as phenol and tretinoin will not activate this drug.

References:
1. Ruiz-Rodriquez R, Sanz-Sanchez T, Cordoba S. Photodynamic
photorejuvenation. Dermatol Surg. 2002;28(8):742-744.
2. Goldman MP, Boyce SM. A single-center study of aminolevulinic acid and 417
nm photodynamic therapy in the treatment of moderate to severe acne vulgaris. J
Drugs Dermatol. 2003;2(4):393-396.
3. Alexiades-Armenakas MR, Geronemus R. Laser-mediated photodynamic
therapy of actinic keratoses. Arch Dermatol. 2003;139:1313-1320.

A 32-year-old woman comes to the office for consultation regarding removal of a


black and red tattoo from the left breast. The most effective intervention for
removal of this patient’s tattoo is treatment with which of the following types of
lasers?

(A) Carbon dioxide


(B) Pulsed-dye
(C) Q-switched alexandrite
(D) Q-switched Nd:YAG
(E) Q-switched ruby

The correct response is Option D.

The current standard of care for removing most unwanted tattoos is use of specific
lasers to target the wavelength of the color pigments. No one laser can selectively
target each pigment type in the spectrum of colors used by professional tattoo
artists.

A Q-switched laser delivers short pulses, which reduces damage to surrounding


normal skin. A frequency-doubled Q-switched Nd:YAG laser has a crystal that
doubles the frequency of the Nd:YAG laser from 1064 nm to 532 nm. In the 532-
nm (green) wavelength, the laser removes red pigments effectively. When
switched to the full 1064-nm (red) wavelength, it effectively removes black
pigments. The mechanism of action is believed to be selective fragmentation of the
targeted pigment followed by phagocytosis and lymphatic clearing.

A carbon dioxide laser removes tattoos by targeting water in the skin. Therefore, it
nonselectively destroys tissue, including pigment-bearing cells. Because it poses a
high risk of hypopigmentation and hypertrophic scarring, it is not the laser of
choice for removing professional tattoos.

A pulsed-dye laser has a wavelength of 510 nm; it can remove red pigments but is
a poor choice for black pigments. A Q-switched alexandrite laser, with a
wavelength of 755 nm, is excellent at removing black and green pigments but not
reds. A Q-switched ruby laser, which has a wavelength of 694 nm, also is
excellent for removing black pigments but is poor at targeting reds. In addition, it
may produce more damage to surrounding tissue than the alexandrite laser.

References:
1. Alster TA. Laser treatment of tattoos. In: Alster TA, ed. Manual of Cutaneous
Laser Techniques. 2nd ed. Philadelphia: Lippincott Williams & Wilkins; 2000:71-
88.
2. Adrian RM, Griffin L. Laser tattoo removal. Clin Plast Surg. 2000;27:181-192.

An 18-year-old woman has a large arteriovenous malformation on the face that has
ulcerated and bled vigorously several times. Which of the following is the most
appropriate treatment option?

(A) Intralesional administration of interferon


(B) Distal embolization under superselective angiographic control
(C) Ligation of the feeding vessels
(D) Intralesional excision to minimize tissue loss
(E) Aggressive resection with immediate flap reconstruction

The correct response is Option E.

This 18-year-old woman has a large arteriovenous malformation (AVM) on the


face characterized by ulceration and bleeding. The most appropriate management
is aggressive resection to eliminate the lesion immediately. In addition, because
aggressive resection of an AVM typically exposes vital structures and leaves a
cosmetically disfiguring defect, immediate reconstruction with a flap is indicated.
Arteriography, Doppler ultrasonography, and/or MRI should be obtained
preoperatively to delineate the extent of the lesion.

Distal embolization of the lesion can be performed under angiographic control


before excision to limit blood loss but is not an effective treatment in itself.

Ligation of the feeding vessels without subsequent resection will further worsen
the AVM because it will result in the development of new collateral vessels.
Intralesional excision is associated with a high risk for recurrence.

Intralesional interferon has not been shown to provide benefit in the treatment of
arteriovenous malformations.

References:
1. Burrows PE. Urgent and emergent embolization of lesions of the head and neck
in children: indications and results. Pediatrics. 1987;80(3):386-394.
2. Han MH, Seong SO, Kim HD, et al. Craniofacial arteriovenous malformations:
preoperative embolization with direct puncture and injection of n-butyl
cyanoacrylate. Radiology. 1999;211:661.
3. Kohout MP, Hanson M, Pribaz JJ, et al. Arteriovenous malformations of the
head and neck: natural history and management. Plast Reconstr Surg.
1998;102:643.
4. Persky MS. Congenital vascular lesions of the head and neck. Laryngoscope.
1986;96:1002.

Which of the following laser wavelengths is ideal for treating the lesion shown
above?
(A) 532 nm
(B) 585 nm
(C) 788 nm
(D) 810 nm
(E) 2940 nm

The correct response is Option B.

The patient depicted in the photograph has a port-wine stain in the VBIII
distribution of the trigeminal (V) nerve. These lesions may occur anywhere on the
body but are most commonly seen on the face. They occur unilaterally in 85% of
patients and involve more than one dermatome in almost 70% of patients. These
lesions are more commonly seen in women than in men (3:1) and may be
hereditary (25%). The natural progression of these lesions with age includes
darkening of the lesion due to the presence of deoxyhemoglobin, with thickening
of the dermis and a cobblestoning appearance. It should be noted that as the
lesions get darker, they are more difficult to treat. Because of its depth,
penetration, and specificity for vascular targets, the 585-nm laser is the best
treatment choice for these lesions. Treatment with the pulsed-dye laser typically
results in less epidermal injury and risks for scarring. It should be used with
caution in patients with pigment.

The 532-nm laser (KTP) is useful for superficial vascular telangiectasis. Both the
788- and 810-nm lasers are useful for pigmentation as well as hair removal.
Finally, the 2940-nm laser (erbium) is a resurfacing device and is not specific for
vascular targets.

References:
1. Tallman B, Tan OT, Morelli JG, et al. Location of port-wine stains and the
likelihood of ophthalmic and/or central nervous complications. Pediatrics.
1991;87(3):323-327.
2. Mills CM, Lanigan SW, Hughes J, et al. Demographic study of port wine stain
patients attending a laser clinic: family history, prevalence of naevus anaemicus
and results of prior treatment. Clin Esp Dermatol. 1997;22(4):166.
3. Goldman MP, Fitzpatrick RE, Ruiz-Esparza J. Treatment of port-wine stains
(capillary malformation) with the flashlamp pumped pulsed dye laser. J Pediatr.
1993;122:71.

Which of the following light sources is noncoherent?


(A) Alexandrite
(B) Diode
(C) Intense pulsed
(D) Pulsed-dye
(E) YAG

The correct response is Option C.

Intense pulsed (broadband) light is a high-intensity light source that emits


polychromatic energy. It supplies noncoherent light over a broad spectrum of
wavelengths from 510 to 1200 nm. This broad spectrum allows variability
regarding target selection and skin types. Intense pulsed-light systems are
commonly used to treat hyperpigmentation, telangiectasis, rosacea, excessive or
unwanted hair, rhytids, and vascular malformations. When used for photoaging,
they have been shown to produce long-term positive results on the face, neck, and
chest. They have also demonstrated improvement in telangiectasis and
pigmentation. Adverse effects of intense pulsed light include crusting, erythema,
and purpura.
Alexandrite, diode, pulsed-dye, and YAG laser systems emit monochromatic
coherent light. Each device has a specific wavelength and chromophore.

References:
1. Raulin C, Greve B, Grema H. IPL technology: a review. Lasers Surg Med.
2003;32(2):78-87.
2. Weiss RA, Weiss MA, Beasley KL. Rejuvenation of photoaged skin: 5 year
results with intense pulsed light of the face, neck, and chest. Dermatol Surg.
2002;28(12):1115-1119.

Local Anesethetics

A 46-year-old woman undergoes suction lipectomy of the abdomen using


tumescent anesthesia containing lidocaine. Despite doses of lidocaine higher than
recommended for local infiltration, toxicity to the drug does not develop in this
patient. Which of the following is the most likely explanation?

(A) Chemical structure of lidocaine


(B) pH of the tissue
(C) Rate of lidocaine absorption
(D) Removal of lidocaine by suction
(E) Toxic threshold for plasma lidocaine concentration

The correct response is Option C.

Lidocaine for local infiltration is available in 0.5%, 1%, 1.5%, and 2% solutions
that contain epinephrine at a concentration of 1:100,000 or 1:200,000. In contrast,
lidocaine for tumescent anesthesia commonly is available in a 0.05% solution that
contains epinephrine at a concentration of 1:1,000,000. This dilute solution results
in a very slow rate of lidocaine absorption from subcutaneous tissue, which
prevents high peak plasma concentrations of the drug as well as toxicity. Although
a standard 1% solution yields a maximum plasma concentration of lidocaine in 1
hour, a tumescent solution provides a maximal plasma concentration in 8 to 12
hours.

The chemical structure of lidocaine can vary with the pH of the tissue. However,
this is not a primary factor in the pharmacokinetics of tumescent lidocaine. Suction
lipectomy removes some lidocaine, which reduces the peak plasma concentration
by approximately 25% (compared with a nonBsuction-lipectomy control).
However, this is not the primary mechanism that allows a fivefold increase in dose
without toxicity. In human studies of the toxic threshold for plasma lidocaine,
central nervous system symptoms occurred at concentrations above 4 _g/mL; this
was equally true for standard and tumescent solutions.

References:
1. Scott DB, Jebson PJR, Braid P, et al. Factors affecting plasma levels of
lignocaine and prilocaine. Br J Anaesth. 1972;44:1040-1048.
2. Klein JA. Tumescent technique for regional anesthesia permits lidocaine doses
of 35 mg/kg for liposuction. J Dermatol Surg Oncol. 1990;16:248-263.
3. Coleman WP III. Tumescent anesthesia with a lidocaine dose of 55 mg/kg is
safe for liposuction. Dermatol Surg. 1996;22:919.

A 27-year-old woman has a true allergy to a local anesthetic agent used in the past.
Use of which of the following local anesthetic agents is most appropriate for this
patient?

(A) Benzocaine
(B) Chloroprocaine
(C) Lidocaine
(D) Procaine
(E) Tetracaine

The correct response is Option C.

True allergic reactions to local anesthetics, although rare, can occur with the ester-
linked agents chloroprocaine, tetracaine, procaine, and benzocaine. The
degradation product of ester-linked anesthetics is para-aminobenzoic acid
(PABA), which is an antigenic substance. The amide-type anesthetics lidocaine,
bupivacaine, mepivacaine, and prilocaine do not cause true allergic reactions.
Allergic reactions to amide-type anesthetics may occur but are usually related to
the preservative methylparaben, which is structurally similar to PABA.
References:
1. Strombergh BV. Anesthesia. In: McCarthy JG, May JW, Littler JW, eds. Plastic
Surgery. Vol 1. Philadelphia: WB Saunders; 1990:143-144.
2. Rizzuti RP, McArthur A, Riefkohl R. Commonly used drugs and their
interactions. In: Georgiade GS, Riefkohl R, Levin LS, eds. Georgiade Plastic,
Maxillofacial, and Reconstructive Surgery. 3rd ed. Baltimore: Williams &
Wilkins; 1997:1327-1330.
3. Dentz ME, Grichnick KP, Silbert KS, et al. Anesthesia and postoperative
analgesia. In: Sabiston DC, Lyerly HK, eds. Textbook of Surgery: The Biological
Basis of Modern Surgical Practice. 15th ed. Philadelphia: WB Saunders;
1997:186-206.

Which of the following is LEAST sensitive to increasing plasma levels of


lidocaine?

(A) Blood pressure


(B) Central nervous system activity
(C) Heart rate
(D) Muscle tone

The correct response is Option A.

Blood pressure is typically insensitive to increasing plasma levels of lidocaine and


other local anesthetics because a compensatory increase in systemic vascular
resistance prevents the blood pressure from increasing.

Adverse reactions in the central nervous system are much more common and are
biphasic. Initially, an excitatory phase occurs, which may be due to inhibition of
the amygdala. This phase may produce muscle twitching in the face and
extremities followed by tremors that can progress to seizures. As the amount of
local anesthetic increases, a depressive phase occurs and is characterized by
drowsiness, unconsciousness, and respiratory arrest.

The cardiovascular system is thought to be more resistant than the central nervous
system to the effects of local anesthetics. However, it can sustain dangerous
reactions, usually at higher plasma levels. With toxic doses of local anesthetics,
cardiovascular reactions may include arrhythmias, cardiovascular depression, and
shock. Cardiovascular depression tends to be serious and difficult to treat. The
more lipid-soluble local anesthetics, such as bupivacaine, tend to have a higher
toxicity than the less lipid-soluble drugs, such as lidocaine.

Toxic effects of local anesthetics result from inappropriately high dosage or


unintentional intravascular injection. Management of lidocaine toxicity consists of
ECG monitoring and oxygen administration. If seizures occur, they are typically
controlled with diazepam or midazolam. If mechanical ventilation is required,
paralytic agents may be administered. Because of the seriousness of the toxic
effects, appropriate monitoring and personnel trained in advanced cardiac life
support are required during local anesthetic use, regardless of the magnitude of the
procedure.

References:
1. Matarasso A. Lidocaine in ultrasound-assisted lipoplasty. Clin Plast Surg.
1999;26(3):431-439.
2. Trott SA, Beran SJ, Rohrich RJ, et al. Safety considerations and fluid
resuscitation in liposuction: an analysis of 53 consecutive patients. Plast Reconstr
Surg. 1998;102(6):2220-2229.
3. Vaughan TM, Burt J. Local anesthetics. Selected Readings in Plastic Surgery.
1999;9(4).
4. Cousins MJ, Bridenbaugh PO. Neural Blockade in Clinical Anesthesia and
Management of Pain. 2nd ed. Philadelphia: LB Lippincott; 1988.
5. Mehra P, Caiazzo A, Maloney P. Lidocaine toxicity. Anesth Prog. 1999;45:38.
6 [No authors listed]: Cardiotoxicity of local anesthetic drugs [Editorial]. Lancet.
1986;2(8517):1192-1194.

Lower Extremity

Which of the following anatomic structures is an important landmark in raising a


reverse sural artery flap?

(A) Achilles tendon


(B) Deep peroneal nerve
(C) Lesser saphenous vein
(D) Plantaris tendon
(E) Posterior tibial artery

The correct response is Option C.

When raising a reverse sural artery flap, the important landmarks are the lesser
saphenous vein and sural nerve, which should bisect the cutaneous paddle. The
blood supply to this flap depends on the medial superficial sural artery and the
lesser saphenous vein with its two small accompanying arteries. The pivot point of
the pedicle is typically 5 cm above the lateral malleolus, where the perforators of
the flap enter a more superficial plane.

The Achilles tendons are not landmarks for raising this flap but, when exposed, are
good indications for this type of flap. The deep peroneal nerve is located in the
lateral compartment. The posterior tibial artery is found in the deep compartment
and is not associated with this flap. The plantaris tendon is deep to the dissection
of the flap.

References:
1. Hollier L, Sharma S, Babigumira E, et al. Versatility of the sural
fasciocutaneous flap in the coverage of lower extremity wounds. Plast Reconstr
Surg. 2002;110(7):1673.
2. Ayyappan T, Chadha A. Super sural neurofasciocutaneous flaps in acute
traumatic heel reconstructions. Plast Reconstr Surg. 2002;109(7):2307.

A 53-year-old man with a comminuted fracture of the midtibia has a 4 H 3-cm


defect of the midanterior surface of the leg at the level of the fracture. He currently
smokes two packs of cigarettes daily. Physical examination shows no palpable
dorsalis pedis pulse. Which of the following surgical interventions is the most
appropriate method of reconstruction in this patient?

(A) Anterior tibialis muscle flap


(B) Below-knee amputation
(C) Gastrocnemius muscle flap
(D) Gracilis free tissue transfer
(E) Soleus muscle flap

The correct response is Option E.

The soleus muscle flap is most appropriate for reconstruction in this patient. The
soleus is a bipenniform muscle; its medial head originates from the posterior tibia,
and the lateral head originates from the proximal fibula. It is located deep to the
gastrocnemius in the superficial posterior compartment. Blood to the medial head
is predominantly supplied by the popliteal and posterior tibial arteries and the
lateral head is predominantly supplied by the peroneal artery. Depending on the
size of the defect, a hemisoleus muscle flap can be used to preserve flexor
function.

Below-knee amputation is an option if salvage of the leg is not possible or if the


extremity is insensate, particularly in older patients.
An anterior tibialis muscle flap can be used for small defects. In this patient with
an absent dorsalis pedis pulse and possible injury to the anterior tibial artery, this
is not an optimum choice.

For lower-extremity reconstruction, the gastrocnemius muscle flap is used for knee
wounds and proximal tibial defects, the soleus for middle third defects, and free
tissue transfer for distal third defects. The gastrocnemius muscle flap might not
reach the defect in the middle third and therefore is not the best option. Free tissue
transfer is often used for reconstruction of high-velocity injuries to avoid the use
of muscle in the zone of injury. Free tissue transfer, however, is not the best option
for this 53-year-old man because his history of cigarette smoking and absent pedal
pulse suggest the possibility of peripheral vascular disease.

References:
1. Nahai F, Love TR. Lower extremity reconstruction: management of soft tissue
defects. In: Cohen M, ed. Mastery of Plastic and Reconstructive Surgery. Vol 3.
Philadelphia: Lippincott Williams & Wilkins; 1994:1773-1799.
2. Kasabian AK, Karp NS. Lower extremity reconstruction. In: Aston SJ, Beasley
RW, Thorne CH, eds. Grabb & Smith’s Plastic Surgery. 5th ed. Philadelphia:
Lippincott Williams & Wilkins; 1997:1031-1047.

A 53-year-old man with diabetes mellitus has a nonhealing wound over the right
calcaneus. A bone scan shows increased uptake at the site of the wound. Which of
the following is the most definitive diagnostic test?

(A) Bone biopsy and culture


(B) CT scan
(C) Indium scan
(D) MRI
(E) Wound biopsy and culture

The correct response is Option A.

This 53-year-old man with a nonhealing wound has most likely developed
osteomyelitis of the foot, which may be caused by direct, penetrating trauma to the
bone or contiguous spread from adjacent soft tissue. The calcaneus is the most
common site of involvement in the foot, followed by the metatarsals and the
cuboid bone. Although osteomyelitis may be suspected clinically, definitive
diagnosis is accomplished with bone biopsy and culture. This can be obtained by
needle aspiration of the interosseous or subperiosteal space.
Cultures of a draining wound are less accurate diagnostically than bone biopsy and
culture. Findings on bone scan are nonspecific and are often positive in patients
with local wound infection or cellulitis. CT scans are not used in the diagnosis of
osteomyelitis. Although MRI is more accurate than bone scan, it cannot be used to
establish a definitive diagnosis.

References:
1. Johnson JE, Hall RL. Management of foot infections. In: Gould JS, ed.
Operative Foot Surgery. Philadelphia: WB Saunders; 1994.
2. Resnick D, Niwayama G, eds. Diagnosis of Bone and Joint Disorders.
Philadelphia: WB Saunders; 1988.

A 27-year-old man develops osteomyelitis after sustaining an open fracture of the


distal tibia and fibula in a motorcycle accident. Following bony debridement, there
is a 9-cm segmental loss of the distal tibia. An external fixator is applied, and a
muscle flap is to be used to cover the skin defect. Which of the following is the
most appropriate management of the bony defect?

(A) Free fibula flap


(B) Iliac crest bone grafting
(C) Papineau bone grafting
(D) Tibiofibular synostosis
(E) Ilizarov bone lengthening

The correct response is Option A.

The most appropriate management of this patient’s bone defect is a free fibula
surrounding flap. Management typically depends on the volume of the bony defect
along with soft-tissue stability and vascularity. If the tissue is unstable and poorly
vascularized, a regional flap or free flap is required. For most bone defects smaller
than 6 cm, traditional tricortical iliac crest bone grafts can be placed beneath the
muscle flap. In patients with larger defects, a vascularized bone flap, such as the
free fibula flap, is required. Vascularized bone also minimizes the risk for
nonunion. However, weight-bearing ambulation should not occur for many months
after flap coverage to allow for healing of bone without infection.

Other sources of free vascularized bone include the iliac crest and scapula.
However, the iliac crest has both inadequate length and excessive curvature, and
the scapula has inadequate length and lacks the tubular depth and width of the
fibula.

Papineau grafts are cancellous grafts that are used to fill the bone defect but are
packed flush with the skin. These grafts are seldom used clinically and are
appropriate only for defects smaller than 6 cm. Tibiofibular synostosis is not a
reasonable option in a patient with osteomyelitis of the distal tibia who has
compromised stability and vascularity of the soft tissue. The Ilizarov technique for
bone transport is less optimal in a patient who requires a long bone segment and
has soft tissue of poor quality.

References:
1. Anthony JP, Mathes SJ. Update on chronic osteomyelitis. Clin Plast Surg.
1991;18:515-523.
2. Nahai F, Love TR. Lower extremity reconstruction: management of soft tissue
defects. In: Cohen M, ed. Mastery of Plastic and Reconstructive Surgery. Vol 3.
Philadelphia: Lippincott Williams & Wilkins; 1994:1773-1799.

Lymphedema

A 6-year-old girl has had swelling of the right cheek for the past five years. Her
mother says that the area of swelling has grown commensurately with the child
and enlarges when the child has a sore throat. Physical examination shows a 4-cm
mass that is soft and doughy on palpation. Which of the following is the most
likely diagnosis?

(A) Arteriovenous malformation


(B) Branchial cleft cyst
(C) Capillary malformation
(D) Lymphatic malformation
(E) Venous malformation

The correct response is Option D.

The most likely diagnosis is lymphatic malformation, a localized malformation in


the development of the lymphatic system. Approximately 50% of lymphangiomas
are diagnosed at birth, and 90% are diagnosed before age 2 years. Lymphangiomas
are soft and doughy to palpation and typically occur in the head and neck. They
may vary in size from a few millimeters to larger than 30 cm. Although the
overlying skin is usually normal, it may be thin, atrophic, or have bluish
discoloration.

Patient symptoms correlate with the size, location, and extent of the
lymphangioma. Respiratory compromise is the most common significant
complication associated with lymphangiomas affecting the head and neck. Viral
infections can cause enlargement of the lesion, as in this patient.

Arteriovenous malformations are high-pressure, high-flow lesions that grow in


rapid spurts. These lesions are often pulsatile and warmer than the surrounding
skin. A port-wine-stainBlike appearance of the skin is commonly associated.
Branchial cleft cysts present as smooth, nontender masses overlying the anterior
border of the sternocleidomastoid. They can fluctuate in size. Capillary
malformations, or port-wine stains, are red macular lesions that are present at birth
and do not regress. They can be localized or extensive. In patients with capillary
malformations of the face, further darkening of skin and hyperplastic skin changes
occur over time.

Hemangiomas are vascular anomalies that appear shortly after birth, proliferate
rapidly during infancy, and then spontaneously regress during childhood. They do
not occur in adults.

References:
1. Achauer BM. Cutaneous vascular lesions and lasers. In: Bentz ML, ed. Pediatric
Plastic Surgery. Stamford, CT: Appleton & Lange; 1998:557-570.
2. Giguere CM, Bauman NM, Smith RJ. New treatment options for lymphangioma
in infants and children. Ann Otol Rhinol Laryngol. 2002;111:1066-1075.
3. Mulliken JB. Vascular anomalies. In: Aston SJ, Beasley RW, Thorne CH, eds.
Grabb & Smith’s Plastic Surgery. 5th ed. Philadelphia: Lippincott Williams &
Wilkins; 1997:191-203.

Ma-facial - 2004
A 35-year-old man has swelling and tenderness of the nose and deviation of the
nose to the left after being accidentally struck in the face while playing squash.
Intranasal examination shows a localized purple mass on the left side of the
septum. A CT scan is shown on page 4. Which of the following is the most
appropriate initial management?

(A) Immediate closed reduction and placement of an external splint


(B) Immediate open reduction of the nasal fractures
(C) Immediate incision and drainage of the septal mass
(D) Immediate submucous resection of the septal fracture

The correct response is Option C.

The most appropriate initial step in the management of this patient is immediate
incision and drainage of the septal hematoma. If left untreated, septal hematomas
cause fibrosis and narrowing of the nasal passages, distortion of the septum, and/or
formation of an abscess. They can also cause pressure necrosis of the septum,
leading to septal perforation and eventually to complete necrosis with formation of
a saddle-nose deformity.

Fracture management is undertaken after the hematoma has been evacuated. In


patients who have significant swelling obscuring the nasal structure, fracture
reduction is delayed until the swelling resolves, typically within five to 10 days. If
swelling is minimal, then reduction can be performed immediately after
evacuation.

Although closed reduction is appropriate in most patients with nasal fractures,


open reduction may be necessary to obtain anatomic reduction. If lacerations are
present, they may be used as incisions for surgical reduction. However, it should
be noted that open techniques have been associated with a small incidence of
necrosis of the traumatized nasal mucosa. In addition, unsatisfactory results
following surgery are likely to lead to severe injury of the nasal septum that may
be difficult to correct. Formal rhinoplasty may be performed at a later date to
resolve breathing difficulties or improve the aesthetic appearance of the nose.

References
1. Manson PN. Facial fractures. In: Aston SJ, Beasley RW, Thorne CH, eds.
Grabb & Smith’s Plastic Surgery. 5th ed. Philadelphia, Pa: Lippincott-Raven;
1997:383-412.
2. Rohrich RJ, Adams WP. Nasal fracture management: minimizing secondary
nasal deformities. Plast Reconstr Surg. 2000;106:266-273.

A 25-year-old woman sustains a frontal sinus fracture in a motor vehicle collision.


On physical examination, there is cerebrospinal fluid rhinorrhea. A CT scan of the
head shows displacement of both the anterior and posterior walls and fracture lines
extending through the nasofrontal ducts. Which of the following is the most
appropriate management?

(A) Ablation
(B) Cranialization
(C) Exenteration
(D) Nasalization
(E) Obliteration

The correct response is Option B.

Cranialization of the frontal sinus is the most appropriate management of this


patient who has a fracture of the posterior table of the frontal sinus with a
concomitant cerebrospinal fluid leak. This procedure is often recommended for
patients with severe comminution of the posterior table to resolve any
cerebrospinal fluid leakage. Bifrontal craniotomy is performed first to repair the
dura, and the posterior table of the frontal sinus and associated mucosa are
removed. The nasofrontal duct is occluded with a pericranial flap, disrupting the
communication of the duct with the frontal sinus. Following surgery, the brain
gradually expands to fill the space previously occupied by the frontal sinus.

Ablation of the frontal sinus involves total removal of the anterior and posterior
tables. This procedure is no longer performed due to its resultant cosmetic defects.

Exenteration involves removal of the anterior table of the frontal sinus only.
Although it results in a cosmetic deformity, it may be considered in patients who
have severe damage to the anterior table resulting from infection and who cannot
undergo immediate reconstruction.

Nasalization is a technique in which the nasofrontal duct is either stented or


enlarged to ensure adequate drainage of the frontal sinus. This procedure is
typically used in patients with frontal basilar fractures involving the nasofrontal
duct or floor of the sinus.

Obliteration of the frontal sinus is accomplished by removing all of the mucosa


within the frontal sinus and allowing the nasofrontal duct to occlude. Graft
material can be used for filling, or the space may remain open and eventually close
through the process of spontaneous osteogenesis. This technique is recommended
for patients in whom the patency of the nasofrontal duct is compromised.

References
1. Manson PN. Facial fractures. In: Aston SJ, Beasley RW, Thorne CH, eds.
Grabb & Smith’s Plastic Surgery. 5th ed. Philadelphia, Pa: Lippincott-Raven;
1997:383-412.
2. Rohrich RJ, Hollier LH. Management of frontal sinus fractures: changing
concepts. Clin Plast Surg. 1992;19:219-232.

A 32-year-old man sustains a fracture of the mandible in a motor vehicle collision.


The likelihood of concomitant cervical spine injury in this patient is closest to

(A) 10%
(B) 20%
(C) 30%
(D) 40%
(E) 50%

The correct response is Option A.

In patients who sustain facial fractures in motor vehicle collisions, the incidence of
cervical spine injury has been shown to range from 5% to 15%, according to the
results of multiple studies. Overall, multiple studies have reported the incidence of
concomitant injuries associated with facial fractures sustained during motor
vehicle collisions as ranging from 11% to 99%. These injuries are most likely to
include closed head injuries, soft-tissue lacerations to the face, head, or other
regions, and fractures of the ribs, pelvis, and lower extremities.

Because of the correlation between facial fractures and cervical spine injuries,
standard Advanced Trauma Life Support (ATLS) protocols recommend that the
cervical spine be immobilized until the absence of cervical spine injury can be
documented definitively. These injuries can result in paresis, paraplegia, and even
death; therefore, the surgeon must maintain a high index of suspicion in any
patient who sustains a mandibular fracture in a motor vehicle collision. Even
patients who have mandibular fractures resulting from physical altercations should
be evaluated carefully, although the incidence of concomitant cervical spine injury
is not as high as in those patients who are involved in motor vehicle collisions.

Patients with mandibular fractures often have other associated injuries, such as
lacerations of the face and head and other associated facial fractures. These
patients should also be evaluated for potential closed head injury, which is a life-
threatening concern associated with high mortality rates.

References
1. Fischer K, Zhang F, Angel MF, et al. Injuries associated with mandible
fractures sustained in motor vehicle collisions. Plast Reconstr Surg. 2001;108:328-
331.
2. Freidrich KL, Pena-Velasco G, Olson RA. Changing trends with mandibular
fractures: a review of 1,067 cases. J Oral Maxillofac Surg. 1992;50:586-589.
A 27-year-old man has malocclusion and tenderness around the orbits and bridge
of the nose after sustaining facial injuries in a motor vehicle collision. A
photograph and CT scan are shown above. The patient is to undergo open
reduction and internal fixation of the fractures.

Which of the following is the most appropriate management of the lacrimal


system?

(A) Observation
(B) Exploration of the lacrimal duct
(C) Placement of a silicone stent
(D) Immediate dacryocystorhinostomy
(E) Primary repair followed by dacryocystorhinostomy in three months

The correct response is Option A.

In this patient who has sustained a naso-orbitoethmoid fracture, the most


appropriate management of the lacrimal system is observation. The incidence of
injury to the nasolacrimal duct in patients who undergo open reduction and
internal fixation of naso-orbitoethmoid fractures but have no overlying lacerations
is fairly low. Although swelling or fracture may contribute to blockage of the duct
at the time of the initial injury, approximately 90% of patients will experience
improvement of symptoms with resolution of the swelling and reduction of the
fracture.

Exploration and/or manipulation of the duct are not recommended if there is no


obvious injury to the duct. These procedures may only result in further damage
because of the edema and friability of the tissues. Immediate
dacryocystorhinostomy is not warranted for the same reason.

Patients who have persistent epiphora after resolution of swelling should undergo
further evaluation. Dacryocystography can be performed for assessment of
possible nasolacrimal duct occlusion. If occlusion is present,
dacryocystorhinostomy is indicated.

References
1. Crawley WA, Vasconez HC. Midface, upper face, and panfacial fractures. In:
Ferraro JW, ed. Fundamentals in Maxillofacial Surgery. New York, NY: Springer-
Verlag; 1997:203-214.
2. Gruss JS, Hurwitz JJ, Nik NA, et al. The pattern and incidence of nasolacrimal
injury in naso-orbital-ethmoid fractures: the role of delayed assessment and
dacryocystorhinostomy. Br J Plast Surg. 1985;38:116-121.
3. Manson PN. Facial fractures. In: Aston SJ, Beasley RW, Thorne CH, eds.
Grabb & Smith’s Plastic Surgery. 5th ed. Philadelphia, Pa: Lippincott-Raven;
1997:383-412.

The CT scans shown above are from a 25-year-old man who sustained facial
injuries in a motor vehicle collision. In this patient, rigid fixation at which of the
following points is most likely to result in stable reduction of the fractures?

(A) Nasomaxillary buttress, inferior orbital rim, and zygomaticomaxillary buttress


(B) Zygomatic arch, central fragment, and nasomaxillary buttress
(C) Zygomatic arch, inferior orbital rim, and zygomaticomaxillary buttress
(D) Zygomaticofrontal suture, inferior orbital rim, and zygomaticomaxillary
buttress
(E) Zygomaticofrontal suture, orbital floor, and zygomaticomaxillary buttress
The correct response is Option D.

This patient has sustained a fracture of the zygomaticomaxillary complex, also


known as a zygoma fracture. To ensure stable reduction of the fracture, rigid
fixation is applied at the zygomaticofrontal suture, inferior orbital rim, and
zygomaticomaxillary buttress. Partial relapse may occur without this three-point
rigid fixation.

Central fragments are associated with naso-orbitoethmoid fractures, not


zygomaticomaxillary complex fractures. This fragment is comprised of the
ascending frontal process of the maxilla and the descending internal angular
process of the frontal bone. It provides the bony support for the medial canthus.

The nasomaxillary buttress is comprised of the piriform rim and is osteotomized


routinely during elective Le Fort I maxillary advancement procedures. This
structure is typically involved in midface fractures and not in fractures of the
zygomaticomaxillary complex.

The orbital floor is often involved in zygomaticomaxillary complex fractures but


not in simple fractures of the zygoma. Therefore, open reduction and internal
fixation of the orbital floor may not be necessary to ensure correction of the
zygomaticomaxillary fracture.

The zygomatic arch is contained within a periosteal sleeve and is often reduced
and fixed adequately with adequate reduction and fixation of the
zygomaticomaxillary complex. Open reduction and internal fixation through a
coronal approach may be required if the zygomatic arch is comminuted
significantly, but this is rare.

References
1. Crawley WA, Vasconez H. Midface, upper face, and panfacial fractures. In:
Ferraro JW, ed. Fundamentals of Maxillofacial Surgery. New York, NY: Springer-
Verlag; 1997:203-214.
2. Manson P. Facial fractures. In: Aston SJ, Beasley RW, Thorne CH, eds. Grabb
& Smith's Plastic Surgery. 5th ed. Philadelphia, Pa: Lippincott-Raven; 1997:383.

A 25-year-old man has diplopia two days after sustaining an orbital fracture in a
motor vehicle collision. On physical examination, he does not have enophthalmos;
review of CT scans obtained immediately after injury shows no bony displacement
or entrapment of the orbital contents within the fracture.

Which of the following is the most appropriate next step in management?


(A) Observation
(B) Administration of a corticosteroid
(C) Administration of a diuretic
(D) MRI
(E) Immediate operative exploration

The correct response is Option A.

Conservative management is recommended in this patient who has diplopia after


sustaining an undisplaced, stable orbital fracture without entrapment or
enophthalmos. Diplopia often occurs following orbital trauma and may be caused
by edema, neurovascular or muscle injury, or entrapment of surrounding structures
within the fracture. If entrapment is the cause, surgical release should be
performed within the first 24 to 48 hours after injury to prevent permanent muscle
damage. However, if there is no entrapment, as in this patient, observation for 10
to 14 days is most appropriate. This will allow for resolution of any edema or
temporary palsy. Repeat evaluation can then be performed at this time.

CT scan, not MRI, is most appropriate for evaluating the condition of the fracture
site.

Corticosteroids or diuretics are not indicated in patients who have diplopia


following orbital fracture.

Operative exploration is appropriate for reduction of bony fragments, release of


entrapped, compressed, or prolapsed tissues, and accurate restoration of orbital
volume, but is not the most appropriate next step in this patient.

References
1. Manson PN. Facial fractures. In: Aston SJ, Beasley RW, Thorne CH, eds.
Grabb & Smith’s Plastic Surgery. 5th ed. Philadelphia, Pa: Lippincott-Raven;
1997:383.
2. Smith ML, Williams K, Gruss JS. Management of orbital fractures. Operative
Techniques Plast Reconstr Surg. 1998;5:312.

A 16-year-old basketball player is undergoing evaluation 12 hours after sustaining


a nasal fracture in a basketball game. Physical examination of the fracture site
shows marked edema. The radix is stable. A single fracture fragment is displaced
from the bony nasal pyramid. Intranasal examination shows septal hematomas
bilaterally.

Which of the following is the most appropriate management?


(A) Drainage of the septal hematomas followed by closed reduction and splinting
in three days
(B) Immediate operative drainage of the septal hematomas, followed by closed
reduction and splinting
(C) Drainage of the septal hematomas and intraoral open reduction and internal
fixation of the nasomaxillary suture line in three days
(D) Closed reduction of the nasal fracture in three days, followed by submucosal
resection and inferior turbinate infracture in six months
(E) CT scan of the face to rule out a naso-orbitoethmoid fracture, followed by
open reduction and internal fixation of the fractures and septoplasty

The correct response is Option A.

In this patient who has sustained a nasal fracture with displacement of one
fragment, intranasal examination shows septal hematomas bilaterally. Therefore,
the most appropriate management is drainage of the hematomas, followed by
closed reduction and splinting in three days. It is imperative to drain the
hematomas immediately to prevent the development of complications, including
thickening of the septum (ie, “cauliflower” deformity) or dissolution and collapse
of the septum, which will ultimately result in a saddle-nose deformity. Because
this patient has significant swelling, closed reduction should be delayed. After the
swelling has decreased (typically at three to five days after injury), the septum and
nasal pyramid should be reduced, and the nasal pyramid should then be splinted.

Immediate closed reduction is difficult in any patient with significant edema.

Internal fixation and/or septoplasty are not required for management of


uncomplicated nasal fractures.

Submucous resection and turbinectomy are rarely necessary because most patients
with nasal fractures do not experience airway compromise.

References
1. Dolezal RF. Fractures of the nose. In: Cohen M, ed. Mastery of Plastic and
Reconstructive Surgery. Boston, Mass: Little, Brown & Co; 1994;2:1126-1135.
2. Manson P. Management of midfacial fractures. In: Georgiade GS, Riefkohl R,
Levin LS, eds. Plastic, Maxillofacial and Reconstructive Surgery. Baltimore, Md:
Williams & Wilkins; 1997:351-376.

A 23-year-old man is undergoing evaluation one week after sustaining a nasal


fracture. Each of the following is appropriate management of this patient’s injuries
EXCEPT
(A) closed realignment of the nasal fracture with forceps
(B) drainage of septal hematomas
(C) intranasal packing
(D) osteotomy and realignment of the nasal fracture
(E) use of a dorsal nasal splint

The correct response is Option D.

Complications associated with fracture of the nasal bones include hemorrhage and
hematoma. Bleeding is common with nasal trauma because of the rich blood
supply of the mucoperichondrium. Fracture of the nasal septum can lead to
hematoma, which frequently occurs bilaterally, as septal fractures communicate
between both sides of the nose. If untreated, a septal hematoma can become thick
and fibrotic, obstructing the nasal passage, or can cause pressure necrosis of the
nasal mucosa and cartilage, ultimately leading to septal perforation. Therefore,
appropriate management involves incision along the base or most inferior portion
of the hematoma, which will allow for drainage and prevent blood from refilling
the cavity. In addition, closed reduction is appropriate for septal fractures and
deviated nasal bones. Intranasal packing and dorsal nasal splints are typically used
to aid in maintaining the reduction.

Osteotomy should not be performed in patients with acute fractures because nasal
collapse may result. This procedure should be delayed until the fracture has healed
significantly.

References
1. Manson PN. Facial injuries. In: McCarthy JG, ed. Plastic Surgery. Philadelphia,
Pa: WB Saunders; 1990;2:988.
2. Verwoerd CD. Present day treatment of nasal fractures: closed versus open
reduction. Facial Plast Surg. 1992;8:220.

Mandible
A 67-year-old man has an ulcerated lesion of the anterior floor of the mouth with
exposed, desiccated mandible one year after undergoing surgical resection and
radiotherapy for squamous cell carcinoma of the anterior floor of the mouth. Initial
recovery from the procedure was uncomplicated. Pathologic evaluation of a
specimen obtained on excisional biopsy shows osteoradionecrosis. Radiation doses
greater than 6500 cGy and which of the following are the most likely precipitating
factors in this patient?

(A) Dental caries


(B) Dental implants
(C) Edentulous mandible
(D) Oral candidiasis
(E) Xerostomia

The correct response is Option A.

Osteoradionecrosis (ORN) is relatively uncommon. However, the risk of ORN


increases when the radiation dosage to the mandible exceeds 6500 cGy. Although
up to 30% of cases of ORN reportedly arise spontaneously, most reports note
dental caries and extraction sites as precipitating factors. Acute and chronic
periodontal disease in mandibulotomy sites can also lead to ORN.

Dental implants, an edentulous mandible, oral candidiasis, and xerostomia may all
be seen in cases of mandibular reconstruction and radiation. However, they do not
increase the risk of developing ORN.

Traditional treatment of ORN includes surgical debridement and antibiotic therapy


if infection is present. In advanced, extensive ORN, hyperbaric oxygen (HBO)
therapy may be used as an adjunct. However, controversy exists about using HBO
therapy instead of surgical resection and reconstruction of the mandible. The
decision to use HBO therapy should be made on an individual basis and with the
understanding that necrotic bone with sequestrum is unlikely to heal with HBO
therapy.

References:
1. Shaha A, Cordeiro P, Hidalgo D, et al. Resection and immediate microvascular
reconstruction in the management of osteoradionecrosis of the mandible. Head
Neck Surg. 1997;19(5):406-411.
2. Marx RE. A new concept in the treatment of osteoradionecrosis. J Oral
Maxillofac Surg. 1983;41:351-357.

Which of the following terms best describes the type of occlusion in which the
upper central incisor lies anterior to the lower central incisor in the sagittal plane?

(A) Buccal crossbite


(B) Lingual crossbite
(C) Open bite
(D) Overbite
(E) Overjet

The correct response is Option E.

Overjet is a horizontal measurement that refers to the distance between the incisal
aspect of the maxillary incisors and the incisal aspect of the mandibular incisors
with the teeth in centric occlusion. When the upper central incisor lies anterior to
the lower central incisor in the sagittal plane, this is known as overjet.
In contrast, overbite is a vertical measurement referring to the distance between
the maxillary incisor edge and the mandibular incisor edge with the teeth in centric
occlusion. An overbite or deep bite is one in which the upper central incisor
overrides the lower central significantly in the vertical dimension.

Buccal and lingual crossbite refer to the positioning of the mandibular molars with
respect to the maxillary molars in the transverse plane.

Open bite occurs when the maxillary and mandibular teeth fail to contact. This can
occur at any point in the dentition.

References:
1. Posselt U. Physiology of Occlusion and Rehabilitation. 2nd ed. Oxford:
Blackwell Scientific; 1968:3-24.
2. Profit W, Fields H. Malocclusion and dentofacial deformity in contemporary
society. In: Profitt W, Fields H, eds. Contemporary Orthodontics. St. Louis, MO:
Mosby-Year Book, Inc; 2000:1-22.

Removal of a tooth in a fracture line of the mandible is indicated in a patient with


which of the following conditions?

(A) Cavities in the tooth


(B) Fracture of the root of the tooth
(C) Loose tooth
(D) Multiple fractures of the mandible
(E) Periodontal disease

The correct response is Option B.

Indications for removal of teeth in mandibular fractures include fracture of the root
of the tooth, severe loosening of the tooth in presence of chronic periodontal
disease, extensive periodontal injury and broken alveolar walls, and displacement
of teeth from their alveolar socket. Periodontal disease alone is not an indication
for tooth removal. Multiple fractures of the mandible are also not an indication for
tooth removal because the teeth usually are needed for intermaxillary fixation prior
to open reduction and internal fixation of the fractures. History of caries would
warrant a referral to a dentist to ascertain whether any intervention would be
required but would not necessitate removal of that tooth at the time of fracture
management. Loose tooth is seen in most cases of mandibular fracture but is
addressed by proper alignment and reduction of all fractures.
References:
1. Crawley WA, Sandel AJ. Fractures of the mandible. In: Ferraro JW, ed.
Fundamentals of Maxillofacial Surgery. New York: Springer; 1997:192-203.
2. Polley JW, Flaff JS, Cohen M. Fractures of the mandible. In: Weinzweig J, ed.
Plastic Surgery Secrets. Philadelphia: Hanley & Belfus; 1999:164-172.

In distraction osteogenesis of the mandible using an external distractor, successful


formation of the bone is most dependent on which of the following?

(A) Consolidation period of two weeks


(B) Distraction rate of 1 mm per day
(C) Lag period of three days before initiation of distraction
(D) Stable fixation of the bone
(E) Supraperiosteal dissection of the bone

The correct response is Option D.

In distraction osteogenesis of the mandible using an external distractor, the most


important element in successful formation of the bone is adequate stabilization of
the bone edges. Unstable fixation of the bone allows excessive motion, which can
result in a fibrous union.

A consolidation period of two weeks is inadequate. A period of at least four to six


weeks usually is needed before the distraction devices can be safely removed.
Although a rate of 1 mm per day commonly is used in mandibular distraction,
rates of 2 mm or more per day have been shown to be successful in mandibular
distraction, particularly when an external device is used. Most surgeons use a lag
period before beginning distraction. However, a lag period has never been
demonstrated to be necessary in craniofacial distraction. Supraperiosteal dissection
of the bone is important in distraction of the extremities but has not been
demonstrated to be necessary in distraction of the mandible.

References:
1. Tavakoli K, Stewart KJ, Poole MD. Distraction osteogenesis in craniofacial
surgery: a review. Ann Plast Surg. 1998;40:88-89.
2. McCarthy JG, Stelnicki EJ, Mehrara BJ, et al. Distraction osteogenesis of the
craniofacial skeleton. Plast Reconstr Surg. 2001;107:1812-1818.
Microsurgery

Clinically proven effects of dextran include which of the following?

(A) Improved flap survival


(B) Increased systemic complications
(C) Ischemia reperfusion protection
(D) Leukopenia

The correct response is Option B.

Low-molecular-weight dextran is a polysaccharide produced by bacteria that is


frequently used as an antithrombotic agent in microsurgical procedures. It is
believed to have multiple actions, including decreasing platelet aggregation,
increasing fibrin degradation, inhibiting alpha-2 plasmin, decreasing factor VII
and von Willebrand factor and thereby altering platelet function, as well as acting
as a volume expander. Anaphylactic reactions to this substance can occur, and
many practitioners recommend administering a test dose of the hapten dextran one
hour before starting infusion. However, severe reactions are uncommon and
generally are noted early in its administration. Cases of pulmonary edema and
acute respiratory distress syndrome have been reported. Some animal experiments
and a few retrospective nonrandomized studies have shown improved patency
rates of anastomoses in free flaps; however, conflicting data have resulted in a lack
of consensus among microsurgeons. An overall increase in systemic complications
(pulmonary, cardiac, anaphylactic) has been demonstrated with dextran compared
with aspirin only. Improved flap survival, leukopenia, or ischemia reperfusion
protection has not been demonstrated with dextran.

References:
1. Conrad MH, Adams WP. Pharmacological optimization of microsurgery in the
new millennium. Plast Reconstr Surg. 2001;108:2088-2096.
2. Disa JD, Polvora VP, Pusic AL, et al. Dextran related complications in head and
neck microsurgery: do the benefits outweigh the risks? A prospective randomized
analysis. Plast Reconstr Surg. 2003;112:1534-1539.

Nasal Reconstruction

A 52-year-old woman has a full-thickness defect of the left nasal ala with a
diameter of 8 mm after undergoing Mohs’ micrographic surgery for removal of a
basal cell carcinoma. On physical examination, the defect involves the skin and a
portion of the lower lateral cartilage, including the free border of the ala. Which of
the following methods of reconstruction is most likely to prevent vestibular
notching and narrowing?

(A) Composite grafting of skin and cartilage from the ear


(B) Coverage with a bilobe flap rotated from the nasal dorsum
(C) Coverage with a pedicled nasolabial groove flap
(D) Excision of the lining and primary closure
(E) Full-thickness skin grafting with pretragal skin

The correct response is Option A.

Nasal defects in the alar rim are challenging to reconstruct. Thin skin coverage,
cartilage support, and thin lining are needed to replace this cosmetically prominent
site. Complications of alar rim reconstruction include notching, scarring, and
nostril obstruction and narrowing. Several choices are available for this region, but
the best cosmetic result will be obtained with a composite full-thickness graft from
the ear. This site gives the best match of the missing tissue in thickness and
structure. Composite cartilage grafts are limited by their ability to revascularize.
Inosculation occurs within 18 hours and vessel ingrowth sustains the graft over the
long term. Grafts greater than 1.5 to 2 cm are more precarious and may not attain
adequate perfusion to live. This 8-mm defect is well within the limits of expected
take of a composite graft. Some authors advocate adjunctive measures to increase
the take of a composite graft, such as cooling, hyperbaric oxygen therapy, or
increasing the surface area of contact between the graft and recipient site.

A forehead flap gives thick tissue without lining. It would have to be folded on
itself or skin grafted. It also requires two stages. Both nasolabial and bilobed flaps
are local options but are bulky if folded. If skin grafted, they can contract and
notch. These flaps also give additional scarring on the face. A skin graft on a local
lining flap is too thin and lacks cartilage support.

References:
1. Burget GC, Menick FJ. Aesthetic Reconstruction of the Nose. St. Louis, MO:
Mosby-Year Book; 1994.
2. Menick F. Reconstruction of the nose. In: Cohen M, ed. Mastery of Plastic and
Reconstructive Surgery. Vol 2. Philadelphia: Lippincott Williams & Wilkins;
1994:883-905.
3. Chandawaskar RY, Cervino AL, Wells MD. Reconstruction of nasal defects
using modified composite grafts. Br J Plast Surg. 2003;56:26-32.
4. Rapley JH, Lawrence WT, Witt PD. Composite grafting and hyperbaric oxygen
therapy in nasal tip reconstruction. Ann Plast Surg. 2001;46:434-438.

Orth - 2004
A 45-year-old woman with myofascial pain dysfunction has had pain in the
preauricular region for the past six months. Plain radiographs of the
temporomandibular joint are most likely to show which of the following?

(A) Anterior displacement of the disk


(B) Erosion of the anterior condyle
(C) Narrowing of the joint space
(D) Osteophytes of the condylar head
(E) No abnormalities

The correct response is Option E.

In myofascial pain dysfunction, radiographs show no abnormalities because the


disorder does not usually produce discernible anatomic abnormalities in the
temporomandibular joint (TMJ). Myofascial pain dysfunction is associated with
preauricular pain, occasional joint clicking, restricted jaw opening, and tenderness
of the masticatory muscles. Its causes are multifactorial and include bruxism,
anxiety, and occlusal abnormalities. Anterior displacement of the disk of the TMJ
cannot be identified on plain radiographs because the disk is composed of fibrous
tissue, which can be seen only on radiographs with contrast.

References:
1. Bessette RW. TMJ dysfunction. In: Achauer BM, Eriksson E, Vander Kolk C,
et al, eds. Plastic Surgery: Indications, Operations, and Outcomes. Vol 2. St Louis,
MO: Mosby; 2000:903.
2. Smith JW, Aston SJ, eds. Grabb and Smith’s Plastic Surgery. 4th ed. Boston,
MA: Little Brown & Co; 1991:335.

A 23-year-old woman undergoes Le Fort I osteotomy with impaction of the


maxilla because of vertical maxillary excess. The final vertical position of the
maxilla is most accurately determined using which of the following studies?

(A) Assessment of maxillary lip-tooth relationship


(B) Cephalometric analysis of the ANB angle
(C) Cephalometric analysis of the SNA angle
(D) Dental model surgery
(E) Panorex radiography

The correct response is Option A.


Assessment of the maxillary lip-tooth relationship is the most accurate study in
determining the final vertical position of the maxilla. This assessment can be made
on a cephalometric radiograph by performing a soft-tissue and skeletal analysis
and prediction tracing. Intraoperative assessment of the amount of tooth show is
also important in determining the final vertical position of the maxilla. Normally,
the lip-tooth relationship is 2 to 3 mm of tooth show.

Cephalometric analysis of the ANB and SNA angles are skeletal measurements.
The ANB angle relates the maxilla to the mandible in the horizontal plane. The
SNA angle relates the maxilla to the base of the cranium in the horizontal plane.
Neither measurement assesses the vertical position of the maxilla or the soft-tissue
envelope.

References:
1. Ferraro JW. Cephalometry and cephalometric analysis. In: Ferraro JW, ed.
Fundamentals of Maxillofacial Surgery. New York: Springer-Verlag; 1997:233-
245.
2. Wolford LM, Fields RT. Surgical planning. In: Booth PW, Hausamen JE,
Schendel SA, eds. Maxillofacial Surgery. Vol 2. London: Churchill Livingstone;
1999:1205-1257.
3. Schendel SA. Vertical maxillary deformities. In: Ferraro JW, ed. Fundamentals
of Maxillofacial Surgery. New York: Springer-Verlag; 1997:284-286.

Three weeks after undergoing bilateral maxillary advancement, a 28-year-old man


has exposure of two of the four maxillary plates in the oral cavity. The maxilla
appears stable. Which of the following is the most appropriate next step in
management?

(A) Instruction in oral hygiene and observation


(B) Irrigation and debridement of the wound followed by replacement of the two
exposed plates
(C) Maxillomandibular fixation with heavy elastic for two weeks
(D) Removal of all plates followed by intermaxillary fixation for six weeks
(E) Removal of the two exposed plates only

The correct response is Option A.

In this patient who has exposure of the internal hardware, the most appropriate
management is maintenance of optimum oral hygiene. As long as oral hygiene is
maintained, the oral mucosa is likely to granulate over the maxillary plates. After
satisfactory bone healing has been achieved, any exposed plates can be removed.
Removal of the hardware is not indicated in a patient who underwent surgery only
three weeks earlier because there is an increased risk of bony malunion. The
maxilla is stable, and maxillomandibular fixation is unnecessary. The patient can
continue nutrition with a soft diet without adverse sequelae.

References:
1. Gruss JS. Complications of internal fixation of the mandible. In: Yaremchuk
MJ, Gruss JS, Manson PN, eds. Rigid Fixation of the Craniomaxillofacial
Skeleton. Boston, MA: Butterworth Heinemann; 1992:228-229.
2. Wolfe A, Spiro S, Wider T. Surgery of the jaws. In: Aston SJ, Beasley RW,
Thorne CH, eds. Grabb & Smith=s Plastic Surgery. 5th ed. Philadelphia:
Lippincott Williams & Wilkins;1997:330.

The percentage of patients who have numbness in the distribution of the mental
nerve one year after undergoing sagittal split osteotomy is closest to

(A) 0%
(B) 10%
(C) 30%
(D) 60%
(E) 80%

The correct response is Option B.

According to the results of several studies, the risk for permanent damage to the
inferior alveolar nerve during sagittal split osteotomy is 5% to 10%. The inferior
alveolar nerve exits from the mental foramen to become the mental nerve, and the
incidence of permanent sensory disturbance in the distribution of the mental nerve
is similar to the incidence in the inferior alveolar nerve.

References:
1. Raveh J, Vuillemin T, Ladrach K, et al. New techniques for reproduction of the
condyle relation and reduction of complications after sagittal ramus split
osteotomy of the mandible. J Oral Maxillofac Surg. 1988;46:751.
2. Wolfe A, Spiro S, Wider T. Surgery of the jaws. In: Aston SJ, Beasley RW,
Thorne CH, eds. Grabb & Smith’s Plastic Surgery. 5th ed. Philadelphia: Lippincott
Williams & Wilkins; 1997:330.

A 27-year-old man has articulation of the mesiobuccal cusp of the first upper
molar with the distobuccal groove of the lower first molar. Cephalometric analysis
shows increased SNB angle and negative ANB angle. Which of the following
interventions are the most appropriate management of this patient’s facial
deformity?

(A) Fronto-orbital advancement and Le Fort I advancement


(B) Le Fort I setback and jumping genioplasty
(C) Le Fort III advancement and mandibular advancement
(D) Maxillary impaction and vertical reduction advancement genioplasty
(E) Sagittal split osteotomy and maxillary advancement

The correct response is Option E.

This patient has mandibular prognathism, which is treated with sagittal split
osteotomy (mandibular setback) and maxillary advancement. Physical
examination shows Angle class III malocclusion, and cephalometric analysis
shows excessive protrusion of the mandible in relation to the maxilla and base of
the cranium. Although mandibular setback alone may seem to be the most logical
treatment, most patients with mandibular prognathism require treatment with a
combination of mandibular setback and maxillary advancement. This corrects the
projecting mandible and fills the soft-tissue envelope, creating a better aesthetic
result.

In a patient with mandibular prognathism, fronto-orbital advancement and Le Fort


I advancement would not treat the mandibular protrusion. Le Fort I setback and
jumping genioplasty also would not correct the mandibular protrusion because
they do not address the mandible as a whole. Le Fort III advancement and
mandibular advancement would worsen the prognathic mandible. Maxillary
impaction and vertical reduction advancement genioplasty do not treat the
malocclusion or mandibular skeletal abnormality.

References:
1. Wolfe SA, Bucky L. Facial osteotomies. In: Georgiade GS, Riefkohl RR, Levin
LS, eds. Georgiade Plastic, Maxillofacial and Reconstructive Surgery. 3rd ed.
Baltimore: Williams & Wilkins; 1997:297-337.
2. Schendel SA. Orthognathic surgery. In: Achauer BM, Eriksson E, Guyuron B,
et al, eds. Plastic Surgery Indications, Operations, and Outcomes. Vol. 2. St.
Louis, MO: Mosby; 2000:871-895.
A 20-year-old woman with juvenile rheumatoid arthritis has worsening occlusion
two years after undergoing sagittal split osteotomy with mandibular advancement.
On examination, there is a loss of posterior facial height bilaterally and an anterior
open bite. She has Angle class II malocclusion. Serial cephalometric analysis
shows progressive posterior movement of the B point. Which of the following is
the most likely cause of the worsening occlusion?

(A) Continued growth of the maxilla


(B) Improper intraoperative seating of the condyles in the glenoid fossae
(C) Improper preoperative and postoperative orthodontic treatment
(D) Loosening of all of the plates of the rigid internal fixation
(E) Progressive condylar resorption

The correct response is Option E.

Progressive condylar resorption is a late cause of open bite that occurs mainly in
young women. It is associated with condylar shortening, a decrease in posterior
facial height, clockwise rotation of the mandible, and Angle class II malocclusion.
Slow progressive posterior movement of the point B on serial cephalometric
analysis is a classic finding. The exact cause of the problem is unknown.

Poor orthodontic treatment can result in a recurrence of malocclusion in the


postoperative period. However, the patient described above exhibits many of the
classic findings of progressive condylar resorption, making that a more likely
diagnosis.

The most likely cause of immediate postoperative open bite is improper seating of
the condyles in the glenoid fossae during surgery. It is important to take the patient
out of intermaxillary fixation after fixation of the osteotomies is completed to
ensure that the condyles are properly seated. During this process, the occlusion
and path of the opening of the mandible are checked. In a skeletally mature
female, continued growth of the maxilla would be unusual. Lastly, it would be
unusual for all of the plates of the rigid internal fixation to loosen.

References:
1. Mason ME, Schendel SA. Revision orthognathic surgery. In: Booth PW,
Schendel SA, Hausamen JE, eds. Maxillofacial Surgery. Vol. 2. London: Churchill
Livingstone; 1999;1321-1334.
2. Sinn DP, Ghali GE. The long-term unfavorable results in orthognathic surgery.
In: Kaban LB, et al., eds. Complications in Oral and Maxillofacial Surgery.
Philadelphia: WB Saunders; 1997:255-264.
A 28-year-old woman who underwent Le Fort I osteotomy six weeks ago comes to
the office for follow-up evaluation. She says her nose is now wider than it was
before the procedure. Addition of which of the following interventions to the
osteotomy procedure would have effectively minimized this adverse result?

(A) External splinting of the nose


(B) Placement of an alar cinch suture
(C) Modified Weir excision
(D) Reduction of the amount of piriform rim exposure
(E) V-Y advancement closure of the lip

The correct response is Option B.

Placement of an alar cinch suture helps decrease the degree of widening of the alar
base that occurs after exposure of the anterior maxilla for orthognathic surgery or
trauma management. This suture is placed in the base of the ala bilaterally and
then is tightened until the desired effect is achieved.

External splinting of the nose has no effect on dimensional changes of the nasal
ala. Although a modified Weir excision is designed to treat alar flare, it does not
address the increased width of the alar base seen after Le Fort I osteotomy.
Reduction of piriform rim exposure is not appropriate for this patient. During Le
Fort I osteotomy, complete exposure of the piriform rim is essential because the
rim serves as a landmark from which to measure movement in the maxilla. Also,
the bone in the piriform rim and the malar buttress provides a stable platform for
rigid fixation.

V-Y advancement is used during oral mucosal closure, especially after Le Fort I
osteotomy, which tends to flatten the upper lip. This suture technique advances the
tissue anteriorly to add fullness to the upper lip, but has no effect on nasal width.

References:
1. Betts NJ. Techniques to control nasal features. Atlas Oral Maxillofac Surg Clin
Am. 2000;8:53-69.

Which of the following orthognathic movements is the most unstable and prone to
relapse?

(A) Mandibular advancement


(B) Mandibular narrowing
(C) Maxillary advancement
(D) Maxillary widening
(E) Sliding genioplasty

The correct response is Option D.

Transverse widening of the maxilla is the most unstable orthognathic movement.


With this procedure, a patient may lose as much as 50% of the movement at one
year after surgery. Maxillary downgrafts and mandibular setbacks are also
relatively unstable procedures. Mandibular advancement, mandibular narrowing,
maxillary advancement, and sliding genioplasty are all considered stable
movements.

References:
1. Philips C, Medland WH, Fields HW, et al. Stability of surgical maxillary
expansion. Int J Adult Orthop Orthogn Surg. 1992;7:139-146.
2. Proffit WR, Phillips C. Physiologic responses to treatment and postsurgical
stability. In: Proffit WR, White RP, Sarver DM, eds. Contemporary Treatment of
Dentofacial Deformities. St. Louis, MO: Mosby; 2003:646-676.

In patients with vertical maxillary excess undergoing Le Fort osteotomy with


maxillary impaction, which of the following findings is most likely
postoperatively?

(A) Increased mentalis strain


(B) Increased upper incisal show
(C) More obtuse nasolabial angle
(D) Retrogenia
(E) Widened alar base

The correct response is Option E.

Patients with vertical maxillary excess, or long-face syndrome, have a narrow alar
base, an obtuse nasolabial angle, and an anterior open bite. Mentalis muscle strain
and labial incompetence are increased, and there is excess gingival show and
exposure of the upper incisors.

Appropriate management is Le Fort I osteotomy with maxillary impaction;


osseous genioplasty is also performed in some patients. These procedures will
correct many of the findings associated with this condition, including decreasing
the mentalis muscle strain and incisal show and creating a more acute nasolabial
angle. The alar base will be widened. Le Fort I osteotomy also rotates the
mandible forward and upward, resolving the retrogenia associated with long-face
syndrome. Postoperative lateral cephalograms will show forward autorotation of
the mandible with counterclockwise rotation.

References:
1. McCarthy JG, Kawamoto HK, Grayson BH, et al. Surgery of the jaws. In:
McCarthy JG, ed. Plastic Surgery. Vol. 2. Philadelphia: WB Saunders; 1990;1187.
2. Wolfe SA, Spiro SA, Wider TM. Surgery of the jaws. In: Aston SJ, Beasley
RW, Thorne CH, eds. Grabb & Smith’s Plastic Surgery. 5th ed. Philadelphia:
Lippincott-Raven; 1997:321-333.

In patients with Treacher Collins syndrome, which of the following is a


characteristic skeletal finding?

(A) Brachycephaly
(B) Hypertelorism
(C) Macrogenia
(D) Malar hypoplasia
(E) Preaxial polysyndactyly

The correct response is Option D.


The characteristic skeletal finding in patients with Treacher Collins syndrome is
hypoplasia of the malar bones, which often occurs in conjunction with clefting
through the zygomatic arches. Patients also have hypoplasia of the maxilla and
mandible and antegonial notching of the angle of the mandible. Occlusion is Angle
class II; there is an anterior open bite and clockwise rotation of the occlusal plane.
Effects on the temporomandibular joint are varied.

Brachycephaly, macrogenia, preaxial polysyndactyly, and hypertelorism do not


occur in patients with Treacher Collins syndrome.

References:
1. Marsh JL, Celin SE, Vannier MW, et al. The skeletal anatomy of
mandibulofacial dysostosis (Treacher Collins syndrome). Plast Reconstr Surg.
1986;78:460.
2. Posnik JC. Treacher Collins syndrome. In: Aston SJ, Beasley RW, Thorne CH,
eds. Grabb & Smith’s Plastic Surgery. 5th ed. Philadelphia: Lippincott Williams &
Wilkins; 1997:313.

Pressure Sore
A pressure sore involving full-thickness skin and subcutaneous tissue to the level
of the underlying muscle fascia is classified as which of the following?

(A) Grade I
(B) Grade II
(C) Grade III
(D) Grade IV
The correct response is Option B.

A lesion with this description is classified as a Grade II pressure sore. Pressure


sores are classified by depth of necrosis. Grade I pressure sores extend to the
epidermis and superficial dermis only. Grade II pressure sores involve full-
thickness skin and extend to adipose tissue. Grade III pressure sores involve full-
thickness skin and extend to subcutaneous tissue and underlying muscle. Grade IV
pressure sores extend through all layers into the underlying bone or joint space.

Common risk factors for developing a pressure sore include neurologic


impairment, old age, and hospitalization. Generally, the greater the number of risk
factors, the greater the patient’s risk of developing a pressure sore.

References:
1. Staas WE Jr, LaMantia JG. Decubitus ulcers and rehabilitation medicine. Int J
Dermatol. 1982;21:437.
2. Berlowitz DR, Wilking SVB. Risk factors for pressure sores: a comparison of
cross-sectional and cohort-derived data. J Am Geriatr Soc. 1989;37:1043.
A 45-year-old man with paraplegia (Ashworth 5 spasticity) recently underwent
coverage of a superficial, cleanly debrided trochanteric hip ulcer with a tensor
fascia lata transposition flap (shown above). Which of the following interventions
is most appropriate to ensure stable coverage of the wound?

(A) Intrathecal administration of baclofen via an implantable pump


(B) Parenteral administration of a broad-spectrum antibiotic for six weeks
(C) Retrogasserian rhizotomy
(D) Ten weeks of bed rest on an air-fluidized mattress (Clinitron)
The correct response is Option A.

To decrease spasticity, baclofen should be administered by an implantable pump


before and after flap coverage. Spasticity contributes to flap breakdown by
shearing force and should be controlled to ensure stable coverage of the wound. In
some studies, flap failure has occurred in nearly 90% of patients with pressure
sores. Prevention of this serious complication requires close control of all
variables, including nutrition and postoperative pressure management. Long-term
parenteral administration of antibiotics plays no role in the stability of coverage of
a clean superficial wound. Ten weeks of bed rest on an air-fluidized mattress is not
likely to be useful for a pressure ulcer in a lateral area. Retrogasserian rhizotomy,
which interrupts the trigeminal (V) nerve, is not appropriate for this patient.

References:
1. Mess SA, Kim S, Davison S, Heckler F. Implantable Baclofen pump as an
adjuvant in treatment of pressure sores. Ann Plast Surg. 2003;51(5):465-467.
A 28-year-old man with a 10-year history of paraplegia has septicemia and a large
grade IV pressure ulcer over the greater trochanter. MRI shows communication
with the hip joint. After excision of the ulcer, which of the following is the most
appropriate next step in management?

(A) Administration of a culture-specific antibiotic for six weeks


(B) Coverage with a tensor fascia lata flap
(C) Coverage with a total thigh flap
(D) Coverage with a vastus lateralis flap
(E) Resection of the femoral head

The correct response is Option E.

The most appropriate management of this patient’s pressure ulcer is resection of


the femoral head, also known as Girdlestone arthroplasty. The sinogram finding of
communication of the ulcer with the hip joint is consistent with osteomyelitis,
which typically occurs in association with pyarthrosis. Resection of the femoral
head will effectively remove the infected tissue in this patient, and vascular tissue
should be used to obliterate the dead space. The vastus lateralis flap can be
advanced into the acetabular fossa as a muscle or musculocutaneous flap.

Administration of an antibiotic for six weeks will control wound sepsis but will
not treat osteomyelitis.

The tensor fascia lata flap is a sensate flap that is appropriate for coverage of less
extensive trochanteric ulcers. This flap lies proximal to the site of the ulcer and
can be easily transferred. Its vascular pedicle is based on perforating vessels from
the tensor fascia lata muscle. However, it cannot be used alone in a patient with
osteomyelitis.

Coverage with a total thigh flap is appropriate only as an end-stage procedure in a


patient who has undergone amputation of the lower limb.

References:
1. Evans GR, Lewis VL, Mason PN, et al. Hip joint communication with pressure
sore: the refractory wound and the role of Girdlestone arthroplasty. Plast Reconstr
Surg 1993;91:288-294.
2. Mancoll JS, Phillips LG. Pressure sores. In: Achauer BM, Eriksson E, Vander
Kolk C, et al, eds. Plastic Surgery: Indications, Operations, and Outcomes. Vol 1.
St Louis, MO: Mosby; 2000:447-462.
3. Mathes SJ, Nahai F. Reconstructive Surgery: Principles, Anatomy, and
Technique. Vol 2. New York: Quality Medical Publishing; 1997:1293-1306.
Skin Grafts
Which of the following is the best donor site for delayed multiple harvesting of
split-thickness skin grafts?

(A) Back
(B) Lateral forearm
(C) Medial arm
(D) Medial forearm
(E) Medial thigh

The correct response is Option A.

The selection of a donor site depends largely on donor site morbidity and skin
thickness. The back provides a nearly ideal donor site for repeated harvesting of
split-thickness skin grafts and has large areas of thick skin available for harvesting.
The lateral forearm exhibits unacceptable donor site morbidity. The medial arm,
medial forearm, and medial thigh have skin of insufficient thickness to allow
multiple harvesting.

The number of times that a donor site can be harvested for split-thickness skin
grafts is limited by the thickness of the dermis at the site. A split-thickness skin
graft includes the epidermis and part of the dermis. The donor site of a split-
thickness graft heals by migration from the remnant epithelia of the dermal
appendages, such as hair roots and sweat and sebaceous glands. Therefore, the
epidermis regenerates but the dermis does not. A repeat split-thickness graft may
be harvested once the skin has reepithelialized, but a thinner dermis will remain at
the donor site.

References:
1. Hardesty RA, Herber SC, Place MJ. Basic technique and principles in plastic
surgery. In: Aston SJ, Beasley RW, Thorne CH, eds. Grabb & Smith’s Plastic
Surgery. 5th ed. Philadelphia: Lippincott Williams & Wilkins; 1997:13-26.
2. Rudolph R, Ballantyne DL Jr. Skin grafts. In: McCarthy JG, May JW, Littler
JW, eds. Plastic Surgery. Vol 1. Philadelphia: WB Saunders; 1990:221-274.

Which of the following bone grafts exhibits the greatest inductive capacity?

(A) Allogenic
(B) Autologous cancellous
(C) Autologous cortical
(D) Free vascularized
(E) Xenogenic
The correct response is Option B.

Cancellous bone grafts have the greatest inductive capacity (ability to stimulate the
formation of new bone) because they contain bone morphogenic proteins that
stimulate bone growth. Cortical bone grafts and allogenic and xenogenic grafts
have less inductive capacity. Free vascularized bone grafts have no inductive
capacity because they do not rely on stimulating new bone formation.

References:
1. Lee WPA, Butler PEM. Transplant biology and applications to plastic surgery.
In: Aston SJ, Beasley RW, Thorne CH, eds. Grabb & Smith’s Plastic Surgery. 5th
ed. Philadelphia: Lippincott Williams & Wilkins; 1997:27-35.
2. Bishop AT. Vascularized bone grafting. In: Green DP, Hotchkiss RN, Pederson
WC, eds. Green=s Operative Hand Surgery. Vol 2. 4th ed. New York: Churchill
Livingstone, 1998:1221-1250.

Which of the following is an advantage of using the dermal regeneration template


(Integra) instead of a thin split-thickness autograft for reconstruction of the hand?

(A) Elimination of donor site


(B) Improved cosmesis
(C) Increased wound contraction
(D) Reduced risk of hematoma
(E) Shorter healing time

The correct response is Option B.

The advantages of Integra include the availability of large quantities, the


simplicity and reliability of the placement technique, and the pliability and
cosmetic appearance of the resulting cover.
Integra does not eliminate the use of a donor site. However, the skin graft used at
the second stage is typically thinner than that used in single-stage grafting, so
Integra does avoid the use of a deep donor site, which decreases the risk of
infection, scarring, and permanent pigment changes.

Wound contraction with Integra is typically less than that with single-stage split-
thickness skin grafting. Compared with autologous skin, Integra purportedly has
no decrease in the hematoma rate. In one study, the incidence of hematoma under
Integra was 9 out of 39 cases. Because Integra requires a second surgery after 3 to
4 weeks for coverage with a thin split-thickness skin graft, it has a longer healing
time until final wound coverage.

References:
1. Dantzer E, Braye FM. Reconstructive surgery using an artificial dermis
(Integra): results with 39 grafts. Br J Plast Surg. 2001;54(8):659-664.
2. Palao R, Gomez P, Huguet P. Burned breast reconstructive surgery with Integra
dermal regeneration template. Br J Plast Surg. 2003;56(3):252-259.

Which of the following grafts has minimal resorption and loss of volume?

(A) Bone
(B) Cartilage
(C) Macro-fat
(D) Micro-fat
(E) Muscle

The correct response is Option B.


Although all of these tissues may be used for grafts, autologous cartilage grafts
offer minimal resorption and loss of volume and provide good results in many
clinical settings. The metabolic rate of cartilage is 1/100 to 1/500 the rate of other
human tissues. It has low glycolytic activity and consumption because of its small
cell population and relative isolation by the cartilage matrix. This isolation helps
protect cartilagenous tissue from resorption. Readily accessible sites for cartilage
grafts are the septum, auricular cartilage, and ribs.

Bone grafts, which depend on the size of the bone, have variable resorption rates
based on the clinical circumstance. Macro-fat grafts are usually unreliable. They
have high resorption rates and are more effectively used as dermal fat grafts to
minimize resorption and fibrosis. Given by lipoinjection, micro-fat grafts may
exhibit greater longevity than macro-fat grafts. However, their resorption may be
25% to 50% of the injected volume. Muscle grafts are not routinely used. Rather,
vascularized muscle flap grafts are preferred for transferring muscle tissue.

References:
1. Lee WPA, Butler PEM. Transplant biology and applications to plastic surgery.
In: Aston SJ, Beasley RW, Thorne CH, eds. Grabb & Smith’s Plastic Surgery. 5th
ed. Philadelphia: Lippincott Williams & Wilkins; 1997:27-38.
2. Brent B. Repair and grafting of cartilage in perichondrium. In: McCarthy JG,
May JW, Littler JW, eds. Plastic Surgery. Vol 1. Philadelphia: WB Saunders;
1990:559-582.
Which of the following bone grafts does NOT rely on creeping substitution as a
mode of remodeling?

(A) Allogenic
(B) Autologous cancellous
(C) Autologous cortical
(D) Free vascularized
(E) Xenogenic

The correct response is Option D.

Free vascularized bone grafts do not rely on creeping substitution (replacement of


necrotic bone with osteoblasts and new vascular ingrowth) for remodeling. They
do not need to stimulate new bone formation because they are used when little or
no bone has been lost.

All nonvascularized bone grafts undergo a degree of resorption and remodeling,


including creeping substitution, and have some degree of inductive capacity
(ability to stimulate new bone formation). Cancellous bone grafts contain bone
morphogenic proteins that stimulate the formation of new bone. Cortical bone
grafts and allogenic and xenogenic grafts have less of this inductive capacity.

References:
1. Lee WPA, Butler PEM. Transplant biology and applications to plastic surgery.
In: Aston SJ, Beasley RW, Thorne CH, eds. Grabb & Smith=s Plastic Surgery. 5th
ed. Philadelphia: Lippincott Williams & Wilkins; 1997:27-35.
2. Bishop AT. Vascularized bone grafting. In: Green DP, Hotchkiss RN, Pederson
WC, eds. Green’s Operative Hand Surgery. Vol 2. 4th ed. New York: Churchill
Livingstone, 1998:1221-1250.

Skin lesion - 2004

In a 56-year-old man who has a lesion on the tip of the nose, histopathologic
examination of a punch biopsy specimen shows basal cell carcinoma. Mohs’
micrographic resection is recommended if this patient has which of the following
types of basal cell carcinoma?
(A) Morpheaform
(B) Nodular
(C) Pigmented
(D) Ulcerated

The correct response is Option A.

Mohs’ micrographic resection involves serial excision of the visible tumor in


layers of increasing depth followed by immediate microscopic examination of the
excised tissue. A map of the specimen is created and divided into sections; the
presence of tumor in a certain section necessitates further excision in that section
only. Serial excision continues until there is no longer any microscopic evidence
of tumor. Cure rates for this technique, when used for treatment of basal cell
carcinoma, have been reported to range from 98.2% to 99.9%, compared with 94%
to 95% for standard surgical excision.

Mohs’ micrographic resection is appropriate for removal of basal cell carcinomas,


especially recurrent lesions and those lesions that are located in anatomically
sensitive sites, such as the eyelid margin, medial canthus, and nasal ala. This
technique is particularly effective for the morpheaform variation of basal cell
carcinoma, also know as sclerosing basal cell carcinoma, because of the
difficulties encountered when attempting to visually determine the tumor margins,
which are often not well demarcated. In addition, because morpheaform basal cell
carcinoma is more prone to recur, pathologic evaluation of the margins is indicated
prior to reconstruction.

Because nodulated, pigmented, and ulcerated basal cells typically have distinct
margins and are amenable to primary excision, Mohs’ micrographic resection is
usually unnecessary.

References
1. Olbricht SM. Cutaneous carcinomas. In: Georgiade GS, Riefkohl R, Levin LS,
eds. Plastic, Maxillofacial and Reconstructive Surgery. Baltimore, Md: Williams
& Wilkins; 1997:126-127.
2. Robins P. Mohs’ micrographic surgery. In: McCarthy JG, ed. Plastic Surgery.
Philadelphia, Pa: WB Saunders Co; 1990;7:3652.

Which of the following medications is most appropriate to prevent and to treat


heterotopic ossification?

(A) Calcitonin
(B) Etidronate
(C) Mithramycin
(D) Phosphate
The correct response is Option B.

Heterotopic ossification is an abnormal proliferation of bone that occurs in patients


who have sustained burns, spinal cord injuries, and closed head trauma. This
condition becomes apparent initially on radiographs one to four months after
injury. It may occur in up to 40% of patients who sustain spinal cord injuries and
is clinically significant in one-half of those patients.

Etidronate (Didronel) and pamidronate (Aredia) are most appropriate for


prevention or treatment of heterotopic ossification. These agents act primarily on
bone to inhibit the formation, growth, and dissolution of hydroxyapatite crystals
and do not impair fracture healing.

Calcitonin, mithramycin, and phosphate decrease serum calcium levels and do not
directly affect heterotopic ossification.

References
1. Donelan MB. Reconstruction of the burned hand and upper extremity. In:
McCarthy JG, ed. Plastic Surgery. Philadelphia, Pa: WB Saunders Co;
1990;8:5473-5476.
2. Hotchkiss RN. Elbow contracture. In: Green DP, et al, ed. Operative Hand
Surgery. New York, NY: Churchill Livingstone, Inc; 1999;1:668-669, 679-681.
3. Physicians’ Desk Reference. Montvale, NJ: Medical Economics Co; 2003:2825.

A 55-year-old man has a lesion on the right forearm that has enlarged over the past
six weeks. A photograph is shown above. Which of the following is the most
likely diagnosis?

(A) Cylindroma
(B) Dermatofibroma
(C) Keratoacanthoma
(D) Seborrheic keratosis
(E) Syringoma
The correct response is Option C.

The most likely diagnosis is keratoacanthoma, a common cutaneous neoplasm that


typically occurs in men older than age 50 years. Keratoacanthomas grow rapidly
over several weeks and are believed to regress spontaneously if left untreated.
They are characterized by an umbilicated center with a keratin plug. Because they
are difficult to distinguish from squamous cell carcinoma and may indeed be
linked, excision for histopathologic confirmation is recommended.

Cylindromas are round, firm, fleshy tumors of the scalp that are rarely solitary.
Dermatofibroma is a fibrous, papular lesion characteristically found on the lower
extremities in young adults.

Seborrheic keratosis has a waxy, greasy, or pressed-on appearance and generally


occurs on the face and trunk of older persons.

Syringoma is a flesh-colored or yellow papule that typically develops in females


during adolescence or early adulthood. These lesions may be multiple and often
occur only on the lower eyelids.

References
1. Gasparro FP. P53 and dermatology. Arch Dermatol. 1998;134:1029-1032.
2. Harris AO, Levy ML, Goldberg LH, et al. Nonepidermal and appendageal skin
tumors. Clin Plast Surg. 1993;20:115-130.
3. Owen C, Telfer N. Keratoacanthoma. In: Lebwohol MG, ed. Treatment of Skin
Disease: Comprehensive Therapeutic Strategies. Philadelphia: Mosby Year -
Book, Inc; 2002:315.

A 51-year-old woman has had multiple lesions around the nostrils for the past two
years. She says that the lesions first appeared similar to pimples and have
enlarged. A photograph is shown above. The lesions have not resolved with
intralesional injections of corticosteroids. Findings on nasal smear and tissue
cultures are negative. Histologic examination of a biopsy specimen of a lesion
shows noncaseating epithelioid granulomata.

Which of the following is the most likely diagnosis?


(A) Cutaneous sarcoidosis
(B) Keloids
(C) Neurofibromatosis
(D) Rhinosporidiosis
(E) Wegener’s granulomatosis

The correct response is Option A.

In this 51-year-woman who has the findings shown in the photograph, the most
likely diagnosis is cutaneous sarcoidosis. This condition occurs in 10 Caucasians
and 35 African Americans per 100,000 of population. Although sarcoidosis
typically affects the lungs and intrathoracic lymph nodes, cutaneous symptoms,
such as maculopapular eruptions, sarcoidal plaques, lupus pernio lesions, and
subcutaneous and cutaneous nodules, occur in 25% of patients. Histologic
examination of a biopsy specimen of affected skin shows noncaseating
granulomata and aggregates of epithelioid cells and giant cells containing
Schaumann (asteroid bodies) within the dermis.

Appropriate treatment of cutaneous sarcoidosis is intralesional administration of a


corticosteroid or oral administration of hydroxychloroquine sulfate or
methotrexate. Oral corticosteroids are reserved for patients with severe cutaneous
disfigurement or symptomatic involvement of internal organs. Surgical options are
limited because of the systemic nature of this condition.

In patients with keloids, histologic examination shows extensive fibrous tissue


with scattered collagen fibers but without a definite architecture or a
granulomatous pattern.

Neurofibromatosis is characterized histologically by swirls of fibroblastic cells,


collagen, and nerve fibers. Granulomas are not present.

Rhinosporidiosis is a fungal granulomatous condition of the nose caused by


Rhinosporidium seeberi bacteria. Affected patients have pedunculated masses with
reddish-purple discoloration. Although findings on culture are usually negative,
microscopic examination of nasal smears typically shows sporangia.

Wegener’s granulomatosis is a systemic disease that initially causes localized


perforation of the septum or multiple intranasal ulcerations. Histologic
examination shows findings consistent with necrotizing granulomatous vasculitis.

References
1. Cody DT, DeSanto LW. Neoplasms of the nasal cavity. In: Cummings CW,
Fredrickson JM, Harker LA, et al, eds. Otolaryngology Head and Neck Surgery.
3rd ed. Saint Louis, Mo: Mosby Year - Book, Inc; 1998;2:883-901.
2. Landau M, Mevorah B. Cutaneous manifestations of systemic diseases. In:
Parish LC, Brenner S, Ramos-e-Silva M, eds. Women’s Dermatology – From
Infancy to Maturity. Pearl River, NY: The Parthenon Publishing Group; 2001:243-
250.
3. Weedon D. The granulomatous reaction pattern. In: Weedon D, ed. Skin
Pathology. 2nd ed. London, England: Churchill Livingstone; 2002:193-209.

A 46-year-old woman has had painful purpura-like lesions on the breasts,


abdomen, and lower extremities for the past three weeks. She currently undergoes
dialysis for end-stage renal disease. Physical examination shows several dry,
leathery, full-thickness areas of skin necrosis. A photograph is shown above.
Calciphylaxis is diagnosed.

Laboratory studies are most likely to show an increased serum level of which of
the following?

(A) Calcium
(B) Glucose
(C) Parathyroid hormone
(D) Protein C
(E) Thyroid-stimulating hormone

The correct response is Option C.

Calciphylaxis, also known as uremic gangrene syndrome, is a rare complication of


end-stage renal disease. This condition is characterized by painful, ischemic,
violaceous skin lesions on the extremities and sometimes the trunk that exhibit a
livido reticularis pattern. Full-thickness skin necrosis ensues, leading first to
secondary infection, then to sepsis, and frequently to death; the mortality rate in
patients with calciphylaxis has been reported to be as high as 60%. Histologic
examination of affected skin shows fat necrosis, calcification of the subcutaneous
tissues, and microcalcifications in the intima and media of small arteries. Luminal
narrowing of the vessels with intimal hyperplasia is common, and complete
occlusion also occurs.

Calciphylaxis is a result of the metabolic imbalance in calcium and phosphate


homeostasis that is frequently observed in patients with renal failure. Secondary
hyperparathyroidism causes an abnormal increase in calcium X phosphate product,
which can lead to soft-tissue calcification. In these patients, serum levels of
parathyroid hormone are most likely to be increased. Hyperphosphatemia is also
characteristic.

The lesions that occur in patients with calciphylaxis rarely heal spontaneously and
frequently become infected. Therefore, various management options should be
considered, including debridement of the lesions, topical administration of
antimicrobial agents, and frequent dressing changes, followed by skin grafting, or
direct excision and closure of the lesions. However, in this patient, the dry,
leathery composition of the affected skin suggests that infection has not yet
developed. Subtotal thyroidectomy is also advocated for treatment in some
patients.

Although hypercalcemia may be present, serum calcium levels are normal in most
patients with this condition.

Many patients with end-stage renal disease also have diabetes mellitus, but
hyperglycemia is not typically associated with calciphylaxis.

A deficiency of, and not an increase in, serum protein C levels has been proposed
to be a contributing factor to tissue death in these patients because it causes
thrombosis in small vessels.

Increased serum levels of thyroid-stimulating hormone occur in patients with


hypothyroidism.

References
1. Burkhart CG, Burkhart CN, Mian A. Calciphylaxis: a case report and review of
the literature. Wounds. 1999;11:58-61.
2. Kane WJ, Paetty PM, Sterioff S, et al. The uremic gangrene syndrome:
improved wound healing in spontaneously forming wounds following subtotal
parathyroidectomy. Plast Reconstr Surg. 1996;98:671-678.
3. Senturk S, Hosnuter M, Tosun Z, et al. Calciphylaxis: cutaneous necrosis in
chronic renal failure. Ann Plast Surg. 2002;48:104-105.

Which of the following pigmented nevi is classified as a hamartoma?


(A) Acral nevus
(B) Becker nevus
(C) Dysplastic nevus
(D) Halo nevus
(E) Spitz nevus

The correct response is Option B.

A Becker nevus is a common benign cutaneous hamartoma with epidermal and/or


dermal elements. These lesions can occur at birth but often develop during the
second and third decades of life, and males are affected more frequently. Becker
nevi are typically brown patches or plaques on the upper trunk. Hypertrichosis is
commonly associated, and an underlying smooth muscle hamartoma is present.
The term “nevus” is actually a misnomer because histologic examination of this
lesion shows no nevus cells.

Acral nevi are typically junctional or compound and are located on the palmar or
plantar surfaces.

A dysplastic nevus is referred to by several terms, including Clark’s nevus,


atypical nevus, or atypical mole. This lesion is a precursor to malignant melanoma.

A halo nevus, or Sutton’s nevus, is a central melanocytic nevus surrounded by a


rim or halo of hypopigmented skin. This loss of pigmentation typically
corresponds with spontaneous regression of the central nevus and is thought to
occur secondary to a cell-mediated and humoral immune response to nevus
antigens.

A Spitz nevus is a benign proliferation of melanocytes on the face, trunk, or


extremities. This lesion develops in children and is characterized by a rapid,
irregular growth phase. Because it is difficult to differentiate from malignant
melanoma, excision for histopathologic confirmation is recommended.

References
1. Barnhill RL. Malignant melanoma, dysplastic melanocytic nevi, and Spitz
tumors: histologic classification and characteristics. Clin Plast Surg. 2000;27:331-
360.
2. Casson P, Colen S. Dysplastic and congenital nevi. Clin Plast Surg.
1993;20:105-113.
3. Schaffer JV, Bolognia JL. The clinical spectrum of pigmented lesions. Clin
Plast Surg. 2000;27:391-408.
A 60-year-old farmer with multiple actinic keratoses undergoes topical therapy
with administration of 5% 5-fluorouracil cream for three weeks. Six months later,
a 3-mm ulcer develops in the right infraorbital region and enlarges to 1 cm over
three months.

Which of the following is the most likely diagnosis?

(A) Basal cell carcinoma


(B) Bowen’s disease
(C) Keratoacanthoma
(D) Seborrheic keratosis
(E) Squamous cell carcinoma

The correct response is Option E.

This 60-year-old farmer with a 1-cm ulcerated lesion in the right infraorbital
region most likely has a squamous cell carcinoma. Actinic keratoses occur in
older, fair-skinned persons and are thought to represent the cumulative effects of
exposure to ultraviolet light. Treatment of actinic keratoses (typically with
administration of 5-fluorouracil) is recommended because of the potential for
malignancy; it has been estimated that 20% of patients with actinic keratoses will
eventually develop cutaneous squamous cell carcinoma. However, this type of
carcinoma rarely metastasizes.

Basal cell carcinomas can arise in this region and appear similar to but are less
common than squamous cell carcinoma. The rapid growth (enlarging to 1 cm over
a period of three months) would not be consistent with a basal cell carcinoma.

Bowen’s disease, or intraepithelial squamous cell carcinoma, occurs commonly in


both sun-exposed and non-sun-exposed areas in older persons. These are typically
solitary, sharply defined lesions that have red discoloration and
are covered with plaque. Potential causes of Bowen’s disease include exposure to
ultraviolet light or arsenic, viral infections, chronic trauma, and heredity.

Keratoacanthomas grow rapidly over several weeks and regress spontaneously


over four to six months if left untreated. Because they are have been shown to
progress to invasive or metastatic carcinoma in rare instances, excision for
histopathologic confirmation is recommended. They are initially firm, round,
flesh-colored or reddish solitary papules, then progress to dome-shaped nodules
with a smooth, shiny surface and an umbilicated center with a keratin plug. The
face, neck, and dorsal aspect of the arms are affected most commonly; lesions on
the trunk are rare.

Seborrheic keratoses are common lesions that occur on the trunk and upper and
lower extremities in middle-aged and older persons. They are sharply
circumscribed and have a waxy, greasy, or pressed-on appearance with a friable
hyperkeratotic surface. Pigmentation varies from mild to deep black. This is not a
premalignant condition.

References
1. Stal S, Spira M. Basal and squamous cell carcinoma of the skin. In: Aston SJ,
Beasley RW, Thorne CH, eds. Grabb & Smith’s Plastic Surgery. 5th ed.
Philadelphia, Pa: Lippincott-Raven; 1997:107-120.
2. Thomas JM. Premalignant and malignant epithelial tumors. In: Sams WM Jr,
Lynch PJ, eds. Principles and Practice of Dermatology. New York, NY: Churchill
Livingstone, Inc; 1996:225-239.
3. Zarem HA, Lowe NJ. Benign growths and generalized skin disorders. In: Aston
SJ, Beasley RW, Thorne CH, eds. Grabb & Smith’s Plastic Surgery. 5th ed.
Philadelphia, Pa: Lippincott-Raven; 1997:141-159.

A 59-year-old man is undergoing follow-up evaluation one year after excision of a


recurrent basal cell carcinoma involving the skin of the cheek. Current
examination shows a slight bulge at the site of excision that is firm to palpation
and is covered by a well-healed scar.

Which of the following is the most appropriate initial management?

(A) Observation
(B) Antibiotic therapy
(C) Massage therapy
(D) Incision and drainage
(E) Incisional biopsy

The correct response is Option E.

The recurrence rate following initial treatment of a basal cell carcinoma smaller
than 2 cm is 10%. This rate increases to 25% following treatment of recurrent
tumors and can be as high as 82% if tumor excision is incomplete. Recurrence rate
is highest for tumors of the periorbital, periauricular, and perinasal regions.
Morpheaform basal cell carcinoma, which is characterized by indistinct margins,
is also prone to recurrence. Any patient who has had one recurrent tumor should
be considered at risk for a second recurrent tumor.

Because of the risk for recurrence, any patient who has had a basal cell carcinoma
(and especially those with recurrent carcinomas) should undergo frequent follow-
up examinations. A biopsy should be performed if a new abnormality is detected
at the site of previous excision; therefore, this patient who has a firm bulge on the
cheek at the site of a previously excised basal cell carcinoma should initially
undergo incisional biopsy. Any treatment should be deferred until the diagnosis is
made. In this patient, a recurrent tumor will most likely be ruled out following
histologic examination of the biopsy specimen. A fluid collection may be
associated with the adenoid cystic variety of basal cell carcinoma. If recurrent
basal cell carcinoma is excluded, the most appropriate management would be
needle aspiration of the fluid and open drainage and packing of the wound or
placement of a drain. If a recurrent carcinoma is found, treatment would involve
surgery, radiation therapy, or a combination of both.

Antibiotics should be administered only to patients with proven infection. Massage


therapy is useful for management of contracted scar tissue but is not indicated as
initial management of a patient in whom a recurrent tumor may be likely.

References
1. Dinehart SM, Jansen GT. Cancer of the skin. In: Myers EN, Suen JY, eds.
Cancer of the Head and Neck. Philadelphia, Pa: WB Saunders Co; 1996:143-159.
2. Stal S, Spira M. Basal and squamous cell carcinoma of the skin. In: Aston SJ,
Beasley RW, Thorne CH, eds. Grabb & Smith’s Plastic Surgery. 5th ed.
Philadelphia, Pa: Lippincott-Raven; 1997:107-120.

Which of the following lesions has been shown to result from sun exposure?

(A) Dermatofibroma
(B) Ephelides
(C) Lentigines
(D) Seborrheic keratoses
(E) Xanthelasma

The correct response is Option C.

Lentigines are benign pigmented macules that result from increased activity of
epidermal melanocytes; these lesions do not fade in the absence of sun exposure.
The two primary types of lentigines are simple, which can occur in any area and
usually develop during childhood, and solar, also known as senile, which occur in
sun-exposed areas such as the face, dorsal aspect of the hands and forearms, and
upper trunk.

Dermatofibromas are common benign, fibrous, papular lesions smaller than 1 cm


that develop on the trunk and upper or lower extremities in early and middle
adulthood. These lesions may be induced by minor trauma and are more common
in women than in men. It is important to differentiate them from
dermatofibrosarcoma protuberans, which is a malignant tumor. Histologic
examination of a biopsy specimen of a dermatofibroma will show localized
proliferations of fibrohistiocytes within the dermis.

Ephelides are common pigmented freckles that are not related to sun exposure.
They contain a normal quantity of melanocytes, but the amount of melanin within
the epidermal cells is increased.

Seborrheic keratoses are benign keratinocytic tumors seen in patients older than
age 30 years. They exhibit a classic waxy, greasy, or pressed-on appearance and
may have a generalized distribution.

Xanthelasma involves multiple, soft, yellow-orange plaques that occur around the
eyes as a result of deposition of lipid-laden macrophages. Approximately 30% of
patients with xanthelasma have a disorder of lipid metabolism.

References
1. Pelc NJ, Nordlund JJ. Pigmentary changes in the skin: an introduction for
surgeons. Clin Plast Surg. 1993;20:53-65.
2. Schaffer JV, Bolognia JL. The clinical spectrum of pigmented lesions. Clin
Plast Surg. 2000;27:391-408.

A 45-year-old man has had a painful lesion on the left ear for the past six months.
The patient says that he typically sleeps on his left side. Physical examination
shows a nodule on the antitragus. Histologic examination of a shave biopsy
specimen shows no evidence of malignancy.

Which of the following is the most likely diagnosis?

(A) Acrochordon
(B) Chondrodermatitis nodularis helicis
(C) Dermatofibroma
(D) Sebaceous hyperplasia
(E) Xanthelasma

The correct response is Option B.

The most likely diagnosis is chondrodermatitis nodularis helicis, also known as


chondrodermatitis nodularis chronica helicis, a benign lesion of the ear that
usually occurs in men older than age 40 years. This is a painful erythematous
nodule that typically involves the helix or antihelix and often occurs on the side
that the patient prefers to sleep. Although intralesional injection of corticosteroids
may be curative, shave excision or direct excision is usually required to exclude
the possibility of cutaneous malignancy.
Acrochordons are simple skin tags that occur more frequently with age.

A dermatofibroma is a common benign, fibrous, papular lesion usually seen on the


trunk and upper or lower extremities in early and middle adulthood. This lesion is
more common in women than in men.

Patients with sebaceous hyperplasia have small, yellowish lesions that usually
occur on or around the face.

Xanthelasma involves multiple soft yellow-orange plaques, typically in the


preorbital area, that result from deposition of lipid-laden macrophages.
Approximately 30% of affected patients have a disorder of lipid metabolism.

References
1. Graham GF. Cryosurgery. Clin Plast Surg. 1993;20:131-147.
2. Harris AO, Levy ML, Goldberg LH, et al. Nonepidermal and appendageal skin
tumors. Clin Plast Surg. 1993;20:115-130.
3. Morganroth GS, Leffell DJ. Nonexcisional treatment of benign and
premalignant cutaneous lesions. Clin Plast Surg. 1993;20:91-104.

Soft Tissue Infection

In a patient with infection of the index finger, drainage is most likely to flow
proximally into which of the following spaces?

(A) Midpalmar space


(B) Parona’s space
(C) Radial bursa
(D) Thenar space
(E) Ulnar bursa

The correct response is Option D.


Infections of the index finger drain from the flexor sheath proximally into the
thenar space. Infections of the long, ring, and small fingers drain into the
midpalmar space. The radial and ulnar bursae communicate by Parona’s space,
which lies deep to the pronator muscle.

References:
1. Jebson PJ. Deep subfascial space infections. Hand Clin. 1998;14(4):557-566.
2. Lille S, Hayakawa T, Neumeister MW, et al. Continuous postoperative catheter
irrigation is not necessary for the treatment of suppurative flexor tenosynovitis. J
Hand Surg [Br]. 2000;25(3):304-307.
3. Gutowski KA, Ochoa O, Adams WP Jr. Closed-catheter irrigation is as effective
as open drainage for treatment of pyogenic flexor tenosynovitis. Ann Plast Surg.
2002;49(4):350-354.

Tissue Expansion

A 65-year-old woman who has undergone multiple resections for recurrent lentigo
maligna melanoma with repeated central advancements of the remaining scalp via
skin grafting over the past 10 years has frequent irritation of the skin at the
grafting sites (shown above). No evidence of further disease has been noted over
the past four years. Which of the following is the most appropriate management at
this time?

(A) Full-thickness skin grafting


(B) Micrograft hair transplantation
(C) Rerotational flap advancement
(D) Staged excision and primary closure
(E) Tissue expansion

The correct response is Option E.

Because of continued skin breakdown in this area of postsurgical alopecia, tissue


expansion followed by adjacent tissue transfer is the most appropriate surgical
procedure at this time. This patient has had multiple excisions with repeated
advancement followed by skin grafting. The remaining native vessels should be
sufficient to supply the expanded scalp skin. Tissue expansion and coverage are
usually tolerated well by patients of this age.
Hair transplantation is unlikely to be successful in this patient because of extensive
scars in the area. Staged excision of the prior skin grafts and primary closure will
not improve this patient’s alopecia and are unlikely to relieve the skin irritation
caused by the thinned skin over the calvaria. Because advancement has already
been done, little additional advancement can be achieved without back-grafting
exposed areas. Full-thickness grafts will not allow sufficient hair growth or
replacement.

References:
1. Argenta LC, Marks MW, Pasyk DA. Advances in tissue expansion. Clin Plast
Surg. 1985;12:159.
2. Oishi SN, Luce EA. The difficult scalp and skull wound. Clin Plast Surg.
1995;22:51-59.

Wound Healing

Which of the following processes of healing provides maximal tensile strength of


a wound?

(A) Accumulation of collagen


(B) Addition of sugar moieties
(C) Hydroxylation of lysine
(D) Hydroxylation of proline
(E) Molecular cross-linking

The correct response is Option E.

Intramolecular and intermolecular cross-linking between collagen fibers accounts


for the maximal tensile strength of a wound. Maximal strength occurs during the
remodeling phase of wound healing. Peak increase in tensile strength occurs three
to six weeks after injury but approaches maximal after about three months when it
achieves up to 80% of the normal skin strength.

Collagen synthesis peaks at about three weeks, and collagen accumulates to its
maximum at six weeks; however, intramolecular and intermolecular cross-linking
between collagen fibers provides the tensile strength of the wound.

The addition of sugar moieties occurs just before cleavage of amino and carboxy
terminal ends. After this, the molecules are termed collagen, which then develops
further intermolecular and intramolecular bonds for strength.

The hydroxylation of lysine and proline in the endoplasmic reticulum of the


fibroblasts is a crucial step in collagen production and is important in future
intermolecular cross-linking. However, this step occurs much earlier in wound
healing, primarily during the proliferative phase.

References:
1. Peacock EE, Cohen IK. Wound healing. In: McCarthy JG, May JW, Littler JW,
eds. Plastic Surgery. Vol 1. Philadelphia: WB Saunders; 1990:161-185.
2. Lawrence TH. Physiology of the acute wound. Clin Plast Surg. 1998;25:321-
340.
3. Monaco JL, Lawrence TH. Acute wound healing: an overview. Clin Plast Surg.
2003;30:1-12.

Which of the following types of cells has been shown to mediate wound
contraction?

(A) Epithelial cells


(B) Lymphocytes
(C) Macrophages
(D) Myofibroblasts
(E) Polymorphonuclear cells

The correct response is Option D.

Myofibroblasts, described by Gabbiani in 1971, are thought by most people to


mediate wound contraction. They are derived from fibroblasts in the wound, which
under conditions of stress elongate and show features of a myocyte. Through
interaction with the matrix, they effectively retract collagen fibrils. Various
mediators such as transforming growth factor-beta (TGF-_) and platelet-derived
growth factor (PDGF) are involved in the process. Myofibroblasts first appear in
the wound by the third day after injury and persist for approximately 21 days, after
which time they slowly disappear. They persist longer in open contracting wounds.

Epithelial cells are required to cover a wound but play no role in the wound
contraction process. Polymorphonuclear cells, lymphocytes, and macrophages are
leukocytes involved in the inflammatory response to injury.

References:
1. Fine NA, Mustoe TA. Wound healing. In: Greenfield LJ, ed. Surgery: Scientific
Principles and Practice. Vol 1. 2nd ed. Philadelphia: Lippincott Raven Publishers;
1997:67-83.
2. Glat PM, Longaker MT. Wound healing. In: Aston SJ, Beasley RW, Thorne
CH, eds. Grabb & Smith’s Plastic Surgery. 5th ed. Philadelphia: Lippincott
Williams & Wilkins; 1997:3-12.
3. Lawrence TW. Wound healing biology and its application to wound
management. In: O’Leary JP, Capote LR, eds. The Physiologic Basis of Surgery.
Baltimore: Lippincott Williams & Wilkins; 1996:118-140.

Which of the following types of collagen is most abundant in a healed scar?

(A) I
(B) II
(C) III
(D) IV
(E) V

The correct response is Option A.

The most abundant type of collagen in a healed scar is Type I. This type is the
most abundant collagen in the body, including the skin. Type II collagen is found
predominantly in cartilage and vitreous. Type III collagen is the second most
abundant collagen in a healed scar. It also exists in elastic tissues, such as blood
vessels. Type IV collagen is located mainly in the basement membranes. Type V
collagen is widespread.

References:
1. Fine NA, Mustoe TA. Wound healing. In: Greenfield LJ, ed. Surgery: Scientific
Principles and Practice. Vol 1. 2nd ed. Philadelphia: Lippincott-Raven; 1997:67-
83.
2. Glat PM, Longaker MT. Wound healing. In: Aston SJ, Beasley RW, Thorne
CH, eds. Grabb & Smith’s Plastic Surgery. 5th ed. Philadelphia: Lippincott
Williams & Wilkins; 1997:3-12.

Which of the following is the predominant cell responsible for the intermediate
phase of wound healing and collagen synthesis (days 3 through 21)?

(A) Erythrocyte
(B) Fibroblast
(C) Myoepithelial cell
(D) Neutrophil
(E) Platelet
The correct response is Option B.

The intermediate phase of wound healing begins on the second or third day after
injury and continues until approximately 21 days after injury. This phase begins
with chemotaxis and proliferation of mesenchymal cells, angiogenesis, and
epithelialization. Ultimately, collagen synthesis, wound contraction, and
proteoglycan synthesis predominate in this phase; fibroblasts and macrophages are
the primary cells involved. Before this phase, the primary effects of wound healing
involve hemostasis and inflammation. Initially, the cellular elements involved in
this initial phase are erythrocytes and platelets. Neutrophils are the first of the
leukocytes found in the area and are mobilized not long after the erythrocytes and
platelets. After approximately 21 days, wound remodeling permeates the overall
healing environment. This phase is said to end after approximately one year,
although wound remodeling is actually a lifelong process.

References:
1. Lawrence WT. Physiology of the acute wound. Clin Plast Surg. 1998;25:321-
340.
2. Monaco JL, Lawrence WT. Acute wound healing: an overview. Clin Plast Surg.
2003;30:1.
3. Steed DL. Modifying the wound healing response with exogenous growth
factors. Clin Plast Surg. 1998;25:397.

Which of the following interventions is LEAST likely to improve the appearance


of a hypertrophic scar?

(A) Application of silicone gel sheeting


(B) Application of vitamin E gel
(C) Intralesional injection of a corticosteroid
(D) Pressure therapy
(E) Prolonged application of paper tape

The correct response is Option B.

Application of vitamin E products is popular in the skin-care industry despite the


paucity of scientific evidence about its effectiveness. Some animal models have
demonstrated improvement in healing of radiation-induced wounds with vitamin
E. However, no studies have shown clear-cut improvement in hypertrophic or
normal scars. In fact, the only controlled study showed no benefit. Localized
dermatitis may occur with application of vitamin E products.

Although various treatments have been used to improve the appearance and
texture of hypertrophic scars, no single method has shown uniform success.
Response rates greater than 50% are considered successful. Application of silicone
gel sheeting has shown significant improvement in fibroproliferative scars in
several controlled trials, although the mechanism is unknown.

Intralesional injection of triamcinolone and other corticosteroids typically have a


response rate greater than 50% but can cause skin atrophy, depigmentation,
telangiectasis, and pain.

Pressure therapy has been used to manage keloids and hypertrophic scars since the
early 1970s. The use of pressure garments (specially fitted elastic garments often
with silicone inserts) to treat postburn scarring and contractures is a standard of
care.

Application of adhesive microporous tape to fresh surgical wounds has been


endorsed by an international panel on scar management. Uncontrolled clinical
trials have shown its efficacy. The mechanism is unknown but may be similar to
the action of silicone gel sheeting.

References:
1. Havlik RJ. Vitamin E and wound healing: safety and efficacy reports. Plast
Reconstr Surg. 1997;100:1901-1902.
2. Mustoe TA, Cooter RD, Gold MH, et al. International clinical recommendations
on scar management. Plast Reconstr Surg. 2002;110:560-571.
3. Rahban SR, Garner WL. Fibroproliferative scars. Clin Plast Surg. 2003;30:77-
89.

You might also like